Practice questions Flashcards

1
Q
A 43-year-old man presents with a painless lump in his groin that appeared 2 weeks ago. He claims that the lump protrudes on defecation and he also reports becoming constipated recently. On examination, the lump is reduced, and a finger is placed over the midpoint of the inguinal ligament. When the patient is asked to cough, the lump reappears. What is the most likely diagnosis? 
A Direct inguinal hernia 
B  Indirect inguinal hernia 
C  Femoral hernia 
D  Obturator hernia 
E  Spigelian hernia
A

A

How well did you know this?
1
Not at all
2
3
4
5
Perfectly
2
Q
Which of the following murmurs is associated with severe aortic regurgitation? 
A Austin-Flint
B Graham-Steell 
C Gibson
D Carey-Coombs 
E Barlow
A

A

How well did you know this?
1
Not at all
2
3
4
5
Perfectly
3
Q
A 67-year-old man is brought into A&E having been involved in a road traffic accident. On examination, he opens his eyes to pain, makes a few grunting noises and withdraws his legs from painful stimuli. What is his GCS? 
A2 
B4 
C6 
D8 
E 10
A

D

How well did you know this?
1
Not at all
2
3
4
5
Perfectly
4
Q

A 2-year-old boy is brought to the GP after his father noticed some swelling around his eyes. On examination, there is periorbital and pedal oedema. A urine dipstick is positive for proteins and negative for blood. What is the most likely diagnosis?
A IgA nephropathy
B Membranous glomerulonephritis
C Rapidly progressive glomerulonephritis
D Minimal change glomerulonephritis
E Henoch-Schӧnlein purpura

A

D

How well did you know this?
1
Not at all
2
3
4
5
Perfectly
5
Q

A 54-year-old man is brought into A&E with a suspected acute coronary syndrome. An ECG is performed, which reveals ST elevation in leads I, aVL, V5 and V6. Which coronary artery has been occluded?
A Left main stem
B Left anterior descending coronary artery
C Left circumflex coronary artery
D Right coronary artery
E Posterior descending artery

A

C

How well did you know this?
1
Not at all
2
3
4
5
Perfectly
6
Q
A 46-year-old housewife visits her GP complaining of pain in the joints of her hands that has gradually got worse over 3 months. It has started affecting her ability to complete daily tasks such as cooking for her children. Both of her hands are affected equally and the pain and stiffness is worst in the morning but gets better when she starts using her hands. On closer inspection, her hands do not appear to be deformed although her metacarpophalangeal joints and proximal interphalangeal joints appear slightly swollen, warm and tender. 
What is the most likely diagnosis? 
A  Reactive arthritis 
B  Osteoarthritis 
C  Rheumatoid arthritis 
D  Psoriatic arthritis 
E  Septic arthritis
A

C

How well did you know this?
1
Not at all
2
3
4
5
Perfectly
7
Q
A 56-year-old man with a history of alcoholism complains of intermittent epigastric pain that radiates through to his back. When questioned, he admits to losing about 3 kg in weight over the past 6 months and says that his stools have become pale and difficult to flush away. Which investigation would you request to aid the diagnosis? 
A  Serum amylase 
B  Blood cultures 
C  Faecal elastase 
D  CA 19-9 
E  OGD
A

C

How well did you know this?
1
Not at all
2
3
4
5
Perfectly
8
Q
A 25-year-old female has suffered from shortness of breath over the past 2 months. She gets particularly breathless when she exerts herself, and has had to stop going on her morning jog. She has not experienced a cough, fever or chest pain. She has no past medical history of note, however, her periods have become quite heavy over the past 3 or 4 months. What is the most likely diagnosis? 
A Hyperthyroidism 
B Anaemia
C Pneumonia
D COPD 
E Asthma
A

B

How well did you know this?
1
Not at all
2
3
4
5
Perfectly
9
Q

A 46-year-old man, with a history of type 1 diabetes, visits the GP for an HbA1c reading. He has recently been feeling more tired than usual and has noticed that the skin on his hands has become darker over the past few months. On examination, hepatomegaly and a tanned complexion (despite not having been on any recent holidays) are noted. Haemochromatosis is suspected and iron studies are requested. Which set of results would be consistent with haemochromatosis?
A High serum iron, high ferritin, high transferrin, low transferrin saturation, low TIBC
B High serum iron, low ferritin, low transferrin, high transferrin saturation, low TIBC
C High serum iron, high ferritin, high transferrin, high transferrin saturation, low TIBC
D High serum iron, high ferritin, low transferrin, high transferrin saturation, high TIBC
E High serum iron, high ferritin, low transferrin, high transferrin saturation, low TIBC

A

E

How well did you know this?
1
Not at all
2
3
4
5
Perfectly
10
Q
10. A 72-year-old woman has recently suffered a fracture of her right distal radius after falling on an outstretched hand. She is at high risk of osteoporosis because she is post-menopausal and has undergone several decades of steroid treatment for her asthma. A DEXA scan is performed. Which result would be diagnostic of 
osteoporosis? 
A  T score of -1.5 or worse 
B  T-score of -2 or worse 
C  T-score of -2.5 of worse 
D  T-score of -3 or worse 
E  T-score of -3.5 or worse
A

C

How well did you know this?
1
Not at all
2
3
4
5
Perfectly
11
Q
A 52-year-old man was watching TV yesterday when he suddenly become very aware of his heart beating rapidly. This lasted around 45 mins and then subsided spontaneously. It has happened several times over the past 2 months. An ECG reveals no abnormalities. However, due to the strong suspicion of atrial fibrillation, the patient is placed on a 24-hr tape, which confirms the diagnosis. Which scoring system should be used to determine the benefit of long-term anticoagulation in this patient? 
A  QRISK2 score 
B  ABCD2 Score 
C  GRACE score 
D  CHA2DS2-VASc score 
E  CURB-65 score
A

D

How well did you know this?
1
Not at all
2
3
4
5
Perfectly
12
Q
Which of the following is not a feature of background diabetic retinopathy? 
A  Hard exudates 
B  Soft exudates 
C  Microaneurysms 
D  Blot haemorrhages 
E  Leakage of lipids from blood vessels
A

B

How well did you know this?
1
Not at all
2
3
4
5
Perfectly
13
Q
A 53-year-old man, who has recently recovered from a diarrhoeal illness, comes to A&E with a 1-week history of gradually worsening weakness in his legs. A neurological examination reveals reduced tone, reduced reflexes and impaired sensation in both lower limbs. He adds that the weakness began in his feet and has gradually progressed up his legs. Guillain-Barré syndrome is suspected. Which of the following parameters should be closely monitored in this patient? 
A  Body temperature 
B  Serum osmolality 
C  Serum potassium 
D  Forced vital capacity 
E  Urine output
A

D

How well did you know this?
1
Not at all
2
3
4
5
Perfectly
14
Q

A 14-year-old school boy was diagnosed with asthma 6 months ago. He was
given a salbutamol inhaler to use PRN, however, he continued to have regular
episodes of breathlessness. He was started on a regular inhaled corticosteroid
(beclomethasone) 3 months ago. Although the frequency of his attacks has reduced
with the medication, he is still experiencing bouts of breathlessness about 4 times
per week. What would be the next most appropriate step in his management?
A Increase the dose of salbutamol
B Add an inhaled corticosteroid
C Add an oral corticosteroid
D Add a long-acting beta agonist
E Montelukast therapy

A

B

How well did you know this?
1
Not at all
2
3
4
5
Perfectly
15
Q
Which of the following is the most common cause of chronic kidney disease? 
A  Hypertension 
B  Diabetes mellitus 
C  Glomerulonephritis 
D  Pyelonephritis 
E  Polycystic kidney disease
A

B

How well did you know this?
1
Not at all
2
3
4
5
Perfectly
16
Q
A 46-year-old woman has been suffering from frequent headaches over the past 4 months, along with some blurring of vision. She also mentions that she has developed a fiercely itchy rash on several occasions, usually occurring soon after she has had a bath. What is the most likely diagnosis? 
A  Anaemia 
B  Acute lymphoblastic leukaemia 
C  Chronic myeloid leukaemia 
D  Polycythaemia vera 
E  Myelofibrosis
A

D

How well did you know this?
1
Not at all
2
3
4
5
Perfectly
17
Q

A 27-year-old female comes to A&E complaining of severe right iliac fossa pain. She has vomited three times whilst waiting to be seen. On examination, there is rebound and percussion tenderness in the right iliac fossa and the pain gets worse
when the doctor extends the patient’s hip. What is the name of the sign being elicited?
A Murphy’s sign
B Cope sign
C Rovsing’s sign
D Psoas sign
E Aarons sign

A

D

How well did you know this?
1
Not at all
2
3
4
5
Perfectly
18
Q
A 74-year-old man presents with a skin lesion on his left cheek, which has gradually grown over 4 months. On closer examination, the lesion has raised, everted edges with an ulcerated centre revealing a keratotic core. What is the most likely diagnosis? 
A  Basal cell carcinoma 
B  Squamous cell carcinoma 
C Melanoma
D Keratoacanthoma 
E Actinic keratosis
A

B

How well did you know this?
1
Not at all
2
3
4
5
Perfectly
19
Q

A 53-year-old man presents to A&E with severe pain in his right flank that radiates to his right groin. Ureteric colic is suspected and a CT-KUB is requested. The CT-KUB confirms the diagnosis but it also shows an abdominal aortic aneurysm with a diameter of 4.7 cm. When questioned, the patient denies any back pain (other than the pain caused by ureteric colic) or symptoms of vascular disease. What is the most appropriate management option for this patient?
A Reassure and discharge
B Surveillance with an ultrasound scan every 1 year
C Surveillance with an ultrasound scan every 6 months
D Surveillance with an ultrasound scan every 3 months
E Surgical repair of the aneurysm

A

D

How well did you know this?
1
Not at all
2
3
4
5
Perfectly
20
Q

A 38-year-old man is complaining of excessive thirst and frequent urination over the past month. He describes the thirst as being ‘insatiable’, and claims to drink about 8-12 litres of water every day. He undergoes a water deprivation test, which revealed the following results:
Urine osmolality
2 hr : 212 mOsm/kg 4 hr: 227 mOsm/kg
6 hr : 221 mOsm/kg 8 hr: 242 mOsm/kg
Following administration of 2 mg IM desmopressin (DDAVP) : 278 mOsm/kg (normal
> 600 mOsm/kg).
What is the most likely diagnosis?

A Diabetes mellitus type 1
B  Central diabetes insipidus 
C  Nephrogenic diabetes insipidus 
D  Psychogenic polydipsia 
E Hyperparathyroidism
A

C

How well did you know this?
1
Not at all
2
3
4
5
Perfectly
21
Q
A 37-year-old man visits the GP because his partner has recently noticed some dark patches on his back. On examination, there are multiple painless red papules on his back, some of which have merged to form purple plaques. He was diagnosed with HIV 8 years ago. What is the most likely diagnosis? 
A  Discoid lupus 
B  Guttate psoriasis 
C  Kaposi’s sarcoma 
D  Shingles 
E  Chronic mucocutaneous candidiasis
A

C

How well did you know this?
1
Not at all
2
3
4
5
Perfectly
22
Q
A 16-year-old school girl has been suffering from depression for 6 months. Last night (12 hours ago) she decided to end her life and ingested 40 paracetamol tablets. She has since decided that she has made a mistake and does not want to end her life. She appears reasonably well when she presents to A&E, only complaining of some mild nausea. What is the most appropriate treatment option? 
A  N-acetylcysteine 
B  Naloxone 
C  Flumezanil 
D  Atropine 
E  Sodium bicarbonate
A

A

How well did you know this?
1
Not at all
2
3
4
5
Perfectly
23
Q

A 7-year-old girl has had severe, bloody diarrhoea with cramping abdominal pain for the past 3 days. She has also been emptying her bladder less and less frequently. Blood tests, including a blood film, are performed which revealed a low
Hb, low platelets and the presence of schistocytes. U&Es are also performed: Creatinine : 182 micromol/L (baseline = 92 micromol/L)
Urea : 9.2 mmol/L (2.5-6.7)
A stool culture identifies E. coli O157. What is the most likely diagnosis?

A  Microangiopathic haemolytic anaemia 
B  Haemolytic uraemic syndrome 
C  Thrombotic thrombocytopaenic purpura 
D Disseminated intravascular coagulation 
E Gastroenteritis
A

B

How well did you know this?
1
Not at all
2
3
4
5
Perfectly
24
Q

A 37-year-old plumber presents to his GP with a week-long history of headache, diarrhoea and fever. In the last 2 days, he has also developed a dry cough and his wife has commented that he has appeared to be quite confused recently. What is the most appropriate investigation?

A  CT head scan 
B  Urinary antigens 
C  Stool culture 
D  LFTs 
E  U&Es
A

B

How well did you know this?
1
Not at all
2
3
4
5
Perfectly
25
Q
Which system is used to stage prostate cancer? 
A  Breslow thickness 
B  Gleason staging 
C  GRACE score 
D  W ells score 
E  Dukes’ staging
A

B

How well did you know this?
1
Not at all
2
3
4
5
Perfectly
26
Q

A 28-year-old girl presents with a 5-day history of urinary frequency, dysuria and mild suprapubic pain. Urine dipstick is positive for nitrites and leucocytes. A urinary tract infection is suspected and an MSU is taken and sent to the lab. Whilst awaiting culture and sensitivities, which empirical treatment option is best for this patient?

A Trimethoprim 
B Benzylpenicillin 
C Vancomycin
D Metronidazole 
E Levofloxacin
A

A

How well did you know this?
1
Not at all
2
3
4
5
Perfectly
27
Q

A 68-year-old male visits his GP complaining of constipation, rectal bleeding and itchiness around his anus. He often feels ‘a lump’ hanging out after defecating which he has to push back in himself. On examination, anal tone is weak and a protruding mass is felt which has palpable muscular rings. What is the most likely diagnosis?

A Grade 3 haemorrhoids
B  Grade 4 haemorrhoids 
C  Perianal abscess 
D  Type 1 rectal prolapse 
E  Type 2 rectal prolapse
A

E

How well did you know this?
1
Not at all
2
3
4
5
Perfectly
28
Q
A 54-year-old man is complaining of sharp, central chest pain that has arisen over the last 24 hours. On inspection, the patient is sitting forward on the examination couch. On auscultation, a scratching sound is heard – loudest over the lower left sternal edge, when the patient is leaning forward. He also has a low-grade fever. He has a past medical history of a ST-elevation MI which was diagnosed, and treated with PCI, 6 weeks ago. What is the most likely diagnosis? 
A  Viral pericarditis 
B  Constrictive pericarditis 
C  Cardiac tamponade 
D  Dressler syndrome 
E  Tietze syndrome
A

D

How well did you know this?
1
Not at all
2
3
4
5
Perfectly
29
Q
A 66-year old man has been experiencing pain in his right calf for the last 6 months. Initially, the pain would come about whenever he went for a walk, and it would be relieved by rest. However, over the last few weeks he has experienced pain during rest. The pain is particularly bad at night, and he gets some relief from dangling his leg over the end of the bed. Recently, he has noticed a small, elliptical ulcer appear in between his toes on his right foot. He has a past medical history of ischaemic heart disease, and underwent a CABG 8 years ago. What is the most likely diagnosis? 
A  Intermittent claudication 
B  Critical limb ischaemia 
C  Acute limb ischaemia 
D  Leriche syndrome 
E  Chronic deep vein thrombosis
A

B

How well did you know this?
1
Not at all
2
3
4
5
Perfectly
30
Q
A 3-year-old boy is brought, by his father, to see his GP because he has had a fever for the past 4 days and has cried in pain every time he has tried to eat. On examination, the patient’s gums look red and swollen and there are small vesicles and ulcerations along the gumline. The GP suspects gingivostomatitis. What is the most likely cause? 
A Varicella
B  Primary HSV1 infection 
C  Reactivation of HSV1 
D  HSV2 infection 
E  Infectious mononucleosis
A

B

How well did you know this?
1
Not at all
2
3
4
5
Perfectly
31
Q
A 31-year-old man presents to his GP with a 2-day history of central chest pain, which gets worse when he breathes in and when exercising. Cardiovascular and respiratory examinations detect no abnormalities, however, the patient winces in pain when the GP palpates for heaves and thrills. What is the most likely diagnosis? 
A  Pulmonary embolism 
B  Myocarditis 
C  Tension pneumothorax 
D  Costochondritis 
E  Pleurisy
A

D

How well did you know this?
1
Not at all
2
3
4
5
Perfectly
32
Q
Which of the following is unlikely to cause pleuritic chest pain? 
A  Tension pneumothorax 
B  Rib fracture 
C  Pulmonary fibrosis 
D  Pneumonia 
E  Pericarditis
A

C

How well did you know this?
1
Not at all
2
3
4
5
Perfectly
33
Q
A 28-year-old PhD student books an appointment to see her GP about some small lumps on her groin. On closer inspection, there are multiple small, firm, dome- shaped lumps with an umbilicated centre. On direct questioning, she reveals that she has recently had a new sexual partner. What is the most likely diagnosis? 
A  Molluscum contagiosum 
B  Varicella zoster 
C  Syphilis 
D  Gonorrhoea 
E  Sebaceous cysts
A

A

How well did you know this?
1
Not at all
2
3
4
5
Perfectly
34
Q
A 71-year-old woman presents to A&E with a headache that has gradually been getting worse over the past week. The pain is localised over the left half of her forehead and does not radiate. She has also been eating less frequently as her jaw becomes painful when she chews her food. On direct questioning, she admits to experiencing some stiffness and pain in her shoulders over the past 6 months. On examination, she has a thickened, non-pulsatile temporal artery. What is the first step in her management? 
A  Check ESR 
B  Temporal artery biopsy 
C  IV hydrocortisone 
D  Oral prednisolone 
E  IV antibiotics
A

D

How well did you know this?
1
Not at all
2
3
4
5
Perfectly
35
Q
A 24-year-old female, who has recently returned from a 3-week trip to Vietnam, complains that she has been feeling ‘under the weather’ with fevers and joint pain. On direct questioning, she reveals that she had unprotected sexual intercourse with a stranger whilst in Vietnam. She is jaundiced and has right upper quadrant tenderness. Hepatitis B serology is requested. The results are shown below: 
HBsAg +
HBeAg -
HBcAb IgM +
HBcAb IgG +
HBsAb -
What is the hepatitis status of this patient? 
A  Acute infection 
B  Chronic infection 
C  Cleared 
D  Vaccinated 
E  Susceptible
A

A

How well did you know this?
1
Not at all
2
3
4
5
Perfectly
36
Q
6. An 18-year-old female is brought to A&E, by ambulance, having been involved in a road traffic accident. She has bled significantly and needs an urgent blood transfusion. Her blood group is AB+. Which of the following blood groups will she be 
able to accept? 
A A+
B AB- 
C B- 
D O-
E All of the above
A

E

How well did you know this?
1
Not at all
2
3
4
5
Perfectly
37
Q
Which clinical test can be used to diagnose ankylosing spondylitis? 
A  Schober's test 
B  Schirmir's test 
C  Buerger's test 
D  Weber's test 
E  Tensilon test
A

A

How well did you know this?
1
Not at all
2
3
4
5
Perfectly
38
Q
A 53-year-old Afro-Caribbean man visits the GP to have his blood pressure measured. He has a history of hypertension and has been taking Amlodipine for 6 months. His blood pressure is 162/110 mm Hg. The GP is not satisfied with his blood pressure control and wants to step up his management. Which medication should be added? 
A Verapamil
B Spironolactone
C Bendroflumethiazide 
D Doxazosin
E Enalapril
A

E

How well did you know this?
1
Not at all
2
3
4
5
Perfectly
39
Q
An 81-year-old man has been urinating about 12 times every day, including at night, and has difficulty starting a stream, which he describes as being ‘very weak’. He has also suffered from lower back pain over the past month. A DRE is performed, revealing an asymmetrically enlarged, nodular prostate gland. Which investigation is most likely to provide a definitive diagnosis? 
A  PSA 
B  Acid phosphatase 
C  Transrectal ultrasound-guided biopsy 
D  CT Scan 
E  Isotope bone scan
A

C

How well did you know this?
1
Not at all
2
3
4
5
Perfectly
40
Q
  1. A 61-year-old man is brought to A&E by his daughter as he has become increasingly breathless over the past 24 hours and he has been coughing up a large amount of green sputum. He has a past medical history of COPD. Arterial blood gases are requested which show the following results (on room air):
    pH : 7.33 (7.35-7.45)
    PaO2 : 6.7 kPa (> 10.6 kPa on air)

PaCO2 : 9.6 kPa (4.7 - 6 kPa on air) HCO3- : 33 mmol/L (22 – 28 mmol/L) Respiratory Rate : 22 /min
What is the diagnosis?
A Partially compensated respiratory acidosis
B Fully compensated respiratory acidosis
C Partially compensated metabolic acidosis
D Fully compensated metabolic acidosis
E Acute type 1 respiratory failure

A

A

How well did you know this?
1
Not at all
2
3
4
5
Perfectly
41
Q
. A 73-year-old man has come to the outpatient clinic with his wife. She says that her husband seems very confused on some days and then seems completely normal on others. During the consultation, the patient appears confused with an AMTS of 4/10. He is distressed and claims that he can see little men running across the desk towards him. The doctor also notices a resting tremor. What is the most likely diagnosis? 
A  Lewy body dementia 
B  Alzheimer’s disease 
C  Depressive pseudodementia 
D  Frontotemporal dementia 
E  Vascular dementia
A

A

How well did you know this?
1
Not at all
2
3
4
5
Perfectly
42
Q

Which of the following is part of the diagnostic criteria for diabetes mellitus?
A Two fasting blood glucose > 7.8 mmol/L in an asymptomatic patient
B One fasting blood glucose > 11.1 mmol/L in an asymptomatic patient
C One random blood glucose > 11.1 mmol/L in a symptomatic patient
D Two random blood glucose > 7 mmol/L in an asymptomatic patient
E Glycosuria and ketonuria on urine dipstick

A

C

How well did you know this?
1
Not at all
2
3
4
5
Perfectly
43
Q

The red reflex is an important part of the ophthalmological examination. Which
of the following conditions can result in loss of the red reflex?
A Herpes simplex keratitis
B Cataract
C Astigmatism
D Conjunctivitis
E Aniridia

A

B

How well did you know this?
1
Not at all
2
3
4
5
Perfectly
44
Q

A 32-year-old man presents to his GP with an 8-month history of diffuse abdominal pain and frequent loose motions. He has also been passing blood with his stools. On examination, a red ring around the cornea is seen in both eyes. The patient is referred for a colonoscopy and biopsy. What would you expect the biopsy to show?
A Non-caseating granulomas
B Eosinophilic infiltration
C Villous atrophy and crypt hyperplasia
D High grade dysplasia and metaplastic columnar epithelium
E Mucosal ulcers, goblet cell depletion and crypt abscesses

A

E

How well did you know this?
1
Not at all
2
3
4
5
Perfectly
45
Q
A 28-year-old professional cyclist visits his GP complaining of headaches and blurred vision. He is worried that his symptoms will affect his performance in an important race in 3 weeks’ time. On direct questioning, he admits to taking ‘performance enhancers’ in preparation for his race. On examination, scratch marks are seen on his trunk. What is the most likely diagnosis? 
A  Thalassaemia 
B  Polycythaemia rubra vera 
C  Secondary polycythaemia 
D  Hodgkin’s lymphoma 
E  Non-Hodgkin’s lymphoma
A

C

How well did you know this?
1
Not at all
2
3
4
5
Perfectly
46
Q
A 22-year-old teacher visits her GP after fainting several times over the past 2 months. She does not experience any palpitations, light-headedness or auras before she faints, and she recovers very quickly. She has not bitten her tongue or become incontinent at any point. When questioned about the timing of these episodes, she reveals that she has only ever collapsed at work after she has been writing on the whiteboard for quite some time. On examination, a firm, immobile lump is palpated in her left supraclavicular fossa. What is the most likely diagnosis? 
A  Paroxysmal atrial fibrillation 
B  Transient ischaemic attack 
C  Atonic seizures 
D  Subclavian steal syndrome 
E  Vasovagal syncope
A

D

How well did you know this?
1
Not at all
2
3
4
5
Perfectly
47
Q

Which of the following triads best describes the main features of nephrotic syndrome?
A Proteinuria, Hypoalbuminaemia, Oedema
B Haematuria, Hypoalbuminaemia, Oedema
C Proteinuria, Haematuria, Hyperlipidaemia
D Proteinuria, Haematuria, Hypoalbuminaemia
E Frequency, Urgency, Dysuria

A

A

How well did you know this?
1
Not at all
2
3
4
5
Perfectly
48
Q
A 63-year-old man with ascending bilateral limb weakness and ascending paraesthesia is diagnosed with Guillain-Barré syndrome. 3 weeks prior to the onset of these symptoms he suffered from gastroenteritis. Which organism is most likely to have caused this infection? 
A  Salmonella 
B  Campylobacter jejuni 
C  E. coli 0157 
D  Rotavirus 
E  Entamoeba histolytica
A

B

How well did you know this?
1
Not at all
2
3
4
5
Perfectly
49
Q
A 79-year-old care home resident is admitted to hospital with a 4-day history of a cough productive of green sputum. She has also experienced some chest pain and shortness of breath. A chest X-ray shows an area of consolidation in the right middle lobe with a right-sided pleural effusion. What is the most appropriate treatment option? 
A  Co-amoxiclav and clarithromycin 
B  Co-trimoxazole 
C  Metronidazole 
D  Flucloxacillin 
E  Rifampicin and isoniazid
A

A

How well did you know this?
1
Not at all
2
3
4
5
Perfectly
50
Q
A 76-year-old care home resident has fractured his neck of femur having fallen out of bed. He is referred to the orthopaedic surgery department and undergoes an operation. Post-operatively, he is in considerable pain and is given 5 mg morphine sulphate. Which of these side-effects is he most likely to experience? 
A Constipation 
B Blurred vision 
C Cough 
D Tremor 
E Rash
A

A

How well did you know this?
1
Not at all
2
3
4
5
Perfectly
51
Q

A 65-year-old man, who is currently undergoing treatment for chronic lymphocytic leukaemia, presents with an extremely painful left great toe. On closer inspection, he has a fiercely inflamed left metatarsophalangeal joint. He has no other symptoms. What would you expect to see on analysis of the joint fluid aspirate?
A High WCC, turbid fluid
B Positively birefringent, rhomboid-shaped crystals
C Positively birefringent, needle-shaped crystals
D Negatively birefringent, rhomboid-shaped crystals
E Negatively birefringent, needle-shaped crystals

A

E

How well did you know this?
1
Not at all
2
3
4
5
Perfectly
52
Q
A 47-year-old woman has had several ‘dizzy spells’ over the past 6 weeks. She has been feeling very faint when getting out of bed in the morning and has also experienced some vague abdominal pain along with weight loss and lethargy. Examination reveals dark palmar creases and vitiligo on her back. What is the most appropriate investigation to request? 
A  Full blood count 
B  Fasting blood glucose 
C ECG 
D  Short synacthen test 
E  Thyroid function test
A

D

How well did you know this?
1
Not at all
2
3
4
5
Perfectly
53
Q
A 78-year-old woman visits her GP with a 4-month history of constipation and blood coating her stools. She has also lost 9 kg of weight and complains that she doesn’t ‘feel empty’ after defecating. Abdominal examination is normal, apart from an enlarged left supraclavicular lymph node. What is the most likely diagnosis? 
A  Cancer of the rectum 
B  Cancer of the sigmoid colon 
C  Gastric carcinoma 
D  Cancer of the caecum 
E  Pancreatic cancer
A

A

How well did you know this?
1
Not at all
2
3
4
5
Perfectly
54
Q

A 75-year-old man is rushed into A&E by ambulance. He finds it difficult to answer simple questions and is struggling to speak. On examination, power is 2/5 in
his right arm, 4/5 in his right leg and 5/5 in his left arm and leg. He has marked facial muscle weakness on the right half of his face and he is blind in the right half of his visual field. A CT head scan is performed and an ischaemic stroke is diagnosed. Which artery is most likely to be involved?
A Right anterior cerebral artery
B Left anterior cerebral artery
C Right posterior cerebral artery
D Right middle cerebral artery
E Left middle cerebral artery

A

E

How well did you know this?
1
Not at all
2
3
4
5
Perfectly
55
Q
A 62-year-old diabetic on metformin sees his GP for a routine blood test. He claims that he has been compliant with his treatment and has not experienced any symptoms recently. His blood test reveals: 
Na+ : 116 mmol/L (135-145)
K+ : 3.7 mmol/L (3.5-5)
Ca2+ : 2.4 mmol/L (2.2-2.6)
Total Cholesterol : 9.2 mmol/L (< 5) Serum Albumin : 48 g/L (35 -50) TFT - Normal 
SST - Normal
What is the most likely cause of his hyponatraemia? 
A  Addison's disease 
B  Hypothyroidism 
C  Erroneous result 
D  Drug side-effect 
E  Nephrotic syndrome
A

C

How well did you know this?
1
Not at all
2
3
4
5
Perfectly
56
Q
Which of the following lung pathologies produces the ‘sail sign’ appearance on CXR? 
A  Right upper lobe collapse 
B  Right middle lobe collapse 
C Right lower lobe collapse 
D  Left upper lobe collapse 
E  Left lower lobe collapse
A

E

How well did you know this?
1
Not at all
2
3
4
5
Perfectly
57
Q
A 47-year-old man has vomited 3 times and has not passed any faeces or flatus for the last 4 days. He had an open cholecystectomy 6 years ago but has otherwise been relatively fit and healthy. What is the best immediate management option for this patient? 
A  NG tube and IV fluids 
B  Surgery to resolve the obstruction 
C  Gastrograffin 
D  IV antibiotics 
E  Reassure and discharge
A

A

How well did you know this?
1
Not at all
2
3
4
5
Perfectly
58
Q
Which of the following is not a major criterion in the Framingham criteria for congestive cardiac failure? 
A  Bilateral ankle oedema 
B  Paroxysmal nocturnal dyspnea 
C  Cardiomegaly 
D  S3 gallop 
E  Acute pulmonary oedema
A

A

How well did you know this?
1
Not at all
2
3
4
5
Perfectly
59
Q
A 46-year-old female has experienced a painful sensation on the outer side of her left thigh for the past 3 months. She mentions that the sensation is very ‘bizarre’ and sometimes feels like it is burning or tingling. She has no other symptoms and has no past medical history of note. What is the most likely diagnosis? 
A  Meralgia paraesthetica 
B  Multiple sclerosis 
C  Sciatica 
D  Peripheral neuropathy 
E  Disc herniation
A

A

How well did you know this?
1
Not at all
2
3
4
5
Perfectly
60
Q
A 47-year-old female suffering from RUQ pain, lethargy and pruritus, is found to have an ALP of 300 IU/L (30-150 IU/L) and serology is positive for anti-mitochondrial antibodies. She also complains of dry, itchy eyes. Examination findings include icterus and xanthelasma. What is the most likely diagnosis? 
A  Type 1 autoimmune hepatitis 
B  Type 2 autoimmune hepatitis 
C Primary sclerosing cholangitis 
D Primary biliary cirrhosis
E Cirrhosis
A

D

How well did you know this?
1
Not at all
2
3
4
5
Perfectly
61
Q
A 53-year-old woman has been suffering from recurrent painful episodes affecting her face – mainly her right cheek. She describes the pain as being extremely intense, sharp and sudden, like an ‘electric shock’. It usually lasts for a few seconds before subsiding. The pain often occurs when she brushes her teeth. What is the most likely diagnosis? 
A  Giant cell arteritis 
B  Trigeminal neuralgia 
C  Ramsay Hunt syndrome 
D  Shingles 
E  Cluster headache
A

B

How well did you know this?
1
Not at all
2
3
4
5
Perfectly
62
Q
A 21-year-old woman visits the GP complaining of a 2-month history of bloating and watery diarrhoea. She adds that she often has to rush to the toilet. During the consultation, she starts furiously itching her elbows. On examination, there is a blistering, papulovesicular rash covering both elbows. What is the most likely diagnosis? 
A  Inflammatory bowel disease 
B  Coeliac disease 
C  Gastroenteritis 
D  Irritable bowel syndrome 
E  Ischaemic colitis
A

B

How well did you know this?
1
Not at all
2
3
4
5
Perfectly
63
Q
Which of the following is not a chest X-ray feature of heart failure? 
A  Kerley B lines 
B  Upper lobe diversion 
C  Cardiomegaly 
D  Pleural effusion 
E  Air bronchograms
A

E

How well did you know this?
1
Not at all
2
3
4
5
Perfectly
64
Q

A 71-year-old man is referred to the oncology clinic having suffered from hip pain, constipation and abdominal pain for the past few months. He has also noticed that he is having to urinate more frequently than usual, and, consequently, he is always extremely thirsty. A full blood count and U&Es are requested:
Hb = 10.5 g/dL (13-18)
MCV = 106 fl (76-96)
Platelets = 120 x109/L (150-400 x 109)
Creatinine = 125 umol/L (baseline: 72 umol/L (3 months ago))
The oncologist requests a blood film. Considering the most likely diagnosis, what would you expect to see on this patient’s blood film?
A Rouleaux formation
B Schistocytes
C Granulocytes with absent granulation and hyposegmented nuclei
D Dacrocytes
E Smear cells

A

A

How well did you know this?
1
Not at all
2
3
4
5
Perfectly
65
Q
A 56-year-old woman, of Somalian origin, presents with a 2-month history of haemoptysis. She has also noticed some weight loss during this time and complains that she is having to change her bed sheets more often than usual as they are often drenched with sweat in the morning. Examination reveals painless cervical lymphadenopathy and tender, purple lumps on her shins. A CXR reveals an area of consolidation in the right upper lobe. Which investigation should be performed next in order to establish a diagnosis? 
A  Bronchoalveolar lavage 
B  Chest CT scan 
C  Sputum sample for acid-fast bacilli 
D  Bronchoscopy and biopsy 
E  Mantoux test
A

C

How well did you know this?
1
Not at all
2
3
4
5
Perfectly
66
Q
. Which type of urinary tract stone is most common? 
A  Magnesium ammonium phosphate 
B  Calcium oxalate 
C Cysteine 
D Urate
E Hydroxyapatite
A

B

How well did you know this?
1
Not at all
2
3
4
5
Perfectly
67
Q
A 62-year-old man presents with severe, acute epigastric pain with nausea and vomiting. The pain radiates to the back and improves when sitting forward. It started 4 days ago, but the patient assumed it was indigestion and refused to come to hospital. On examination, there is epigastric tenderness and ecchymoses over the periumbilical area and flank. The patient drinks in moderation and has not had any alcohol recently. Serum amylase is 600 U/L (< 140). Which investigation should be performed to confirm the diagnosis? 
A  ERCP 
B  Abdominal CT scan 
C  Abdominal X-ray 
D  Abdominal ultrasound 
E  MRCP
A

D

How well did you know this?
1
Not at all
2
3
4
5
Perfectly
68
Q
A 39-year-old carpenter is brought into A&amp;E having fallen from a 3rd floor balcony. He landed on his head and has been unconscious since the incident. An intracranial haemorrhage is suspected and an urgent CT scan is requested. The patient’s vital signs are recorded: BP = 195/120 mm Hg; HR = 47 bpm (60-100). His breathing also appears to be irregular – shallow breaths interspersed with periods of apnoea. What is the name given to this phenomenon? 
A  Kussmaul sign 
B  Cushing’s reflex 
C  Beck’s triad 
D  Charcot’s triad 
E  Baroreceptor reflex
A

B

How well did you know this?
1
Not at all
2
3
4
5
Perfectly
69
Q

A 46-year-old woman visits A&E complaining of a fever and episodes of shivering. She returned from Nigeria 2 weeks ago and confesses that she was not very compliant with her antimalarial medication. Therefore, malaria is suspected. Which investigation should be performed to diagnose malaria?
A Thick and thin blood films
B Blood cultures
C Heterophile antibody test
D Enzyme-linked Immunosorbent Assay (ELISA)
E Urinalysis

A

A

How well did you know this?
1
Not at all
2
3
4
5
Perfectly
70
Q
A patient suffering an acute exacerbation of COPD has become hypoxic with an SaO2 of 83%. He requires administration of oxygen at a tightly regulated concentration. Which of the following methods of administering oxygen would be most appropriate? 
A  Nasal cannula 
B  Hudson face mask 
C  Venturi mask 
D  Non-rebreathing mask 
E  BiPAP
A

C

How well did you know this?
1
Not at all
2
3
4
5
Perfectly
71
Q

A 48-year-old man has been suffering from frequent urination for the past 5 months. He has been going to the toilet around 10 times per day and he has been drinking excessive volumes of water. He has also been constipated for the past month with vague ‘tummy pains’ and complains of joint pain in his hands. A blood test is requested, which shows:
Na+: 137 mmol/L (135-145) K+: 4.6 mmol/L (3.5-5) Ca2+: 3.0 mmol/L (2.2-2.6) ALP: 197 iU/L (30-150) PTH: 102 ng/L (10-65)
What is the most likely diagnosis?

A Vitamin D Toxicosis
B Parathyroid Adenoma
C Paget’s Disease 
D Malignancy 
EMilk-Alkali Syndrome
A

B

How well did you know this?
1
Not at all
2
3
4
5
Perfectly
72
Q
A 61-year-old male comes to A&amp;E complaining of chest pain and mentions that he can feel his heart ‘pumping out of his chest’. An ECG shows regular broad complex tachycardia, with no P waves. His blood pressure is 124/87 mm Hg. How should this patient be treated? 
A  Defibrillation 
B  DC cardioversion 
C  Amiodarone 
D  Adenosine 
E Atropine
A

C

How well did you know this?
1
Not at all
2
3
4
5
Perfectly
73
Q
A 30-year-old man presents to his GP complaining of a swollen scrotum, which he first noticed 3 weeks ago. He adds that the swelling feels like a ‘bag of worms’, and, despite being a little uncomfortable, it is not painful. On examination, the patient’s scrotum looks normal when he is supine, however, the left hemiscrotum becomes swollen when he stands up. The GP can get above the swelling and distinguish it from the testicle. What is the most likely diagnosis? 
A  Indirect inguinal hernia 
B  Direct inguinal hernia 
C  Hydrocoele 
D  Varicocoele 
E  Epididymal cyst
A

D

How well did you know this?
1
Not at all
2
3
4
5
Perfectly
74
Q
A 74-year-old woman is brought to A&amp;E having suffered several violent bouts of vomiting. On examination, she is clearly distressed and has a massively distended abdomen. When questioned, she struggles to answer but complains of generalised abdominal pain and mentions that she hasn't passed any faeces or flatus since the pain began. Bowel obstruction is suspected and an AXR is requested. The AXR shows a massively distended loop of large bowel which looks like an embryo. What is the most likely cause of this bowel obstruction? 
A  Colorectal cancer 
B  Sigmoid volvulus 
C  Caecal volvulus 
D  Adhesions 
E  Femoral hernia
A

C

How well did you know this?
1
Not at all
2
3
4
5
Perfectly
75
Q
A 36-year-old female presents to the GP complaining that the nail bed of her ring finger has detached and she is worried that the same is happening to her other finger nails. The GP suspects onycholysis. Which of the following is not associated with onycholysis? 
A  SLE 
B  Psoriasis 
C  Thyrotoxicosis 
D  Trauma 
E  Fungal infection
A

A

How well did you know this?
1
Not at all
2
3
4
5
Perfectly
76
Q
A 72-year-old man has recently suffered a stroke. He has recovered well and appears to have regained much of his physical strength, however, his speech has changed quite considerably. His daughter says that he will talk the same amount as he always did but his sentences will not make any sense, and he doesn’t seem to notice. When asked to describe what he did this morning, he responds: ‘the bugle fidget and that I played tractor to you before’. Damage to which part of the brain is likely to manifest in this way? 
A  Wernicke’s area 
B  Broca’s area 
C Arcuate fasciculus 
D Hippocampus
E Amygdala
A

A

How well did you know this?
1
Not at all
2
3
4
5
Perfectly
77
Q

An inpatient on the surgical ward is recovering after having a kidney stone removed. A routine blood test is performed which shows the following results:
Na+ : 135 mmol/L (135 – 145)
K+ : 8.7 mmol/L (3.5 – 6.0)
Ca2+ : 0.2 mmol/L (2.2 – 2.6)
An ECG is performed which shows no obvious abnormalities.
Given above information, what should be the next step in the management of this patient?
A Urgently draw another blood sample
B 10 mL 10% calcium gluconate
C 20 mL 20% calcium gluconate
D 50 mL 50% dextrose + 10 U insulin
E IV salbutamol

A

A

How well did you know this?
1
Not at all
2
3
4
5
Perfectly
78
Q

What are the ‘B symptoms’ of lymphoma?
A Fever, Lymphadenopathy, Rigors
B Fever, Night Sweats, Weight Loss
C Recurrent Infections, Anaemia, Easy Bruising
D Night Sweats, Pruritus, Weight Loss
E Lymphadenopathy, Weight Loss, Loss of Appetite

A

B

How well did you know this?
1
Not at all
2
3
4
5
Perfectly
79
Q
A 40-year-old teacher has recently heard several distressing comments about how flushed she is looking. On examination, she has very red cheeks and a loud S1 with a mid-diastolic murmur is heard over the apex when the patient is in the left lateral position. What is the most likely diagnosis? 
A  Mitral stenosis 
B  Mitral regurgitation 
C  Aortic stenosis 
D  Aortic regurgitation 
E  Tricuspid regurgitation
A

A

How well did you know this?
1
Not at all
2
3
4
5
Perfectly
80
Q

A 59-year-old female presents with epigastric pain that gets worse a few hours after eating. The patient has taken ibuprofen every day for the past 2 years for her chronic back pain. A urea breath test is negative. What is the most appropriate treatment option for this patient?
A Stop ibuprofen and give omeprazole
B Stop ibuprofen and give ranitidine
C Give amoxicillin, metronidazole and pantoprazole
D Give lifestyle advice and arrange to see the patient in 3 months
E Oral steroids

A

A

How well did you know this?
1
Not at all
2
3
4
5
Perfectly
81
Q
A 58-year-old woman presents to her GP with a 5-month history of worsening shortness of breath on exertion. She has also lost about 3 kg in weight and has experienced a dry cough. She has never smoked before and has a past medical history of rheumatoid arthritis, which was diagnosed 15 years ago. On examination, her fingers are clubbed and fine inspiratory crackles are heard bilaterally across the lower lung zones. What is the most likely diagnosis? 
A  COPD 
B  Lung cancer 
C  Bronchiectasis 
D  Pulmonary fibrosis 
E  Tuberculosis
A

D

How well did you know this?
1
Not at all
2
3
4
5
Perfectly
82
Q

A 38-year-old woman has been experiencing palpitations, sweating and diarrhoea for the past week. Before these symptoms began, she was on sick leave for 3 days with a fever, sore throat and cough. During the consultation, she appears to be very anxious with a slight tremor in her hands. Vital signs: HR = 114, Temp =
38.6°C. A thyroid examination reveals a warm, tender and slightly enlarged thyroid gland. A thyroid uptake scan is requested. What would you expect to see?

A Diffuse uptake throughout an enlarged gland
B No uptake
C Multinodular gland with multiple hot nodules
D Multinodular gland with a single hot nodule
E Diffuse uptake with a single cold nodule

A

B

How well did you know this?
1
Not at all
2
3
4
5
Perfectly
83
Q
A 24-year-old waitress presents to her GP after noticing a small, firm lump in her left breast. She first noticed the lump 1 week ago and is unsure about whether it has grown. She reports no nipple discharge or skin changes over the breast. Examination reveals a 1 x 2 cm lump in the upper outer quadrant of the left breast with no axillary lymphadenopathy. She is worried because her grandmother died of breast cancer. What is the next most appropriate step in the management of this patient? 
A  Urgent mammogram 
B  Urgent ultrasound scan 
C  Urgent CT Scan 
D  Arrange elective mastectomy 
E  Arrange elective wide local excision
A

B

How well did you know this?
1
Not at all
2
3
4
5
Perfectly
84
Q
A 73-year-old man was watching TV with his family, 2 hours ago, when his speech suddenly became slurred and he lost the ability to grip the remote control. His son, who accompanied him to A&amp;E, noticed that the left half of his father’s face drooped during this episode. His symptoms eventually subsided after around 15 minutes and he claims that he feels back to normal, albeit slightly shaken by his ordeal. What is the first step in his management? 
A Administer aspirin 
B Thrombolysis 
C CT Head Scan
D ECG
E Carotid Endarterectomy
A

A

How well did you know this?
1
Not at all
2
3
4
5
Perfectly
85
Q

A 9-year-old girl is brought to see her GP, by her father, because she has been complaining of pain in her knees and ankles and a tummy ache, which began yesterday. Urinalysis is positive for blood and protein. On examination, small purple spots are seen on her buttocks and her knees feel warm and swollen. Her father
adds that, 2 weeks ago, she took time off school because of the flu. What is the most likely diagnosis?
A Post-infectious glomerulonephritis
B Immune thrombocytopenic purpura
C Disseminated intravascular coagulation
D Henoch- Schӧnlein purpura
E Minimal change glomerulonephritis

A

D

How well did you know this?
1
Not at all
2
3
4
5
Perfectly
86
Q
A 64-year-old woman presents with severe left iliac fossa pain with nausea and vomiting. She adds that she has had a few episodes of rectal bleeding recently where the blood has coated the stools. She admits to eating a lot of fast food and having a low-fibre diet. She has not noticed any weight loss. Her left iliac fossa is tender on palpation and blood is found on DRE. What is the most likely diagnosis? 
A  Gastroenteritis 
B  Diverticulitis 
C  Angiodysplasia 
D  Colorectal carcinoma 
E  Inflammatory bowel disease
A

B

How well did you know this?
1
Not at all
2
3
4
5
Perfectly
87
Q
Which of the following organisms is responsible for causing whooping cough? 
A  Bordatella pertussis 
B  Treponema pallidum 
C  Cryptosporidium 
D  Mycoplasma pneumoniae 
E  Yersinia pestis
A

A

How well did you know this?
1
Not at all
2
3
4
5
Perfectly
88
Q
A 59-year-old man is brought to A&amp;E by his daughter after having collapsed at home. He has very little recollection of the incident – one minute he was doing the dishes, and next minute he was lying on his back on the floor. He has no history of recent head trauma and mentions that he felt ‘absolutely fine’ when he regained consciousness. An ECG is performed showing bradycardia (36 bpm) and dissociation between the p waves and QRS complexes. A diagnosis of complete heart block is made. What might be seen on close inspection of his JVP? 
A Large V waves 
B Cannon A waves 
C Kussmaul sign 
D Raised JVP with absent pulsation 
E Slow Y descent
A

B

How well did you know this?
1
Not at all
2
3
4
5
Perfectly
89
Q
A 36-year-old supermarket manager has been suffering from chronic back pain and stiffness. He first saw his GP about this matter 6 months ago and was given paracetamol, however, the pain worsened and has started affecting his job. He finds restocking the shelves particularly difficult as it requires repetitively bending down and picking up products. He mentions that the pain and stiffness is worst in the morning and gradually gets better with activity. What is the most likely diagnosis? 
A  Ankylosing spondylitis 
B  Lumbar disc herniation 
C  Osteoarthritis 
D  Muscle strain 
E  Vertebral fracture
A

A

How well did you know this?
1
Not at all
2
3
4
5
Perfectly
90
Q

A 69-year-old man is recovering in the inpatient respiratory ward having been diagnosed with pneumonia yesterday. A right-sided pleural effusion is identified on the chest X-ray. Which of the following findings on clinical examination of the right lung base, would be most consistent with a right-sided pleural effusion?
A Resonant percussion note, increased vocal resonance, vesicular breathing
B Resonant percussion note, reduced vocal resonance, reduced breath
sounds
C Dull percussion note, increased vocal resonance, bronchial breathing
D Dull percussion note, decreased vocal resonance, reduced breath sounds
E Dull percussion note, decreased vocal resonance, vesicular breathing

A

D

How well did you know this?
1
Not at all
2
3
4
5
Perfectly
91
Q
A 34-year-old woman is complaining of a drooping eyelid that has been affecting her vision. She has also been suffering from fatigue over the past 3 months, which has impacted on her job as a yoga instructor. She feels relatively fine in the morning, however, she feels very weak towards the end of the day and struggles to complete her evening sessions. What is the most likely diagnosis? 
A  Polymyalgia rheumatic 
B  Anaemia 
C  Myasthenia Gravis 
D  Lambert-Eaton syndrome 
E  Horner’s syndrome
A

C

How well did you know this?
1
Not at all
2
3
4
5
Perfectly
92
Q

A 19-year-old asthmatic visits his GP because he is having to use his salbutamol inhaler more than 3 times per week. What is the next step in the management of this patient’s asthma?
A Add oral prednisolone
B Reassure and send home
C Increase the dose of inhaled salbutamol
D Add inhaled salmeterol
E Add inhaled low-dose beclomethasone

A

E

How well did you know this?
1
Not at all
2
3
4
5
Perfectly
93
Q

A 71-year-old owner of a dye factory presents to his GP having experienced 3 episodes of blood in his urine over the past week. When asked to elaborate, he says that his urine is bright red, however, he experiences no pain when passing urine and has not experienced any trauma to his genitals recently. Otherwise, he has generally been quite healthy although he has noticed that his clothes have become quite loose-fitting despite not having changed his diet or exercised. What is the most likely diagnosis?

A Pyelonephritis 
B Glomerulonephritis 
C Bladder Cancer 
D Prostate Cancer 
E Ureteric Stone
A

C

How well did you know this?
1
Not at all
2
3
4
5
Perfectly
94
Q
A 44-year-old woman presents with a 7-month history of heartburn, an acidic taste in the back of her mouth and painful swallowing. The GP suspects gastro- oesophageal reflux that is aggravated by a medication that she is taking for a heart CONDITION. Which of the following options could cause or worsen gastro-oesophageal 
A  Beta-blockers 
B  ACE inhibitors 
C  Nitrates 
D  Diuretics 
E  Angiotensin receptor blockers
A

C

How well did you know this?
1
Not at all
2
3
4
5
Perfectly
95
Q
A 27-year-old man presents with palpitations and light-headedness. An ECG shows features consistent with a supraventricular tachycardia. Adenosine is administered and the SVT is terminated. A repeat ECG shows a short PR interval and a QRS complex with a slurred upstroke. What is the diagnosis? 
A  Brugada syndrome 
B  LBBB 
C  Romano-Ward syndrome 
D  Wolff-Parkinson-White syndrome 
E  Complete heart block
A

D

How well did you know this?
1
Not at all
2
3
4
5
Perfectly
96
Q
An 18-year-old man visits his GP complaining of an itchy scalp and nose. He admits that he has been feeling quite self-conscious since a friend pointed out that he has dandruff. On examination, there are patchy erythematous plaques along his scalp covered with yellow scales and white flakes of dead skin in his hair. Similar lesions are also found in the nasolabial folds. Which type of eczema is this likely to be? 
A Nummular 
B Seborrhoeic
 C Contact
D Atopic
E Pompholyx
A

B

How well did you know this?
1
Not at all
2
3
4
5
Perfectly
97
Q

Which of the following options fits the criteria for giving long-term oxygen therapy in COPD?
A PaO2 7.3-10 kPa despite maximal treatment
B PaO2 7.3-10 kPa and pulmonary hypertension
C PaO2 < 7.3 kPa despite maximal treatment
D PaCO2 > 6 kPa despite maximal treatment
E PaCO2 > 8 kPa despite maximal treatmen

A

C

How well did you know this?
1
Not at all
2
3
4
5
Perfectly
98
Q
A 71-year-old female, with a history of atrial fibrillation, presents to A&amp;E with severe, diffuse abdominal pain. Her blood pressure is 84/60 mm Hg and her pulse is irregularly irregular with a rate of 130 bpm. Abdominal examination is normal. An abdominal X-ray is performed. Which of these radiological features is most likely to be seen? 
A  Rigler’s sign 
B  Pneumoperitoneum 
C  Toxic megacolon 
D  Gasless abdomen 
E  Coffee bean sign
A

D

How well did you know this?
1
Not at all
2
3
4
5
Perfectly
99
Q
Which of the following results would you expect to see in a patient with toxic multinodular goitre? 
A  High TSH, High TRH &amp; High T3/T4 
B  Low TSH, Low TRH &amp; High T3/T4 
C  Low TSH, High TRH &amp; High T3/T4 
D  High TSH, Low TRH &amp; High T3/T4 
E  High TSH, High TRH &amp; Low T3/T4
A

B

How well did you know this?
1
Not at all
2
3
4
5
Perfectly
100
Q

A 68-year-old man has suddenly developed an extremely painful left leg. On examination, his left leg is pale, cold and his dorsalis pedis and posterior tibial pulses are impalpable. His radial pulse is 120 bpm and has an irregularly irregular rhythm. What is the first step in the management of this patient?
A Duplex ultrasound scan of the lower limb vessels
B Oral aspirin
C IV heparin
D Measure Ankle-Brachial Pressure Index (ABPI)
E DC Cardioversion

A

C

How well did you know this?
1
Not at all
2
3
4
5
Perfectly
101
Q

A 22-year-old man presents with a headache, neck stiffness and photophobia. A diagnosis of viral meningitis is suspected. Once raised ICP is excluded, a lumbar puncture is performed. Which set of results would be consistent with viral meningitis?
A Cloudy CSF, high neutrophils, high protein and low glucose
B High lymphocytes, high protein and normal glucose
C High lymphocytes, low protein and normal glucose
D High neutrophils, high protein and high glucose.
E Fibrinous CSF, high lymphocytes, high protein and low glucose

A

B

How well did you know this?
1
Not at all
2
3
4
5
Perfectly
102
Q
Which of the following is not a clinical feature of anaemia? 
A  Conjunctival pallor 
B  Glossitis 
C  Angular stomatitis 
D  Ruddy/Red complexion 
E  Shortness of breath
A

D

How well did you know this?
1
Not at all
2
3
4
5
Perfectly
103
Q

A 39-year-old female presents at her GP practice having coughed up blood last night. This has happened on two previous occasions. She has no significant past medical history although she does experience regular nosebleeds. Blood tests reveal a high ESR and urinalysis reveals proteinuria and haematuria. The presence of which antibody would support the most likely diagnosis?
A Anti-GBM antibody
B pANCA
C cANCA
D Anti-liver/kidney microsomal antibody
E Anti-smooth muscle antibody

A

C

How well did you know this?
1
Not at all
2
3
4
5
Perfectly
104
Q

Which of the following is not part of the criteria for diagnosing sepsis?
A Heart Rate > 90 bpm
B Respiratory Rate > 20 breaths per minute
C Temperature > 38°C
D White Cell Count < 4 x 109/L
E Blood Pressure < 90/60 mm Hg

A

E

How well did you know this?
1
Not at all
2
3
4
5
Perfectly
105
Q
A 72-year-old patient with severe COPD has recently experienced worsening dyspnoea despite maximal treatment. On examination, he is cyanotic with a raised JVP and ankle oedema. Palpation reveals hepatomegaly. What is the most likely diagnosis? 
A  Left ventricular failure 
B  Congestive cardiac failure 
C  Cor pulmonale 
D  Pulmonary hypertension 
E  Restrictive cardiomyopathy
A

C

How well did you know this?
1
Not at all
2
3
4
5
Perfectly
106
Q
A 32-year-old man with psoriasis presents to his GP with deformed hands that have been affecting his ability to do his daily tasks. It has gradually got worse over several years. On closer inspection, his fingers are badly deformed and appear to be telescoped. What is the most likely diagnosis? 
A  Rheumatoid arthritis 
B  Arthritis mutilans 
C  Psoriatic spondylopathy 
D  Osteoarthritis 
E  Distal interphalangeal joint disease
A

B

How well did you know this?
1
Not at all
2
3
4
5
Perfectly
107
Q

A 21-year-old woman has fainted 4 times in the past 3 months. She becomes sweaty and nauseous before she faints and is usually unconscious for a few seconds. Her friends have told her that she looks abnormally pale before she collapses. She doesn’t know if she jerks whilst unconscious, but has not lost control of her bladder or bitten her tongue. When she regains consciousness, she feels slightly dizzy but does not feel confused. What is the most likely cause of her fainting?
A Hypoglycaemia
B Epileptic seizure
C Vasovagal syncope
D Arrhythmia
E Hypertrophic obstructive cardiomyopathy

A

C

How well did you know this?
1
Not at all
2
3
4
5
Perfectly
108
Q
A 50-year-old taxi driver attends a GP appointment because he has recently been ‘bumping into things’ quite regularly and has had to take a break from work over fears about his vision. He struggles to see anything in the left half of his visual field. Examination reveals a left homonymous hemianopia. In which part of the visual pathway is the lesion likely to be located? 
A  Optic chiasm 
B  Left optic tract 
C  Left optic radiation 
D  Right optic nerve 
E  Right optic tract
A

E

How well did you know this?
1
Not at all
2
3
4
5
Perfectly
109
Q
. A 75-year-old woman has had a 3-week history of lower abdominal discomfort and bloating. She is embarrassed to admit that she has recently started wearing adult diapers because she has been soiling her underwear. Her stools are usually very watery and drip into the pan. She has not lost any weight or noticed any blood in the stool. She claims to have a balanced, healthy diet. She has taken codeine every day for the past 4 months since she suffered a hip fracture. On examination, her abdomen is mildly distended and a solid mass is palpated in the left iliac fossa. On digital rectal examination, her underwear is soiled and liquid stool is seen on withdrawal of the finger. What is the most likely diagnosis? 
A  Rectal carcinoma 
B  Faecal impaction 
C  Inguinal hernia 
D  Ischaemic colitis 
E  Rectocoele
A

B

How well did you know this?
1
Not at all
2
3
4
5
Perfectly
110
Q
  1. A 24-year-old female presents to her GP complaining that her periods have become extremely irregular. She normally has 26-29 day cycles, but in the past 6 months her periods have been much less frequent. On closer inspection, she appears to have an abnormally large amount of facial hair for a young female and she is also suffering from acne, which, she claims, she never had as a teenager. She has gained weight over the past few months, which, alongside the acne and facial
    hair growth, has made her feel depressed. What is the most likely diagnosis?
    A Hypothyroidism
    B Turner’s syndrome
    C Polycystic ovarian syndrome
    D Pregnancy
    E Panhypopituitarism
A

C

How well did you know this?
1
Not at all
2
3
4
5
Perfectly
111
Q
Which of the following is a respiratory cause of asterixis? 
A  Hypoxia 
B  Carbon dioxide retention 
C  Salbutamol side-effect 
D  Secondary polycythaemia 
E  Bronchiectasis
A

B

How well did you know this?
1
Not at all
2
3
4
5
Perfectly
112
Q
A 23-year-old student has arranged an appointment with his GP to discuss his ‘incredibly itchy’ eyes. Yesterday morning he noticed that his left eye became quite red and itchy, and started watering. A few hours later, his right eye also started to display the same symptoms. On closer inspection, both eyes show conjunctival injection and watering. A yellow crust is seen across the margins of the eyelids. 
What is the most likely diagnosis? 
A  Hypopyon 
B  Hyphaema 
C  Bacterial Conjunctivitis 
D  Viral Conjunctivitis 
E  Uveitis
A

C

How well did you know this?
1
Not at all
2
3
4
5
Perfectly
113
Q
A 53-year-old man visits his GP to discuss a swollen scrotum that has caused him some discomfort and much embarrassment since he first noticed it 3 weeks ago. The swelling has grown gradually and, although it is uncomfortable, it is not painful. He reports no difficulties with passing urine. On examination, his left hemiscrotum is considerably enlarged, fluctuant and non-tender. It is possible to get above the swelling, however, the left testicle cannot be distinguished. When a pen torch is shone on the swelling, it illuminates brightly. What is the most likely diagnosis? 
A  Varicocoele 
B  Hydrocoele 
C  Testicular tumour 
D  Epididymal cyst 
E  Indirect inguinal hernia
A

B

How well did you know this?
1
Not at all
2
3
4
5
Perfectly
114
Q
  1. A 52-year-old man has been experiencing some chest pain and shortness of
    breath, which is worse when lying down. He has also collapsed 3 times in the past couple of months. His father died of a heart condition when he was 58 years old, although he cannot recall the details of the condition. On examination, a jerky carotid
    pulse is palpated and a crescendo-decrescendo murmur is heard over the carotid artery. What is the most likely diagnosis?
    A Aortic stenosis
    B Hypertrophic obstructive cardiomyopathy
    C Left heart failure
    D Mitral regurgitation
    E Constrictive pericarditis
A

B

How well did you know this?
1
Not at all
2
3
4
5
Perfectly
115
Q
A 48-year-old man has been in hospital for 2 days receiving treatment for pneumonia. He has a past medical history of acute pancreatitis, which occurred 3 years ago. He has a long history of alcohol abuse. In the last hour, he has started sweating excessively, complains of palpitations and appears very anxious. He is clearly agitated and begins shouting at the nurses to ‘get these creatures off me!’ What is the most appropriate treatment? 
A Diazepam 
B Phenobarbital
C Loperamide
D Chlordiazepoxide 
E Risperidone
A

D

How well did you know this?
1
Not at all
2
3
4
5
Perfectly
116
Q

A 56-year-old man has recently registered at a new GP practice. As part of the registration process he has been asked to undergo some routine blood tests. FBC reveals the following results:
Hb : 106 g/L (130-180)
WBC : 95 x 109 /L (4-11) Platelets : 86 x 109 /L (150-400) Lymphocytes : 85 x 109 (1.5-4.5)
A diagnosis of chronic lymphocytic leukaemia is suspected. Which of the following features are you most likely to see on his blood film?
A Smear cells
B Atypical lymphocytes
C Auer rods
D Reed-Sternberg cells
E Schistocytes

A

A

How well did you know this?
1
Not at all
2
3
4
5
Perfectly
117
Q
A 21-year-old university student presents with a 1-week history of sore throat, fever and malaise. On examination, there is cervical lymphadenopathy, splenomegaly and inflamed tonsils. The GP diagnoses the patient with bacterial tonsillitis and prescribes ampicillin. The patient comes back 2-days later with a widespread maculopapular rash. What is the underlying diagnosis? 
A  Penicillin allergy 
B  Erythema multiforme 
C  Stevens-Johnson syndrome 
D  Infectious mononucleosis 
E  Idiopathic thrombocytopaenic purpura
A

D

How well did you know this?
1
Not at all
2
3
4
5
Perfectly
118
Q

A 47-year-old man presents to his GP having experienced a few episodes of haemoptysis over the past month. He returned from a holiday to Bangladesh 6 weeks ago. On direct questioning, he admits to losing approximately 5 kg in weight over the past month and he has had to replace his bed sheets more frequently because they are often drenched with sweat when he wakes up in the morning. A chest X-ray reveals an area of consolidation in the right upper zone. Sputum microscopy using Ziehl-Neelsen stain reveals acid-fast bacilli. What is the most appropriate treatment option?
A Rifampicin and isoniazid for 6 months; ethambutol and pyrazinamide for the first 2 months
B Ethambutol and pyrazinamide for 6 months; rifampicin and isoniazid for the first 2 months
C Rifampicin and isoniazid for 6 months; ethambutol and pyrazinamide for the first 4 months
D Rifampicin and pyrazinamide for 6 months; ethambutol and isoniazid for the first 2 months
E Rifampicin, pyrazinamide, ethambutol and isoniazid for 6 months

A

A

How well did you know this?
1
Not at all
2
3
4
5
Perfectly
119
Q
  1. A 23-year-old university student is brought to A&E at 2 am by his friends. He is
    clearly inebriated and struggles to maintain conversation. His friends explain that he had been celebrating the recent election results at a bar when he began to vomit. He vomited several times and splashes of ‘bright red’ blood was seen the last two times that he vomited. What is the most likely diagnosis?
    A Peptic ulcer disease
    B Boerhaave syndrome
    C Mallory-Weiss syndrome
    D Gastritis
    E Osler-Weber-Rendu Syndrome
A

C

How well did you know this?
1
Not at all
2
3
4
5
Perfectly
120
Q
A 65-year-old man is brought in to A&amp;E by his wife. She says that he has been very confused over the last few days and has fallen over several times. She adds that her husband has wet the bed twice over the last 2 days – this has never happened before. What is the most likely diagnosis? 
A  Alzheimer’s disease 
B  Obstructive hydrocephalus 
C  UTI 
D  Subdural haematoma 
E  Normal pressure hydrocephalus
A

E

How well did you know this?
1
Not at all
2
3
4
5
Perfectly
121
Q
Which of the following is a feature of limited cutaneous systemic sclerosis? 
A Calcinosis 
B Cyanosis
C Striae
D Onycholysis 
E Clubbing
A

A

How well did you know this?
1
Not at all
2
3
4
5
Perfectly
122
Q
A 32-year-old basketball player is brought to A&amp;E extremely breathless. He was at basketball training when he suddenly felt himself getting more and more breathless and developed a ‘stabbing’ pain on the right side of his chest. He has never experienced anything like this before. On examination, he is very tall and thin, and breath sounds are reduced over the right side of his chest. What is the most likely diagnosis? 
A  PE 
B  Primary pneumothorax 
C  Secondary pneumothorax 
D  Myocardial infarction 
E  Asthma attack
A

B

How well did you know this?
1
Not at all
2
3
4
5
Perfectly
123
Q
A 22-year-old student presents with a severe headache and fever that has lasted 1 day. On examination, he has a stiff neck and a rash across his arms and legs. The junior doctor gently flexes the patient’s neck. As he does this, the patient’s hips flex. What is the name of this sign? 
A Uhthoff’s sign
B Lhermitte’s sign 
C Kernig’s sign
D Brudzinski’s sign 
E Tinel’s sign
A

D

How well did you know this?
1
Not at all
2
3
4
5
Perfectly
124
Q
A 61-year-old woman visits the GP complaining of 13 kg of weight loss over the past 6 months. On direct questioning, she admits that her faeces are lighter in colour than normal and her urine has become darker. She is jaundiced and a large non- tender mass is palpated in her right upper quadrant. What is the most likely diagnosis? 
A  Gallstones 
B  Hepatocellular carcinoma 
C  Pancreatic cancer 
D  Bile duct stricture 
E  Cirrhosis
A

C

How well did you know this?
1
Not at all
2
3
4
5
Perfectly
125
Q
A 46-year-old man has been admitted to A&amp;E after experiencing palpitations, which began about 4 hours ago. An ECG is performed, which reveals atrial fibrillation. He has no previous history of ischaemic heart disease. He refuses DC cardioversion. What is the next most appropriate treatment option? 
A  Defibrillation 
B  Low molecular weight heparin 
C  Warfarin 
D  Flecainide 
E  Digoxin
A

D

How well did you know this?
1
Not at all
2
3
4
5
Perfectly
126
Q
A 4-year-old boy is referred to the paediatric department by his GP after a 3-week history of fatigue, shortness of breath and recurrent chest infections. A thorough examination is performed, which revealed extensive bruising across the child’s body, hepatosplenomegaly and cervical lymphadenopathy. Based on the information provided, what is the most likely diagnosis? 
A  Acute lymphoblastic leukaemia 
B  Acute myeloid leukaemia 
C  Chronic lymphocytic leukaemia 
D  Chronic lymphoblastic leukaemia 
E  Hodgkin’s lymphoma
A

A

How well did you know this?
1
Not at all
2
3
4
5
Perfectly
127
Q

A 79-year-old woman is accompanied by her granddaughter to A&E. She has had a productive cough and a fever for the past 4 days. On examination, she has an AMTS of 5/10, respiratory rate of 31/min and blood pressure of 92/66 mm Hg. Her urea is 3 mmol/L (2.5-6.7). A CXR reveals an area of consolidation in the right middle lobe. Community-acquired pneumonia is suspected. What is her CURB-65 score?
A There is not enough information to tell.
B2
C3
D4
E5

A

C

How well did you know this?
1
Not at all
2
3
4
5
Perfectly
128
Q

A 28-year-old man has experienced several episodes of sweating, palpitations and anxiety over the past 6 months. He has a past medical history of thyroid cancer (aged 19) which was treated with total thyroidectomy. What is the most appropriate investigation?
A Serum 17-hydroxyprogesterone levels
B 24 hr urine 5-hydroxyindoleacetic acid levels
C 24 hr urine vanillylmendelic acid
D Plasma adrenaline concentration
E Thyroid uptake scan

A

C

How well did you know this?
1
Not at all
2
3
4
5
Perfectly
129
Q
Which of the following triads best describes Horner’s syndrome? 
A  Ptosis, miosis, anhydrosis 
B  Proptosis, miosis, hyperhidrosis 
C  Ptosis, mydriasis, anhydrosis 
D  Ptosis, enophthalmos, hyperhidrosis 
E  Proptosis, miosis, anhydrosis
A

A

How well did you know this?
1
Not at all
2
3
4
5
Perfectly
130
Q
Which virus is implicated in around 50% of cases of Hodgkin’s lymphoma? 
A  Human cytomegalovirus 
B  Herpes simplex virus 2 
C  Varicella zoster 
D  Epstein-Barr virus 
E Human herpesvirus 7
A

D

How well did you know this?
1
Not at all
2
3
4
5
Perfectly
131
Q
A 54-year-old man is complaining of abdominal heaviness and shortness of breath. On examination, his abdomen is distended, non-tender and exhibits shifting dullness with a fluid thrill. The junior doctor suspects ascites and requests a diagnostic paracentesis. It reveals a Serum-Ascites Albumin Gradient (SAAG) of 9 g/L. Which of the following is a potential cause of his ascites? 
A  Cirrhosis 
B  Congestive cardiac failure 
C  Portal hypertension 
D  Budd-Chiari syndrome 
E  Nephrotic syndrome
A

E

How well did you know this?
1
Not at all
2
3
4
5
Perfectly
132
Q
A 56-year-old man has been waking up several times at night to empty his bladder. He says he doesn’t feel completely empty after finishing and his stream seems to be quite ‘stop and start’. He often has to strain to maintain the flow. Which of his symptoms is considered irritative? 
A  Incomplete emptying 
B  Having to start and stop 
C  Increased urination at night 
D  Straining 
E  Weak flow
A

C

How well did you know this?
1
Not at all
2
3
4
5
Perfectly
133
Q
An 8-year-old boy is brought to the GP by his mother. He has a very swollen and painful knee which arose yesterday without any preceding trauma. On closer inspection, he is afebrile and the joint, despite being swollen, does not appear inflamed. He also has several bruises across his torso. His mother mentions that her father suffered from haemophilia and that she is worried that her son may have the same disease. Blood tests are requested. Which of the following results would be suggestive of a diagnosis of haemophilia? 
A  High APTT, Normal PT 
B  Normal APTT, High PT 
C  High APTT, High PT 
D  Low bleeding time 
E  Low vWF
A

A

How well did you know this?
1
Not at all
2
3
4
5
Perfectly
134
Q
An 82-year-old man is brought into A&amp;E complaining of severe flank pain that started suddenly about 30 minutes ago. On examination, he looks very unwell and his palms are cold and sweaty. Vital Signs: HR = 132 bpm; BP = 84/52 mm Hg. What is your top differential? 
A  Myocardial infarction 
B  Ruptured abdominal aortic aneurysm 
C  Ureteric colic 
D  Disc prolapse 
E  Muscle sprain
A

B

How well did you know this?
1
Not at all
2
3
4
5
Perfectly
135
Q
A 31-year-old lady, who is 7 months pregnant, is brought to A&amp;E having become extremely short of breath this morning. She has also experienced sharp chest pain on her right side. Examination reveals no abnormalities and an ECG shows sinus tachycardia. A pulmonary embolism is suspected. What is the most appropriate investigation to request? 
A  D-Dimer 
B  CTPA 
C  VQ scan 
D  Chest X-ray 
E  Doppler ultrasound of the lower limbs
A

C

How well did you know this?
1
Not at all
2
3
4
5
Perfectly
136
Q
A 76-year-old woman is brought to A&amp;E by her daughter. She is complaining of severe left iliac fossa pain accompanied by nausea, vomiting and fever. On inspection, she shows signs of peritonism. Vital signs: HR = 123 bpm, RR = 24 bpm, Temp = 38.7°C and BP = 87/54 mm Hg. An erect CXR is requested, which shows air under the diaphragm. A diagnosis of perforated diverticulitis localised to the sigmoid colon is made. What is the most appropriate surgical procedure? 
A  Left colectomy 
B  Abdominoperineal resection 
C  Hartmann’s procedure 
D  Delorme procedure 
E  Anterior resection
A

C

How well did you know this?
1
Not at all
2
3
4
5
Perfectly
137
Q
Which of the following tumour markers is associated with ovarian cancer? 
A  CA 15-3 
B  CA 19-9 
C  CA125 
D  CEA 
E  aFP
A

C

How well did you know this?
1
Not at all
2
3
4
5
Perfectly
138
Q
A 61-year-old man visits his GP complaining of a ‘shooting pain’ in his legs. The pain comes on when he walks his dog, and it gets particularly bad when walking downhill. On questioning, he reveals that he has been urinating about 10 times every day, which is much more than usual. On examination, there is a loss of sensation up to the T10 vertebral level, increased tone in his legs and brisk ankle jerks. The GP also notices that the patient has a stooped posture. What is the most likely diagnosis? 
A  Benign prostate hypertrophy 
B  Motor neuron disease 
C  Sciatica 
D  Spinal cord stenosis 
E  Cauda equina syndrome
A

D

How well did you know this?
1
Not at all
2
3
4
5
Perfectly
139
Q
An 11-year-old girl, who has recently moved to the UK from Cambodia, is referred to the cardiology department after her GP identified a heart murmur a few weeks after diagnosing her with a throat infection. She has also experienced intermittent joint pain, mainly affecting her knees and hips. On examination, she has a mid-diastolic murmur heard loudest over the mitral area and a few small, mobile nodules are palpated along the ulnar border of her forearms. What is the most likely diagnosis? 
A  Infective endocarditis 
B  Rheumatic fever 
C  Septic arthritis 
D  Rheumatoid arthritis 
E  Lyme disease
A

B

How well did you know this?
1
Not at all
2
3
4
5
Perfectly
140
Q
A 12-year-old boy is brought into A&amp;E – he is extremely drowsy, appears dehydrated and has vomited whilst in the ambulance. He is also clutching his abdomen and appears to be in considerable pain. He is a known diabetic, and DKA is suspected. The patient begins breathing in a very deep and laboured manner. What is the name given to this pattern of breathing? 
A  Cheyne-Stokes breathing 
B  Hypoventilation 
C  Kussmaul breathing 
D  Biot’s respiration 
E  Apnoea
A

C

How well did you know this?
1
Not at all
2
3
4
5
Perfectly
141
Q
A 21-year-old man has been experiencing some scrotal discomfort over the past month, which he describes as feeling ‘heavier than usual’. On examination, a firm, non-tender lump can be palpated at the base of the right testicle. The patient had an undescended testicle as a child, which was corrected with orchidopexy. A diagnosis of testicular cancer is suspected. The registrar recommends performing a CT scan to assess for spread. Which group of lymph nodes does testicular cancer spread to? 
A Inguinal
B Femoral
C Para-aortic 
D Iliac
E Mesenteric
A

C

How well did you know this?
1
Not at all
2
3
4
5
Perfectly
142
Q
Which of the following matches the criteria for type 2 respiratory failure? 
A  PaO2 < 10.5 kPa, PaCO2 > 6 kPa 
B  PaO2 <8kPa,PaCO2 >6kPa 
C  PaO2 < 10.5 kPa, PaCO2 > 8 kPa 
D  SaO2 < 90%, PaCO2 < 8 kPa 
E  SaO2 <90%,PaO2 <8kPa
A

B

How well did you know this?
1
Not at all
2
3
4
5
Perfectly
143
Q
A 26-year-old model comes to see her GP after having noticed some blood streaked on the paper after emptying her bowels. This started two weeks ago. She adds that defecation is very painful. When asked about her diet, she reveals that she often eats ready meals and drinks relatively little water because her job involves regular travelling making it difficult for her to maintain a healthy diet. What is the most likely diagnosis? 
A Haemorrhoids
B  Anal fissure 
C Anal fistula 
D  Colorectal cancer 
E  Ulcerative colitis
A

B

How well did you know this?
1
Not at all
2
3
4
5
Perfectly
144
Q
A 71-year-old man presents with an 8-month history of worsening shortness of breath on exertion, orthopnoea and a cough productive of pink, frothy sputum. He has a past medical history of ischaemic heart disease and type 2 diabetes mellitus. Heart failure is suspected. What is the best investigation to confirm a diagnosis of heart failure? 
A  ECG 
B  Brain natriuretic peptide 
C  Atrial natriuretic peptide 
D  Echocardiogram 
E  Chest X-ray
A

D

How well did you know this?
1
Not at all
2
3
4
5
Perfectly
145
Q
An inpatient on the respiratory ward is currently undergoing treatment for a pneumonia that he developed 2 days ago. A blood test is performed which shows a low white cell count, with a particularly low neutrophil count. The patient is re- examined and found to have a considerably enlarged spleen. On further questioning, He has suffered from 3 infections in the past 5 months and complains that his rheumatoid arthritis has been getting worse. 
What is the most likely diagnosis? 
A  Malaria 
B  Tuberculosis 
C  Felty’s syndrome 
D  Lymphoma 
E  Chronic lymphocytic leukaemia
A

C

How well did you know this?
1
Not at all
2
3
4
5
Perfectly
146
Q
Which stain is used when testing for TB? 
A  Giemsa 
B  Gram 
C  India Ink 
D  Sudan Black 
E  Ziehl-Neelsen
A

E

How well did you know this?
1
Not at all
2
3
4
5
Perfectly
147
Q

A 60-year-old man, with a history of hypertension and type 1 diabetes mellitus, is brought to A&E by his daughter. She says that 3 hours ago, when they were eating dinner, he suddenly dropped his fork and started slurring his words. On examination, the right side of his face is drooping, muscle power is 1/5 in the right arm and 5/5 in the left; 3/5 in the right leg and 5/5 in the left. What is the most appropriate management option?
A CT head to exclude bleed, then treatment dose of warfarin
B CT head to exclude bleed, then give antiplatelets and perform a swallow
assessment
C CT head to exclude bleed, then IV thrombolysis
D Control blood pressure and IV mannitol
E Craniotomy and evacuation

A

A

How well did you know this?
1
Not at all
2
3
4
5
Perfectly
148
Q
A 42-year-old wildlife photographer returns from a 6-month trip to South Africa. He has noticed a small, dark mole on his right calf, which, he claims, has not always been there. Examination reveals an asymmetrical, dark lesion with irregular borders that measures 1 cm in diameter. Malignant melanoma is suspected and an excisional biopsy is taken and sent to the pathologist. Which feature of the histological analysis is the most useful prognostic indicator in this situation? 
A  Number of mitoses 
B  Surface area of lesion 
C  Depth of lesion 
D  Mass of lesion 
E  Lymphocyte infiltration
A

C

How well did you know this?
1
Not at all
2
3
4
5
Perfectly
149
Q
An 85-year-old man is brought to A&amp;E having been found on a roundabout in the middle of the night. He is very confused with an AMTS of 4/10. U&amp;Es are requested, which reveal hyponatraemia (Na+ : 118 mmol/L (135-145)). Care is taken to increase the sodium concentration slowly. What is a major consequence of raising plasma sodium concentration too rapidly? 
A  Stroke 
B  Rhabdomyolysis 
C  Central pontine myelinolysis 
D  AKI 
E  Cerebral oedema
A

C

How well did you know this?
1
Not at all
2
3
4
5
Perfectly
150
Q
Which of the following antibodies is most sensitive for primary sclerosing cholangitis? 
A AMA
B ASLA
C ALKM-1
D pANCA 
E ANA
A

D

How well did you know this?
1
Not at all
2
3
4
5
Perfectly
151
Q
Which of the following is not a sign of cirrhosis? 
A  Gynaecomastia 
B  Asterixis 
C  Koilonychia 
D  Hepatic fetor 
E  Clubbing
A

C

How well did you know this?
1
Not at all
2
3
4
5
Perfectly
152
Q
A 67-year-old pensioner, with a 40 pack-year smoking history, visits his GP complaining of shortness of breath that has gradually been getting worse over the past 6 months. He used to be able to walk 500 metres to the shops but now he struggles to make it up the stairs at home. He has also been suffering from a persistent cough productive of clear sputum. Which investigation is required to confirm the diagnosis? 
A  Spirometry 
B  Peak Expiratory Flow 
C  Sputum Culture 
D  Chest X-Ray 
E  Bronchoscopy and Biopsy
A

A

How well did you know this?
1
Not at all
2
3
4
5
Perfectly
153
Q
An 18-year-old man visits the GP having recently returned from a holiday to Thailand. He complains of a 2-day history of watery, bloody diarrhoea and has vomited 6 times. On examination, he is pyrexial with diffuse abdominal tenderness. Which organism is most likely to be causing his symptoms? 
A  Giardia lamblia 
B  Vibrio cholera 
C  Norovirus 
D  E. coli 0157 
E  Salmonella
A

D

How well did you know this?
1
Not at all
2
3
4
5
Perfectly
154
Q

A 42-year-old man is brought into A&E by his wife. He is clutching his head and appears to be drowsy and distressed. Though a clear history is difficult to ascertain, he mentions that he has an ‘absolutely devastating headache’ that suddenly came on this morning. He has never experienced pain like this before. Towards the end of
the consultation he begins to vomit. He is apyrexial and denies any trauma to the head. On examination, the patient’s neck is slightly stiff, he is hypertensive and has large bilateral palpable masses in his abdomen. On direct questioning, he reveals that his father died suddenly at the age of 49. Which underlying disease has predisposed the patient to this clinical scenario?
A Renal cell carcinoma
B Polycystic kidney disease
C Medullary sponge kidney
D Phaeochromocytoma
E Subdural haemorrhage

A

B

How well did you know this?
1
Not at all
2
3
4
5
Perfectly
155
Q

A 33-year-old man has recently been diagnosed with hypertension following the incidental finding of abnormally high blood pressure during a routine check-up at his GP practice. Without further investigation, he was started on Ramipril. 1 week later, he begins to feel very nauseous and vomits several times. He is taken to A&E where his renal function is monitored:
Urea : 8.1 mmol/L (2.5-6.7)
Creatinine : 240 micromol/L (baseline : 102) eGFR : 53 (> 90)
Urine Output : 20 mL/hour (> 0.5 mL/kg/hr) What is the most likely cause of his condition?
A Acute tubular necrosis
B Acute interstitial nephritis
C Glomerulonephritis
D Renal artery stenosis
E Vasculitis

A

D

How well did you know this?
1
Not at all
2
3
4
5
Perfectly
156
Q

A 47-year-old man comes to A&E having experienced palpitations. He has a past medical history of hypertension which is being treated with ramipril and spironolactone. An ECG shows tented T waves and flattened P waves. A blood test reveals:
Na+ : 137 mmol/L (135-145) K+ : 6.8 mmol/L (3.5-5) Ca2+ : 2.3 mmol/L (2.2-2.6) pH : 7.35 (7.35-7.45)
What is the first step in the management of this patient?
A IV salbutamol
B 50 ml 50% dextrose with 10U insulin
C 50 ml 5% dextrose with 10U insulin
D 10 ml 10% calcium gluconate
E IV sodium bicarbonate

A

D

How well did you know this?
1
Not at all
2
3
4
5
Perfectly
157
Q
  1. Whilst eating dinner with his family, an 11-year-old boy suddenly drops his cutlery
    and begins to stare blankly into space. His eyelids begin to flutter, his eyes roll upwards and this continues for 10 seconds. His dad notices the event and asks him about it, but he can’t remember what happened. What type of seizure is this describing?
    A Absence
    B Simple partial
    C Complex partial
    D Myoclonic
    E Atonic
A

A

How well did you know this?
1
Not at all
2
3
4
5
Perfectly
158
Q
A 71-year-old man has had a 2-week history of shortness of breath that improves when lying flat. On examination, the patient has an oxygen saturation of 88%, his fingers are clubbed and there are multiple spider naevi on his chest. Shifting dullness is demonstrated, his spleen is enlarged and there are dilated veins around his umbilicus. The patient has a history of alcohol abuse. What is the most likely diagnosis? 
A  Congestive cardiac failure 
B  Portal hypertension 
C  Hepatopulmonary syndrome 
D  GI bleed 
E  Alcoholic hepatitis
A

C

How well did you know this?
1
Not at all
2
3
4
5
Perfectly
159
Q
  1. A 72-year-old man is admitted to the orthopaedic surgery ward after fracturing his distal humerus whilst gardening. He mentioned that he suddenly felt a severe pain in his right arm and denies any significant preceding trauma. He adds that he has
    recently been urinating a lot more frequently (often up to 12 times per day) and has suffered from constipation, which he attributes to being ‘part of growing old’. An X- ray of the affected arm reveals a pathological fracture with lytic deposits throughout the bone.
    Blood tests reveal:
    ESR = 48 mm/hr (0-22) Ca2+ = 3.1 mmol/L (2.2-2.6)
    What is the most likely diagnosis?
    A Multiple myeloma
    B Paget’s disease
    C Osteoporosis
    D Vitamin D deficiency
    E Thyrotoxicosis
A

B

How well did you know this?
1
Not at all
2
3
4
5
Perfectly
160
Q
A 49-year-old man is referred to the respiratory department by his GP. He has been suffering from gradual-onset, worsening shortness of breath over the past 3 months. His 50 pack-year smoking history makes COPD the top differential. Spirometry confirms these suspicions. What is the most appropriate first step in the pharmacological management of this patient? 
A  Inhaled corticosteroid 
B  Inhaled ipratropium bromide 
C  Inhaled tiotropium 
D  Symbicort 
E  Long-term oxygen therapy
A

A

How well did you know this?
1
Not at all
2
3
4
5
Perfectly
161
Q
Which of the following is a sign of a lower motor neuron lesion? 
A  Hyperreflexia 
B  Spasticity 
C  Fasciculations 
D  Clonus 
E  Babinski’s sign
A

C

How well did you know this?
1
Not at all
2
3
4
5
Perfectly
162
Q
A 47-year-old man has been suffering from rhinitis and recurrent nosebleeds for the past 3 months. At first, he attributed this to the cold weather, however, over the last 3 weeks he has started coughing up a small amount of blood. A series of bedside tests are performed, including a urine dipstick, which reveals proteinuria and haematuria. Blood tests and antibody screens reveal a raised ESR and cANCA. 
What is the most likely diagnosis? 
A  Microscopic polyangiitis 
B  Goodpasture’s syndrome 
C  Granulomatosis with polyangiitis 
D  Churg-Strauss syndrome 
E Behçet’s disease
A

C

How well did you know this?
1
Not at all
2
3
4
5
Perfectly
163
Q
Which of the following organisms most commonly causes gas gangrene? 
A  Streptococcus pyogenes 
B  Staphylococcus aureus 
C  Staphylococcus epidermidis 
D  Clostridium perfringens 
E  Haemophilus influenzae
A

D

How well did you know this?
1
Not at all
2
3
4
5
Perfectly
164
Q
A 16-year-old schoolgirl books an appointment with her GP after noticing a lump in her left breast during self-examination. She, sensibly, decided to seek medical attention. She complains of no other symptoms. On examination, a 1 cm x 1 cm firm, smooth and very mobile lump is palpated in the upper-outer quadrant of her left breast. There is no pain on palpation nor is there any axillary or cervical lymphadenopathy. 
What is the most likely diagnosis? 
A Fibrocystic disease 
B Fibroadenoma
C Breast cancer
D Breast abscess 
E Fat necrosis
A

B

How well did you know this?
1
Not at all
2
3
4
5
Perfectly
165
Q
Which of the following drug classes is most likely to cause iatrogenic hypoglycaemia in diabetes patients? 
A Sulfonylureas 
B Metformin
C Glucagon 
D Hydrocortisone
 E Orlistat
A

A

How well did you know this?
1
Not at all
2
3
4
5
Perfectly
166
Q
  1. A 24-year-old swimwear model presents to A&E with severe right iliac fossa pain. The pain was initially poorly localised to the umbilical region, before moving to
    the right iliac fossa. This has been accompanied by nausea, anorexia and fever. A diagnosis of appendicitis is made and she is referred for an appendicectomy. What is the most suitable surgical incision for this patient?
    A Lanz
    B Kocher
    C Pfannenstiel
    D Rutherford-Morrison E Gridiron
A

A

How well did you know this?
1
Not at all
2
3
4
5
Perfectly
167
Q
  1. A 55-year-old man presents to his GP having coughed up blood on several
    occasions over the past 6 months. He said that he has had a ‘smokers’ cough’ for years but the appearance of blood has been a recent change. On direct questioning, he admits to unintentionally losing about 5 kg of weight over the past 6 months. A chest X-ray shows a 2 cm cavitating lesion in the right upper lobe. There appears to be a few other smaller nodules surrounding the large cavitating lesion. The left lung appears slightly fibrosed, but is otherwise normal. What is the most likely diagnosis?
    A Small cell lung cancer
    B Squamous cell lung cancer
    C Atypical pneumonia
    D Lung abscess
    E Goodpasture’s syndrome
A

B

How well did you know this?
1
Not at all
2
3
4
5
Perfectly
168
Q
  1. A 46-year-old airline pilot presents to A&E with severe pain in his right flank. He
    adds that the pain moves down towards his right groin. Though examination is difficult, as he is writhing around in pain, no abnormalities are detected. However, a urine dipstick reveals haematuria. Which investigation would you do next?
    A Renal ultrasound
    B Cystoscopy
    C CT-KUB
    D MRI
    E Urine MC&S
A

C

How well did you know this?
1
Not at all
2
3
4
5
Perfectly
169
Q
A 32-year-old man, with a history of IV drug abuse, presents to A&amp;E with a high fever and rigors. He has also been very breathless and has experienced epigastric pain that is worse on exertion. On examination, giant V waves are seen in the JVP and tender, pulsatile hepatomegaly is palpated. What is the most likely diagnosis? 
A  Mitral stenosis 
B  Tricuspid regurgitation 
C  Pulmonary hypertension 
D  Portal hypertension 
E  Viral hepatitis
A

B

How well did you know this?
1
Not at all
2
3
4
5
Perfectly
170
Q
A 31-year-old female presents to her GP with a 2-month history of fatigue and worsening muscle weakness. She also complains that her left eyelid droops considerably more than her right. She feels fine in the morning but her strength decreases throughout the day, especially if she exerts herself more so than usual. Myasthenia gravis is suspected. Which of the following investigations may provide evidence supporting this diagnosis? 
A  Dix-Hallpike test 
B  Schirmer’s test 
C  Romberg’s test 
D  Tensilon test 
E  Trendelenberg test
A

D

How well did you know this?
1
Not at all
2
3
4
5
Perfectly
171
Q
A 15-year-old school boy is rushed into A&amp;E having accidentally ingested some peanuts, to which he is extremely allergic. His face and lips are swollen, he is wheezing and struggling to breathe. Vital signs: HR = 132 bpm, BP = 88/53 mm Hg. His airway has been secured and he is being given 100% oxygen. What is the next most appropriate step in the management of this patient? 
A  IV chlorpheniramine 
B  IV hydrocortisone 
C  IV saline 
D  IV adrenaline 
E  IM adrenaline
A

E

How well did you know this?
1
Not at all
2
3
4
5
Perfectly
172
Q
Which of the following is a cause of microcytic anaemia? 
A  Myelodysplasia 
B  Multiple myeloma 
C  Thalassaemia 
D  Myelofibrosis 
E  Aplastic anaemia
A

C

How well did you know this?
1
Not at all
2
3
4
5
Perfectly
173
Q
A 63-year-old type 2 diabetic presents with a rash on her shins that has gradually got worse over the past 3 months. On closer inspection, there are three areas of raised, reddened and hardened skin with a yellowish centre. What is the most likely diagnosis? 
A  Acanthosis nigricans 
B  Diabetic dermopathy 
C  Necrobiosis lipoidica diabeticorum 
D  Granuloma annulare 
E  Pruritus
A

C

How well did you know this?
1
Not at all
2
3
4
5
Perfectly
174
Q
A 76-year-old woman is brought into A&amp;E with central crushing chest pain that radiates to her jaw and left arm. An ECG is performed, which shows ST elevation in leads ll, lll and aVF. Her SaO2 is 90%. Before she is sent to the cathlab for percutaneous coronary intervention, she is started on a combination of drugs. Which of the following should not be given? 
A  Morphine 
B  Oxygen 
C  Aspirin 
D  Clopidogrel 
E  Warfarin
A

E

How well did you know this?
1
Not at all
2
3
4
5
Perfectly
175
Q
A 42-year-old amateur rugby player presents to his GP complaining that his team mates have been making fun of his 'man boobs'. He admits that he appears to have developed breasts over the past few months and it is causing him considerable embarrassment and distress. Which of the following drugs is most likely to have caused this unfortunate circumstance? 
A Cimetidine 
B Aspirin
C Salbutamol 
D Ramipril
E Omeprazole
A

A

How well did you know this?
1
Not at all
2
3
4
5
Perfectly
176
Q
Which part of the prostate gland undergoes progressive hyperplasia in BPH? 
A  Central zone 
B  Transitional zone 
C  Peripheral zone 
D  Ejaculatory duct 
E  Anterior fibromuscular stroma
A

B

How well did you know this?
1
Not at all
2
3
4
5
Perfectly
177
Q
A 62-year-old obese man visits his GP complaining of a cramping pain in his buttocks that comes on when he walks his Pomeranian, Skippy. This pain was first noticed 4 months ago and it is relieved by sitting down. He has a 40 pack-year smoking history. On direct questioning, he sheepishly admits to suffering from erectile dysfunction over the last 2 months. Examination is normal except for noticeably weak pedal pulses. What is the most likely diagnosis? 
A  Chronic compartment syndrome 
B  Leriche syndrome 
C  Critical limb ischaemia 
D  Spinal stenosis 
E  Chronic deep vein thrombosis
A

B

How well did you know this?
1
Not at all
2
3
4
5
Perfectly
178
Q
A 43-year-old man, who has recently moved to the UK from Greece, has been referred for an outpatient appointment to discuss his recurrent mouth ulcers. He says that, for the past 8 months, he has regularly suffered from oral ulcers and he, quite timidly, mentions that he has also noticed ulcers on his penis. During the consultation, you notice that his eyes look quite red. When asked about it, he mentions that his eyes have been very itchy and painful recently, but attributes this to hay fever. What is the most likely diagnosis? 
A  Inflammatory bowel disease 
B  Behçet’s disease 
C  Herpes simplex virus 
D  Syphilis 
E  Reactive arthritis
A

B

How well did you know this?
1
Not at all
2
3
4
5
Perfectly
179
Q
Which of the following ECG findings is associated with pulmonary embolism? 
A  Bradycardia 
B  Tall tented T waves 
C  Reverse tick sign 
D  S1Q3T3 
E  T wave inversion
A

D

How well did you know this?
1
Not at all
2
3
4
5
Perfectly
180
Q
Which of the following is not a feature of Parkinson’s disease? 
A  Hypomimia 
B  Hypophonia 
C  Micrographia 
D  Up-gaze impairment 
E  Wide-based gait
A

E

How well did you know this?
1
Not at all
2
3
4
5
Perfectly
181
Q
What is the inheritance pattern of polycystic kidney disease? 
A  Autosomal dominant 
B  Autosomal recessive 
C  X-linked dominant 
D  X-linked recessive 
E  Mitochondrial
A

A

How well did you know this?
1
Not at all
2
3
4
5
Perfectly
182
Q

Which of the following statements about inflammatory bowel disease is false?
A Crohn’s disease causes transmural inflammation, whereas UC causes inflammation of the mucosa or submucosa
B Crohn’s disease causes skip lesions, whereas UC is continuous
C Crohn’s disease is associated with abscesses, fistulae, adhesions and
strictures, whereas UC is associated with toxic megacolon
D Crohn’s disease favours the rectum, whereas UC favours the terminal ileum
E Barium follow-through will show rose-thorn ulcers and a cobblestone mucosa in Crohn’s disease, but a lead pipe mucosa in UC

A

D

How well did you know this?
1
Not at all
2
3
4
5
Perfectly
183
Q
A 68-year-old care home resident is brought into A&amp;E hyperventilating and complaining of a ‘ringing sound’ in her ears. She has a low-grade fever and appears to be confused. She has a past medical history of depression and a TIA (2 months ago). What is the most likely diagnosis? 
A  Aspirin overdose 
B  Paracetamol overdose 
C  TCA overdose 
D  Cerebrovascular accident 
E  Pneumonia
A

A

How well did you know this?
1
Not at all
2
3
4
5
Perfectly
184
Q
A 25-year-old male presents to his GP complaining of a lump in his armpit. He says it doesn’t usually hurt except for when he goes out binge drinking with his friends. In the past few months, he has noticed that his clothes have become quite loose-fitting and he has been getting very hot and sweaty more than usual. On examination, he has firm, rubbery axillary lymphadenopathy, splenomegaly and scratch marks on his arms. 
A Multiple myeloma
B  Chronic lymphocytic leukaemia 
C  Chronic myeloid leukaemia 
D  Non-Hodgkin’s lymphoma 
E  Hodgkin’s lymphoma
A

E

How well did you know this?
1
Not at all
2
3
4
5
Perfectly
185
Q
A 68-year-old man presents to his GP complaining of a cough that has been bothering him for 3 months. He says that he has coughed up large volumes of ‘rusty- coloured’ sputum. According to his hospital notes, he has been admitted 4 times in the past 12 months due to pneumonia. On examination, his fingers are clubbed and coarse crepitations are heard at the lung bases. What is the most likely underlying diagnosis? 
A  COPD 
B  Bronchiectasis 
C  Pneumonia 
D  Interstitial lung disease 
E  TB
A

B

How well did you know this?
1
Not at all
2
3
4
5
Perfectly
186
Q

A 55-year-old woman is receiving treatment for chronic myeloid leukaemia. The consultant is concerned that this patient may have developed tumour lysis syndrome and requests some blood tests. What would you expect to see in the blood results of a patient with tumour lysis syndrome?

A LowK ,HighPO4 ,HighCa andHighUricAcid

B HighK ,HighPO4 ,LowCa andHighUricAcid

C HighK ,LowNa ,LowCa andHighMg

D LowK ,HighNa ,LowMg andHighUricAcid
E High Ca , Low PO4 and High Uric Acid

A

B

How well did you know this?
1
Not at all
2
3
4
5
Perfectly
187
Q
Which of the following is a cardiac cause of finger clubbing? 
A  Congenital cyanotic heart disease 
B  Viral pericarditis 
C  Dilated cardiomyopathy 
D  Rheumatic fever 
E  Wolff-Parkinson-White syndrome
A

A

How well did you know this?
1
Not at all
2
3
4
5
Perfectly
188
Q

A 39-year-old homeless man is brought into A&E having been found lying in a pool of blood on the street. He is known to the A&E department having frequently been admitted for alcohol-related issues. There are no obvious signs of trauma and blood is seen in and around his mouth. Vital Signs: HR = 110 bpm; BP = 87/61 mm Hg. On examination, splenomegaly, shifting dullness and dilated veins on the anterior abdomen are identified. The registrar suspects a variceal bleed secondary to portal hypertension. What is the first step in this patient’s management?
A TIPS procedure
B Band ligation
C Terlipressin and prophylactic antibiotics
D Beta-blockers
E Terlipressin and beta-blockers

A

C

How well did you know this?
1
Not at all
2
3
4
5
Perfectly
189
Q
A 23-year-old woman comes to see her GP about some breast lumps that she has noticed over the past 6 months. She mentions that her breasts become quite painful and feel ‘lumpy’, especially in the few days before her period. The pain is relieved when she has her period. What is the most likely diagnosis? 
A  Fibrocystic disease 
B  Fibroadenoma 
C  Breast cancer 
D  Breast abscess 
E  Duct ectasia
A

A

How well did you know this?
1
Not at all
2
3
4
5
Perfectly
190
Q
A 92-year-old female, with a history of osteoporosis, is brought into A&amp;E by her grandson. He says that she has been drifting in and out of consciousness for the past 2 weeks and has been complaining of a headache that has been keeping her up at night and getting progressively more severe. On examination, her left pupil is dilated and displaced downwards and outwards. What investigation should be performed first? 
A  Lumbar puncture 
B  Carotid artery Doppler 
C  CT head 
D  EEG 
E  Transthoracic echocardiogram
A

C

How well did you know this?
1
Not at all
2
3
4
5
Perfectly
191
Q

A 29-year-old man presents with a 4-day history of high fever. On inspection, you notice some needle track marks on his arms and a pansystolic murmur is heard
on auscultation, which had not previously been documented in his hospital notes. What is the most likely diagnosis?
A Mitral regurgitation
B Pericarditis
C Infective endocarditis
D Aorticstenosis
E Mitral valve prolapse

A

C

How well did you know this?
1
Not at all
2
3
4
5
Perfectly
192
Q
Which of the following is not part of the criteria for diagnosing SLE? 
A  Pleurisy 
B  Thrombocytopaenia 
C  Anti-dsDNA antibodies 
D  Oral ulcers 
E  Heliotrope rash
A

E

How well did you know this?
1
Not at all
2
3
4
5
Perfectly
193
Q
13. A 43-year-old woman presents with a 2-month history of diarrhoea and weight loss. She has also been feeling anxious about her appearance, as many people have commented that she always looks like she is staring. On examination, her eyes appear slightly protruded and lid lag is demonstrated. She has a fine tremor in both her hands and a lumpy skin lesion is noticed on her shins. What is the most likely 
diagnosis? 
A  Toxic multinodular goitre 
B  Graves’ disease 
C  De Quervain’s thyroiditis 
D  Hashimoto’s thyroiditis 
E  Riedel’s thyroiditis
A

B

How well did you know this?
1
Not at all
2
3
4
5
Perfectly
194
Q
. A 76-year-old man is admitted to hospital with a cough productive of green sputum. He has also experienced some shortness of breath and a fever. A week before his admission, his carers noted that he had a high fever, malaise and myalgia for a few days. A chest X-ray shows a cavitating lesion with an air fluid level. What is the most likely causative organism? 
A  Staphylococcus aureus 
B  Streptococcus pneumoniae 
C  Legionella pneumophila 
D  Mycoplasma pneumonia 
E  Haemophilus influenzae
A

A

How well did you know this?
1
Not at all
2
3
4
5
Perfectly
195
Q

The urine output of a 78-year-old inpatient on the surgical ward has decreased gradually over the past 24 hours despite maintaining an adequate fluid intake. The nurses add that he has recently become rather confused and complains of nausea. U&Es are requested:
Creatinine : 231 micromol/L (baseline : 97) Urea : 12.5 mmol/L (2.5-6.7)
Na+ : 139 mmol/L (135-145)
K+ : 6.1 mmol/L (3.5-5)
An AKI is diagnosed. He is currently on ramipril (for his hypertension), bisoprolol (for his paroxysmal AF) and ibuprofen.
Which of the following steps is inappropriate in this patient’s management?
A Assess and optimise fluid status
B 10 mL of 10% calcium gluconate IV
C Stop Ramipril
D Stop bisoprolol
E Stop ibuprofen

A

D

How well did you know this?
1
Not at all
2
3
4
5
Perfectly
196
Q
A 57-year-old man is complaining of numbness and weakness in his arms. It began in his hands, 2 weeks ago, but for the last 3 days his forearms have also felt numb. On examination, there is no sensation below his elbows, tone is reduced bilaterally and the biceps and brachioradialis reflexes cannot be elicited. He adds that he recently recovered from a bout of diarrhoea and vomiting. What is the most likely diagnosis? 
A  Multiple sclerosis 
B  Motor neuron disease 
C  Parkinson’s disease 
D  Guillain-Barré syndrome 
E  Huntington’s disease
A

D

How well did you know this?
1
Not at all
2
3
4
5
Perfectly
197
Q
A 77-year-old patient with cirrhosis presents to A&amp;E with diffuse abdominal pain, abdominal heaviness and fever. Associated symptoms include nausea and vomiting. On examination shifting dullness is demonstrated and a fluid thrill is observed. What investigation should form part of the initial diagnostic work-up? 
A  Abdominal X-ray 
B  Abdominal ultrasound 
C  Abdominal CT 
D  Diagnostic paracentesis 
E  Stool sample for MC&amp;S
A

D

How well did you know this?
1
Not at all
2
3
4
5
Perfectly
198
Q

Which of the following is the correct chronological sequence of retinal changes that occur in hypertensive retinopathy?
A Papilloedema Silver Wiring Flame Haemorrhages AV Nipping
B Silver Wiring AV Nipping Flame Haemorrhages Papilloedema
C Silver Wiring Flame Haemorrhages AV Nipping Papilloedema
D AV Nipping Papilloedema Silver Wiring Flame Haemorrhages
E AV NippingSilver wiringPapilloedemaFlame Haemorrhages

A

B

How well did you know this?
1
Not at all
2
3
4
5
Perfectly
199
Q
A 44-year-old woman is complaining of pain and a tingling feeling in the lateral half of her right hand. She often finds that she wakes up in the middle of the night because of the pain, which is then relieved by shaking her hand vigorously. Which nerve has been affected? 
A  Ulnar nerve 
B  Radial nerve 
C  Musculocutaneous nerve 
D  Median nerve 
E  Posterior interosseous nerve
A

D

How well did you know this?
1
Not at all
2
3
4
5
Perfectly
200
Q

Which of the following full blood count and clotting screen results is consistent with a diagnosis of disseminated intravascular coagulation?
A High platelets, High Hb, High APTT/PT, High fibrinogen
B High platelets, High Hb, Low APTT/PT, High fibrinogen
C Low platelets, High Hb, Low APTT/PT, Low fibrinogen
D Low platelets, Low Hb, High APTT/PT, Low fibrinogen
E Low platelets, Low Hb, Low APTT/PT, Low fibrinogen

A

D

How well did you know this?
1
Not at all
2
3
4
5
Perfectly
201
Q

A 40-year-old woman is admitted to A&E with shortness of breath that began suddenly a day after she returned from a holiday to the Maldives. What is the first step in her management?
A D-dimer
B High flow oxygen and low molecular weight heparin
C IV heparin
D CTPA
E Venous ultrasound of the lower limbs

A

B

How well did you know this?
1
Not at all
2
3
4
5
Perfectly
202
Q
What is the gold standard diagnostic test for acromegaly? 
A  Insulin suppression test 
B  Oral glucose tolerance test 
C  Short synacthen test 
D  IGF-1 levels 
E  Thyroid function test
A

B

How well did you know this?
1
Not at all
2
3
4
5
Perfectly
203
Q

Which of the following sets of results would be consistent with alcoholic hepatitis?
A Elevated MCV, ALT:AST > 2 and elevated GGT
B Elevated MCV, AST:ALT > 2 and elevated GGT
C Reduced MCV, AST:ALT >2 and elevated GGT
D Elevated MCV, ALT:AST > 2 and reduced GGT
E Reduced MCV, AST:ALT > 2 and reduced GGT

A

B

How well did you know this?
1
Not at all
2
3
4
5
Perfectly
204
Q
An inpatient on the orthopaedic surgery ward has recently developed a cough, high fevers and chills. Blood cultures are taken which identify MRSA. Which of the following antibiotics is often used in the treatment of MRSA infections? 
A Vancomycin 
B Flucloxacillin 
C Tazocin
D Metronidazole 
E Cefuroxime
A

A

How well did you know this?
1
Not at all
2
3
4
5
Perfectly
205
Q
A 77-year-old man is referred to the outpatient clinic by his GP having presented with chest pain and worsening shortness of breath. He has a history of COPD, diagnosed 12 years ago. On examination, his JVP is elevated, a parasternal heave is palpated and auscultation reveals an early diastolic murmur. An ECG is performed which shows right-axis deviation, a tall R wave in V1 and peaked P waves in lead ll. What is the most likely diagnosis? 
A  Aortic regurgitation 
B  Mitral stenosis 
C  Pulmonary hypertension 
D  Right heart failure 
E  Exacerbation of COPD
A

C

How well did you know this?
1
Not at all
2
3
4
5
Perfectly
206
Q
Which of the following is not a histopathological type of malignant melanoma? 
A  Superficial spreading 
B  Acral lentiginous 
C  Bowen’s disease 
D  Nodular 
E  Lentigo maligna
A

C

How well did you know this?
1
Not at all
2
3
4
5
Perfectly
207
Q
An 82-year-old man has recently suffered from a right-sided stroke and is undergoing physiotherapy. He is referred for an upper limb neurological examination. The power in his right arm is normal. He can abduct his left arm by himself, but fails to maintain that position as soon as any resistance is applied. What is the MRC grading of his left shoulder abduction? 
A  Grade 1 
B  Grade 2 
C  Grade 3 
D  Grade 4 
E  Grade 5
A

C

How well did you know this?
1
Not at all
2
3
4
5
Perfectly
208
Q
A 54-year-old female, with a BMI of 28, presents with a 2-year history of epigastric pain that radiates to the neck. It gets worse when lying down, and she also complains of painless regurgitation of food. What is the most appropriate investigation to confirm the diagnosis? 
A  Chest X-ray 
B  Barium swallow 
C ECG
D OGD
E Manometry
A

B

How well did you know this?
1
Not at all
2
3
4
5
Perfectly
209
Q
Which of the following is not an indication for dialysis in the context of acute kidney injury? 
A  Refractory hyperkalaemia 
B  Refractory pulmonary oedema 
C  Uraemic pericarditis 
D  Severe metabolic acidosis 
E  Macroscopic haematuria
A

E

How well did you know this?
1
Not at all
2
3
4
5
Perfectly
210
Q

A 25-year-old man, with no past medical history, presents to A&E with sudden- onset shortness of breath and right-sided chest pain, which started whilst he was playing football. Vital Signs: RR = 24 /min; HR = 125 bpm; BP = 85/59 mm Hg. There is a hyper-resonant percussion note and reduced breath sounds over the right upper zone and the trachea is deviated to the left. A chest X-ray confirms the presence of a pneumothorax measuring 3 cm. What is the most appropriate management option?
A Give analgesia and reassure
B Admit to hospital, monitor vital signs and repeat chest X-ray in 3 hours
C Insert a chest drain
D Insert a large-bore cannula into the right 2nd intercostal space in the
midclavicular line
E Surgical pleurectomy

A

D

How well did you know this?
1
Not at all
2
3
4
5
Perfectly
211
Q
Which of the following organisms is a recognised cause of hospital-acquired pneumonia? 
A  Streptococcus pneumoniae 
B  Pseudomonas aeruginosa 
C  Haemophilus influenzae 
D  Legionella pneumophila 
E  Chlamydophila psittaci
A

B

How well did you know this?
1
Not at all
2
3
4
5
Perfectly
212
Q
A 19-year-old girl visits her GP after experiencing painful urination over the past week. She has also been urinating more frequently than usual and complains that her urine looks cloudy and smells particularly bad. A urinary tract infection is suspected. Which investigation can definitively confirm the diagnosis? 
A  Urine dipstick 
B  CRP 
C  Blood cultures 
D  MSU 
E  U&amp;Es
A

D

How well did you know this?
1
Not at all
2
3
4
5
Perfectly
213
Q

. A 31-year-old man presents with a 2-day history of diffuse watery diarrhoea and nausea. He admits to recently eating a BBQ at a friend’s house. How should this patient be managed?
A Bed rest and oral rehydration
B Bed rest, oral rehydration and antibiotics
C Anti-diarrhoeal agents
D Call an ambulance and admit to hospital
E Refer to a gastroenterologist for further investigation

A

A

How well did you know this?
1
Not at all
2
3
4
5
Perfectly
214
Q
  1. A 16-year-old boy presents to his GP after noticing the growth of several small, fleshy tags on his torso. He has also noticed that he has many more ‘birthmarks’ now compared to when he was younger. On examination, there are 8 coffee-coloured, flat
    skin lesions (‘birthmarks’) which are about 2-3 cm in diameter. Freckling around both axillae is also noted. What is the most likely diagnosis?
    A Tuberous sclerosis
    B Neurofibromatosis type 1
    C Neurofibromatosis type 2
    D Xeroderma pigmentosum
    E Dercum disease
A

B

How well did you know this?
1
Not at all
2
3
4
5
Perfectly
215
Q
Which of the following is an extra-articular feature of ankylosing spondylitis? 
A Erythema ab igne 
B  Subcutaneous nodules 
C  Apical lung fibrosis 
D  Mitral regurgitation 
E Tophi
A

C

How well did you know this?
1
Not at all
2
3
4
5
Perfectly
216
Q
A 43-year-old man has been involved in a bar fight. He is rushed, unconscious, into A&amp;E with a stab wound to the chest. On examination, his JVP is raised and he is hypotensive (BP : 86/70 mm Hg). Auscultation of his chest reveals very quiet heart sounds. What is the most likely diagnosis? 
A Acute heart failure 
B Haemopericardium 
C Pneumothorax 
D  Septic shock 
E  Hypovolaemic shock
A

B

How well did you know this?
1
Not at all
2
3
4
5
Perfectly
217
Q
A 52-year-old man attends the respiratory clinic complaining of a dry cough that has been bothering him for 2 months. He has never smoked and has not experienced any shortness of breath or chest pain. Respiratory examination detects no abnormalities. He has a past medical history of hypertension, for which he started treatment 4 months ago. What is the most likely diagnosis? 
A  Asthma 
B  Interstitial lung disease 
C  Bronchiectasis 
D  Drug side-effect 
E  Atypical pneumonia
A

D

How well did you know this?
1
Not at all
2
3
4
5
Perfectly
218
Q
A 41-year-old female is referred to the dermatology clinic because she has developed multiple purple nodules on her shins. They are tender and have a diameter of 1-2 inches. Erythema nodosum is suspected. Which of the following is not a cause of erythema nodosum? 
A  Tuberculosis 
B  Reaction to sulphonamides 
C  Inflammatory bowel disease 
D  Ankylosing spondylitis 
E  Behçet’s disease
A

D

How well did you know this?
1
Not at all
2
3
4
5
Perfectly
219
Q
A 49-year-old woman is admitted to A&amp;E complaining of severe right upper quadrant pain that began last evening and has not subsided. She admits to eating a lot of fast food and mentions that, in the past, she has experienced a stabbing pain for a couple of hours after eating. The pain during these episodes is less intense than the pain she is currently experiencing, and it tends to be localised around her epigastrium. She drinks no more than 12 units of alcohol per week and has not lost any weight recently. On examination, she is jaundiced and Murphy’s sign is positive. LFTs are requested: 
Bilirubin : 45 micromol/L (3-17) AST : 50 iU/L (5-35)
ALT : 45 iU/L (5-35)
ALP : 400 iU/L (30-150) 
What is the most likely diagnosis? 
A  Gallstones 
B  Alcoholic hepatitis 
C  Viral hepatitis 
D  Hepatocellular carcinoma 
E  Gilbert’s syndrome
A

A

How well did you know this?
1
Not at all
2
3
4
5
Perfectly
220
Q
A 34-year-old Nigerian man brings his 3-year-old son to A&amp;E. He has been crying and complaining of severe pain in his hands. On examination, his fingers are swollen and warm. The junior doctor suspects a painful crises of sickle cell disease. What is the mode of inheritance of sickle cell disease? 
A  Autosomal recessive 
B  Autosomal dominant 
C  X-linked recessive 
D X-linked dominant 
E Y-linked
A

A

How well did you know this?
1
Not at all
2
3
4
5
Perfectly
221
Q

A 16-year-old girl is rushed to A&E by her parents. She is unconscious and shaking uncontrollably. Her mother says that she has been fitting for over half an hour. What is the most appropriate first step in the management of this patient?
A IV phenytoin
B IV thiopentone
C IV lorazepam/PR diazepam
D Reassure her parents and let the seizure terminate by itself
E Oral sodium valproate

A

C

How well did you know this?
1
Not at all
2
3
4
5
Perfectly
222
Q
A 61-year-old, with a 40 pack-year smoking history, presents with confusion. He has also had a 3-month history of weight loss and haemoptysis. A blood test shows the following results: 
Na+ : 121 mmol/L (135 - 145)
K+ : 4.1 mmol/L (3.5 - 5)
Ca2+ : 2.3 mmol/L (2.2-2.6)
What is the most likely diagnosis? 
A  Addison’s disease 
B  Hypothyroidism 
C  Heart failure 
D  SIADH 
E  Cirrhosis
A

D

How well did you know this?
1
Not at all
2
3
4
5
Perfectly
223
Q

A 27-year-old man presents complaining of sharp chest pain. He mentions that he has taken a few days off work recently because of the flu. What would you expect to see on his ECG?
A ST elevation in leads II, III and aVF
B Widespread saddle-shaped ST elevation
C ST depression
D Tented T waves
E Absent P waves

A

B

How well did you know this?
1
Not at all
2
3
4
5
Perfectly
224
Q

Why are urinary tract infections more common in women?
A Women have a shorter urethra
B Men have a larger bladder
C Men have a shorter distance between their urethral opening and their anus
D Women are more likely to be catheterised
E Men have longer ureters

A

A

How well did you know this?
1
Not at all
2
3
4
5
Perfectly
225
Q
A 44-year-old bus driver from the West Indies has suffered from shortness of breath and a dry cough for the last 4 months. He also complains of some ‘sore lumps on his shins’. Closer inspection reveals tender violet nodules on both shins. A chest X-ray is requested, which shows bilateral hilar lymphadenopathy. Blood tests are also requested, including U&amp;Es – which parameter would you expect to be raised? 
A Sodium
B Potassium 
C Calcium
D pH
E Urea
A

C

How well did you know this?
1
Not at all
2
3
4
5
Perfectly
226
Q
A 35-year-old woman presents to clinic with a 6-month history of watery diarrhoea and abdominal pain that improves after defecation. She admits to defecating 4-5 times per day compared to her normal frequency of once per day. On examination, a papulovesicular rash is seen on both elbows. Which investigation would be most useful in aiding the diagnosis? 
A  Stool sample for MC&amp;S 
B  Serology for anti-tTG antibodies 
C  Full blood count 
D  Blood cultures 
E  Barium follow-through
A

B

How well did you know this?
1
Not at all
2
3
4
5
Perfectly
227
Q
Which of the following is a risk factor for breast cancer? 
A Breastfeeding 
B Late menarche 
C Early menopause 
D Not having children 
E Age<50yrs
A

D

How well did you know this?
1
Not at all
2
3
4
5
Perfectly
228
Q
A 26-year-old man presents with an acutely swollen left knee. The pain and swelling started 2 days ago, but he didn’t pay much attention to it as he thought it was just a muscle strain. He has also developed a fever over the past 24 hours. On examination, his left knee is red, swollen and extremely painful on passive flexion. Septic arthritis is suspected and a joint aspirate is requested. Which of the following organisms most commonly causes this condition? 
A  Haemophilus influenza 
B  Staphylococcus aureus 
C  Neisseria meningitidis 
D  Escherichia coli 
E  Mycobacterium tuberculosis
A

B

How well did you know this?
1
Not at all
2
3
4
5
Perfectly
229
Q

A 44-year-old female presents to her GP complaining of worsening hearing. A full cranial nerves examination is performed. When Weber’s test is performed, she hears the sound louder in her left ear. Then, Rinne’s test is performed and she reports that, in both ears, the sound is loudest when the tuning fork is held in front of the auditory canal rather than when the fork is held against her mastoid processes. Which of the following best describes the patient’s condition?
A Conductive hearing loss in the right ear
B Conductive hearing loss in the left ear
C Sensorineural hearing loss in the right ear
D Sensorineural hearing loss in the left ear
E Bilateral sensorineural hearing loss

A

C

How well did you know this?
1
Not at all
2
3
4
5
Perfectly
230
Q

Which of the following best defines chronic kidney disease?
A GFR < 60 mL/min/1.73m2 for more than 3 months
B GFR < 60 mL/min/1.73m2 for more than 6 months
C GFR < 90 mL/min/1.73m2 for more than 3 months
D GFR < 90 mL/min/1.73m2 for more than 6 months
E Requirement of long-term renal replacement therapy

A

A

How well did you know this?
1
Not at all
2
3
4
5
Perfectly
231
Q

A 77-year-old man, with a history of ischaemic heart disease, presents to A&E with acute-onset dyspnoea, a wheeze and a cough productive of pink frothy sputum.
A diagnosis of acute left ventricular failure, resulting in pulmonary oedema, is made. Which of the following is not part of the immediate management of this patient?
A Administer oxygen
B Lie the patient down
C IV Diamorphine
D GTN infusion
E IV furosemide

A

B

How well did you know this?
1
Not at all
2
3
4
5
Perfectly
232
Q
Which of the following is not a cause of macrocytic anaemia? 
A  Iron deficiency 
B  Vitamin B12 deficiency 
C  Folate deficiency 
D  Methotrexate 
E  Hypothyroidism
A

A

How well did you know this?
1
Not at all
2
3
4
5
Perfectly
233
Q
A 62-year-old shop owner is brought to A&amp;E, by his daughter, having experienced worsening shortness of breath. His face and arms have also become quite swollen. On examination, he has a plethoric face and his JVP is raised and non-pulsatile. He seems disinterested when the history is taken because he does not trust doctors. His daughter adds that he has been coughing up blood and losing weight for about 6 months, however, he has refused to seek medical attention until his recent worsening of symptoms. What is the most likely diagnosis? 
A  Asbestosis 
B  Congestive cardiac failure 
C  Polycythaemia 
D  Superior vena cava syndrome 
E  Mesothelioma
A

D

How well did you know this?
1
Not at all
2
3
4
5
Perfectly
234
Q
  1. An 82-year-old man is brought to A&E with a severe headache. He complains of an intense aching pain, focused around his right eye, that has rapidly worsened. He has never experienced anything like this before. A nurse informs you that he vomited whilst waiting to be admitted, and has been complaining of nausea since. Closer inspection reveals a red, congested right eye with a cloudy cornea. He complains that his vision has worsened with the onset of this headache and he has started
    seeing haloes around all sources of light. What is the most likely diagnosis?
    A Meningitis
    B Subarachnoid haemorrhage
    C Acute glaucoma
    D Cluster headache
    E Migraine
A

C

How well did you know this?
1
Not at all
2
3
4
5
Perfectly
235
Q
Which of the following is associated with left ventricular systolic failure? 
A  Pulsus alternans 
B  Pulsus paradoxus 
C  Water-hammer pulse 
D  Pulsus parvus et tardus 
E  Pulsus bisferiens
A

A

How well did you know this?
1
Not at all
2
3
4
5
Perfectly
236
Q
26. A 46-year-old truck driver is accompanied by his wife to see his GP after he fell asleep at the wheel two days ago. He appears to be quite shaken by the ordeal as he recalled having to veer away from oncoming traffic. On further questioning, he says that his sleep hasn’t been disrupted, however, he has been feeling very tired during the day. His wife interjects and mentions that her own sleep has been disturbed because her husband has been ‘snoring ferociously’. When asked about diet and exercise, he admits to eating badly and exercising very little since he started 
working as a truck driver 3 years ago. In that time, he has gained a considerable amount of weight. 
What is the most likely diagnosis? 
A  Narcolepsy 
B  Cataplexy 
C  Absence Seizure 
D  Obstructive Sleep Apnoea 
E  Central Sleep Apnoea
A

D

How well did you know this?
1
Not at all
2
3
4
5
Perfectly
237
Q
A 28-year-old IV drug-user visits his GP practice for an annual check-up. Hepatitis serology is requested and the following results are reported. 
HBsAg - HBeAg - HBcAb IgM - HBcAb IgG + 
HBsAb +
What is the hepatitis status of this patient? 
A  Acute infection 
B  Chronic infection 
C  Cleared 
D  Vaccinated 
E  Susceptible
A

C

How well did you know this?
1
Not at all
2
3
4
5
Perfectly
238
Q

A 24-year-old student presents at his GP practice with a 2-day history of blood in his urine. Urine dipstick reveals proteinuria and haematuria. On questioning, he mentions that he has been recovering from a sore throat and a cough over the last 4 days. What is the most likely diagnosis?
A Minimal change disease
B IgA nephropathy
C Membranous nephropathy
D Post-streptococcal glomerulonephritis
E Acute tubular necrosis

A

B

How well did you know this?
1
Not at all
2
3
4
5
Perfectly
239
Q
Which of the following is not a feature of Cushing’s syndrome? 
A  Central obesity 
B  Poor wound healing 
C  Hypotension 
D  Striae 
E  Proximal myopathy
A

C

How well did you know this?
1
Not at all
2
3
4
5
Perfectly
240
Q
A 24-year-old female is brought to A&amp;E having fallen off a stage in a nightclub whilst under the influence of LSD. Her eyes open when the registrar squeezes her trapezius and she makes a few incomprehensible sounds. Her arms flex, wrists clench and legs extend and internally rotate in response to pain. What is her GCS? 
A5 
B7 
C8 
D9 
E 10
A

B

How well did you know this?
1
Not at all
2
3
4
5
Perfectly
241
Q
An 8-year-old girl visits her GP, with her mother, 4 weeks after being prescribed antibiotics for a sore throat. Her urine has become tea-coloured and she has been feeling nauseous with a headache. Urinalysis reveals proteinuria and haematuria and her blood pressure is 137/72 mm Hg. A diagnosis of post-infectious glomerulonephritis is suspected. Which organism is most likely responsible? 
A  Streptococcus agalactiae 
B  Streptococcus pyogenes 
C  Escherichia coli 
D  Diphtheria 
E  Bordetella pertussis
A

B

How well did you know this?
1
Not at all
2
3
4
5
Perfectly
242
Q
Which of the following drugs is not used in the long-term management of chronic 
heart failure?
A Indomethacin 
B Carvedilol
C Spironolactone 
D Candesartan
E Digoxin
A

A

How well did you know this?
1
Not at all
2
3
4
5
Perfectly
243
Q
A 32-year-old man has been suffering from worsening shortness of breath over the past 5 months. He used to be very physically active, but, recently, he has become breathless whilst doing relatively low intensity tasks such as walking to the bus stop. He has also developed a chronic cough, productive of clear sputum. He regularly visits the hospital to monitor his liver function because of a ‘liver disease’ that he has had since he was a child. He has never smoked and does not drink alcohol. What is the most likely diagnosis? 
A  COPD 
B  Autoimmune hepatitis 
C  1 antitrypsin deficiency 
D  Haemochromatosis 
E  Wilson’s disease
A

C

How well did you know this?
1
Not at all
2
3
4
5
Perfectly
244
Q
A 55-year-old woman presents with a 2-month history of jaundice and right upper quadrant pain. She has a history of ulcerative colitis. LFTs and serology are requested: 
ALP : 390 iU/L (30-150) AST : 40 iU/L (5-35) ALT : 40 iU/L (5-35) GGT : 150 iU/L (7-32) pANCA : Positive 
What is the most likely diagnosis? 
A  Haemochromatosis 
B  Primary biliary cirrhosis 
C  Primary sclerosing cholangitis 
D  Autoimmune hepatitis 
E  Wilson’s disease
A

C

How well did you know this?
1
Not at all
2
3
4
5
Perfectly
245
Q
  1. An 89-year-old woman is brought into A&E after she suddenly became very disorientated and was unable to recognise her own son. On examination, she is blind
    in the left half of her visual field. An ischaemic stroke is suspected. Which artery is most likely to be involved?
    A Right anterior cerebral artery
    B Right posterior cerebral artery
    C Left posterior cerebral artery
    D Right middle cerebral artery
    E Left middle cerebral artery
A

B

How well did you know this?
1
Not at all
2
3
4
5
Perfectly
246
Q
Which of the following conditions is strongly associated with Giant Cell Arteritis? 
A  Takayasu’s aortitis 
B  Myalgic encephalomyelitis 
C  Fibromyalgia 
D  Polymyalgia rheumatic 
E  Polymyositis
A

D

How well did you know this?
1
Not at all
2
3
4
5
Perfectly
247
Q

A 58-year-old woman presents with a scaly rash around her right nipple. She says that the rash has been there for the last 3 weeks. On examination, there is a crusty rash around her right nipple and palpation reveals a firm lump just below the areola that appears to be tethered to surrounding tissues. What is the most likely diagnosis?

A Intraductal papilloma
B Phyllodes tumour
C Paget’s disease of the breast D Breast abscess
E Fibroadenoma

A

C

How well did you know this?
1
Not at all
2
3
4
5
Perfectly
248
Q
A 43-year-old businesswoman has had a TIA. Soon after landing in the UK from a business trip to Australia, she suddenly became unable to move her right arm. She began slurring her speech, the right side of her face started drooping and she temporarily lost vision in her right eye. She adds that she experienced some pain in her right leg as she was disembarking the plane, however, she assumed it was a muscle strain from wearing high-heels for several days. What underlying defect is most likely to have caused her TIA? 
A  Atrial fibrillation 
B  Atrial septal defect 
C  Carotid atherosclerosis 
D  Infective endocarditis 
E  Prosthetic heart valve
A

B

How well did you know this?
1
Not at all
2
3
4
5
Perfectly
249
Q
Which of the following is not a recognised cause of acute pancreatitis? 
A Hyperlipidaemia 
B Hypothermia
C Hypocalcaemia 
D Mumps
E Azathioprine
A

C

How well did you know this?
1
Not at all
2
3
4
5
Perfectly
250
Q

A 34-year-old man is brought to A&E having collapsed in a shopping mall. He did not lose consciousness but mentioned that he felt dizzy and could feel his ‘heart racing’. He has a past medical history of asthma. His ECG shows a regular narrow
complex tachycardia with no visible P waves. Vagal manoeuvres failed to terminate the tachycardia. What is the next most appropriate step in the management of this patient?
A Verapamil
B Amiodarone
C Adenosine
D Bisoprolol
E Flecainide

A

A

How well did you know this?
1
Not at all
2
3
4
5
Perfectly
251
Q
Which of the following is a cause of primary amenorrhoea? 
A  Prolactinoma 
B  Pregnancy 
C  Haemochromatosis 
D  Polycystic ovarian syndrome 
E  Turner syndrome
A

E

How well did you know this?
1
Not at all
2
3
4
5
Perfectly
252
Q
In which part of the nephron does bendroflumethiazide have its effect? 
A  Proximal convoluted tubule 
B  Descending limb of the loop of Henle 
C  Ascending limb of the loop of Henle 
D  Distal convoluted tubule 
E  Collecting duct
A

D

How well did you know this?
1
Not at all
2
3
4
5
Perfectly
253
Q
A 62-year-old heavy smoker is being investigated for lung cancer having presented with a 4-month history of unintentional weight loss, haemoptysis and fatigue. He claims that his voice has become hoarse and a junior doctor adds that he has a textbook ‘bovine cough’. In which part of the lung is the tumour most likely to be found? 
A  Left apex 
B  Right middle lobe 
C  Right base 
D  Left base 
E Pleura
A

A

How well did you know this?
1
Not at all
2
3
4
5
Perfectly
254
Q
Which of the following most accurately describes the sodium and potassium requirements of a 70 kg man over a 24 hour period? 
A  100 mmol Na+ &amp; 40-50 mmol K+ 
B  120 mmol Na+ &amp; 5-10 mmol K+ 
C  120 mmol Na+ &amp; 10-20 mmol K+ 
D  120 mmol Na+ &amp; 60-70 mmol K+ 
E  140 mmol Na+ &amp; 60-70 mmol K+
A

E

How well did you know this?
1
Not at all
2
3
4
5
Perfectly
255
Q

A 27-year-old female presents to her GP complaining of an episode of painful loss of vision that lasted 1 day and resolved spontaneously. 6 months ago, she lost sensation across the lateral half of her left leg which also resolved spontaneously. Which of the following would you expect to see in her diagnostic work up?
A Bence Jones proteins
B Oligoclonal bands on CSF electrophoresis
C High CSF protein
D Xanthochromia
E Raised ICP

A

B

How well did you know this?
1
Not at all
2
3
4
5
Perfectly
256
Q
Which of the following is an acquired cause of long QT syndrome? 
A  Romano-Ward syndrome 
B  Hyponatraemia 
C  Hyperkalaemia 
D  Hypomagnesaemia 
E  Hypercalcaemia
A

D

How well did you know this?
1
Not at all
2
3
4
5
Perfectly
257
Q
A 70-year-old man presents with a 3-month history of polyuria. He has been urinating up to 12 times per day and has also experienced some constipation, abdominal pain and back pain. More recently, he has noticed that his face appears ‘puffier’ than usual and his ankles are swollen. He is on citalopram to treat his depression and takes no other regular medications. Blood tests reveal: ESR = 64 mm/hr (< 22). Urinalysis reveals: 
Protein: Positive
Blood: Negative
24 hr urine protein (g): 9.8 (< 3.5) What is the most likely diagnosis? 
A  Cushing’s syndrome 
B  Amyloidosis 
C  Glomerulonephritis 
D  Malignancy 
E  Congestive cardiac failure
A

B

How well did you know this?
1
Not at all
2
3
4
5
Perfectly
258
Q
What is the most common cause of urinary tract infections? 
A  Staphylococcus aureus 
B  Staphylococcus saprophyticus 
C  Enterococcus faecalis 
D  Escherichia coli 
E  Klebsiella pneumonia
A

D

How well did you know this?
1
Not at all
2
3
4
5
Perfectly
259
Q
A 36-year-old man presents to his GP with a 1-month history of shortness of breath on exertion. He has also experienced a low-grade fever and a dry cough. He has a past medical history of HIV. A pulse oximeter is attached showing an oxygen saturation of 97% at rest. The patient is then asked to walk up and down the room a few times and his oxygen saturation drops to 88%. What is the most likely diagnosis? 
A  Interstitial lung disease 
B  Pulmonary embolism 
C  Pneumocystis jirovecii pneumonia 
D  Mycobacterium avium complex 
E  Bronchiectasis
A

C

How well did you know this?
1
Not at all
2
3
4
5
Perfectly
260
Q

An 88-year-old care home resident is receiving oral clarithromycin to treat a chest infection. She develops profuse watery diarrhoea and her temperature rises to 38.2C. She has vomited three times and is experiencing diffuse abdominal
discomfort. A stool sample is positive for Clostridium difficile toxin. Which antibiotic should be given to this patient?
A Co-amoxiclav
B Penicillin
C Metronidazole
D Ciprofloxacin
E Tetracycline

A

C

How well did you know this?
1
Not at all
2
3
4
5
Perfectly
261
Q
What is another name for target cells? 
A Codocyte
B Dacrocyte 
C Spherocyte 
D Reticulocyte 
E Schistocyte
A

A

How well did you know this?
1
Not at all
2
3
4
5
Perfectly
262
Q
A 34-year-old man with Marfan’s syndrome, comes to A&amp;E having experienced a sudden tearing chest pain. He adds that the pain seems to move to his back. On examination, an early diastolic murmur is heard over the aortic valve and unequal arm pulses are palpated. What is the most likely diagnosis? 
A  Ruptured aortic aneurysm 
B  Coarctation of the aorta 
C  Aortic dissection 
D  Myocardial infarction 
E  Tension pneumothorax
A

C

How well did you know this?
1
Not at all
2
3
4
5
Perfectly
263
Q

Which set of spirometry results is most likely to be seen in a patient with COPD?
A FEV1 > 0.8 and FEV1:FVC > 0.7
B FEV1 < 0.8 and FEV1:FVC < 0.7
C FEV1 > 0.8 and FEV1:FVC < 0.7
D FEV1 < 0.8 and FEV1:FVC > 0.7
E Impossible to tell without FVC measurement

A

B

How well did you know this?
1
Not at all
2
3
4
5
Perfectly
264
Q
A 15-year-old boy arrives at A&amp;E with sudden-onset pain and swelling in his scrotum, which began 2 hours ago whilst playing a rugby match. He also starts vomiting and complains of pain in his right iliac fossa. On examination, his right hemiscrotum is red and swollen. What is the most appropriate first step in his management? 
A  Doppler ultrasound of the testes 
B  CT Scan 
C  Exploratory surgery 
D  Empirical antibiotics 
E  Abdominal X-ray
A

C

How well did you know this?
1
Not at all
2
3
4
5
Perfectly
265
Q
Which scoring system is used to determine a patient’s risk of developing 
pressure sores? 
A  GRACE score 
B  ABCD2 score 
C  Ranson score 
D  W aterlow score 
E  Rockall score
A

D

How well did you know this?
1
Not at all
2
3
4
5
Perfectly
266
Q
A 36-year-old man presents with a 3-week history of fatigue, frequent urination and excessive thirst. He also mentions that he has been unable to take part in his weekly 5-a-side football sessions for the past month because his ‘muscles feel weak’. The patient’s notes reveal that, during a previous appointment 6 months ago, his blood pressure was measured at 164/98 mm Hg. He was offered a follow-up appointment to discuss management options, however, he did not attend. The GP measures the patient’s blood pressure again, and it is 172/102 mm Hg. What would you expect to see on the ECG of this patient? 
A  Tented T waves 
B  Absent P waves 
C  ST elevation 
D  J waves 
E  U waves
A

E

How well did you know this?
1
Not at all
2
3
4
5
Perfectly
267
Q
. A 40-year-old man is brought into A&amp;E after he was found lying unconscious on the side of the road with an empty bottle of whisky next to him. Once he regains consciousness, he starts yelling at the ward staff expressing that he thinks he has been kidnapped and is being held hostage. He jumps out of bed, but finds it difficult to walk. He has a wide-based gait and he is taking small steps. Given the most likely diagnosis, what should form part of the immediate management? 
A Acamprosate
B Chlorediazepoxide 
C Thiamine
D Naloxone
E Disulfiram
A

C

How well did you know this?
1
Not at all
2
3
4
5
Perfectly
268
Q
Which of the following is not a reversible cause of cardiac arrest? 
A  Hypothermia 
B  Tension pneumothorax 
C  Cardiac tamponade 
D  Hypokalaemia 
E  Pleurisy
A

E

How well did you know this?
1
Not at all
2
3
4
5
Perfectly
269
Q
A 44-year-old woman presents with right upper quadrant pain that radiates to the tip of her right shoulder with an intermittent fever, severe chills and sweats. She has also recently experienced hiccups, breathlessness and a dry cough. On examination, there is tenderness over the 8th to 11th ribs on the right side and dullness to percussion, diminished breath sounds and reduced chest expansion over the lower zone of the right lung. 10 days prior to the onset of symptoms, she had a laparoscopic appendectomy. 
What is the most likely diagnosis? 
A  Acute cholangitis 
B  Basal pneumonia 
C  Subphrenic abscess 
D  Atelectasis 
E  Liver abscess
A

C

How well did you know this?
1
Not at all
2
3
4
5
Perfectly
270
Q
Which of the following endocrine conditions can cause hyperprolactinaemia? 
A  Cushing’s syndrome 
B  Phaeochromocytoma 
C  Addison’s disease 
D  Graves’ disease 
E  Hypothyroidism
A

E

How well did you know this?
1
Not at all
2
3
4
5
Perfectly
271
Q

An 18-year-old student is brought to see his GP by his mother. Since returning from university 2 days ago, he has become drowsy and has developed a fever. He has also vomited 3 times. On examination, the patient complains of pain when the GP flexes his hip and extends his knee, and a non-blanching rash is seen on his trunk. What should the GP do next?
A Reassure and discharge
B Administer IV or IM benzylpenicillin and call an ambulance
C Administer IV dexamethasone and call an ambulance
D Prescribe oral benzylpenicillin and arrange a follow up appointment
E Prescribe oral dexamethasone and arrange a follow up appointment

A

B

How well did you know this?
1
Not at all
2
3
4
5
Perfectly
272
Q
A 24-year-old man presents to the outpatient clinic with a 3-month history of lower abdominal pain and bloody diarrhoea. He often defecates more than 4 times per day and sometimes does not feel completely empty afterwards. On examination, his fingers are clubbed and a large irregular ulcer is found on his left shin. What is the most likely diagnosis? 
A  Irritable bowel syndrome 
B  Gastroenteritis 
C  Crohn’s disease 
D  Ulcerative colitis 
E  Coeliac disease
A

D

How well did you know this?
1
Not at all
2
3
4
5
Perfectly
273
Q
Which joint is most commonly affected in gout? 
A  1st Metacarpophalangeal joint 
B  1st Metatarsophalangel joint 
C  1st Tarsometatarsal joint 
D  1st Interphalangeal joint 
E  Talonavicular joint
A

B

How well did you know this?
1
Not at all
2
3
4
5
Perfectly
274
Q

A 10-year-old boy is brought to the respiratory clinic by his mother. Since he was very young, he has suffered from recurrent infections and has been hospitalized many times. As he has frequently moved country, a formal diagnosis has never been made. In the past 6 months, he has become increasingly breathless and has
experienced a chronic cough, productive of large volumes of purulent sputum. A chest X-ray is performed revealing widespread bronchiectasis, and situs inversus. What is the most likely diagnosis?
A Cystic fibrosis
B Young’s syndrome
C Kartagener’s syndrome
D Caplan’s syndrome
E 1 antitrypsin deficiency

A

C

How well did you know this?
1
Not at all
2
3
4
5
Perfectly
275
Q
Which system is used to stage Hodgkin’s lymphoma? 
A  Ann Arbor 
B  Gleason 
C  Dukes’ 
D  Rai and Binet 
E  Breslow
A

A

How well did you know this?
1
Not at all
2
3
4
5
Perfectly
276
Q
A 63-year-old woman presents with a moist, shallow ulcer just superior to the medial malleolus of her left foot. It is diagnosed as a venous ulcer. Which of the following features is not associated with venous ulcers? 
A  Varicose veins 
B  Calloused edges 
C  Stasis eczema 
D  Haemosiderin deposition 
E  Lipodermatosclerosis
A

B

How well did you know this?
1
Not at all
2
3
4
5
Perfectly
277
Q
Which of these conditions does not typically cause eye signs? 
A  Ulcerative colitis 
B  Crohn’s disease 
C  Ankylosing spondylitis 
D  Reactive arthritis 
E  Cervical spondylosis
A

E

How well did you know this?
1
Not at all
2
3
4
5
Perfectly
278
Q

What is the most common cause of encephalitis in the UK?

A Herpes simplex virus
B Syphilis
C EBV 
D Varicella zoster virus 
E Coxsackie virus
A

B

How well did you know this?
1
Not at all
2
3
4
5
Perfectly
279
Q
Whilst enjoying a drink in a bar with some friends, a 36-year-old woman feels a sudden sensation of tingling in her ring finger which spreads to the rest of her hand over a couple of seconds. She only feels the sensation in her right hand and she maintains awareness throughout the episode, which lasts less than a minute. What type of seizure is this describing? 
A  Absence 
B  Simple partial 
C  Complex partial 
D  Myoclonic 
E  Atonic
A

B

How well did you know this?
1
Not at all
2
3
4
5
Perfectly
280
Q
An 82-year-old lady has been in hospital for 4 weeks due to a hip fracture that she sustained after falling at home. Due to her immobility, a pressure sore has developed on the heel of her right foot. There is an intact fluid-filled blister measuring 3 inches in diameter. The ulcer is superficial and there is no subcutaneous tissue visible. According to the EPUAP, what grade of severity is this pressure ulcer? 
A Grade 1 
B Grade 2 
C Grade 3 
D Grade 4 
E Ungradable
A

A

How well did you know this?
1
Not at all
2
3
4
5
Perfectly
281
Q
A 49-year-old bird keeper has become more and more breathless over the last 6 months. She used to be able to easily complete her daily dog walk, however, recently, she has found that she is having to taking more breaks to catch her breath. She has also had a dry cough. Examination reveals fine inspiratory crackles and a chest X-ray shows reticulo-nodular shadowing. What is the most likely diagnosis? 
A  Extrinsic allergic alveolitis 
B  COPD 
C  Pneumoconiosis 
D Aspergillosis 
E Asbestosis
A

A

How well did you know this?
1
Not at all
2
3
4
5
Perfectly
282
Q
A 64-year-old man has been referred for an outpatient appointment with the urology department. Over the past 5 months, he has been urinating around 10-12 times per day. He often takes several minutes to start urinating and his stream is much weaker than it used to be. Once he has finished, he does not feel ‘completely empty’ and finds that he ‘leaks a little bit’ as well. Digital rectal examination reveals a smoothly enlarged prostate gland with a palpable midline sulcus. A diagnosis of benign prostatic hyperplasia is made. He is eager to avoid surgery if possible. Which treatment would be best for him? 
A Oxybutynin
B Solifenacin
C Tamsulosin
D Nitrofurantoin 
E Co-trimoxazole
A

C

How well did you know this?
1
Not at all
2
3
4
5
Perfectly
283
Q
A 53-year-old lady has been feeling increasingly short of breath over the past 6 months. She adds that she always feels very tired and has, more recently, experienced a tingling sensation in both hands. Neurological examination reveals a sensory neuropathy affecting only the hands and feet. What is the most likely cause of her symptoms? 
A  Iron deficiency anaemia 
B  Anaemia of chronic disease 
C  Folic acid deficiency 
D  Vitamin B12 deficiency 
E  Thalassemia trait
A

D

How well did you know this?
1
Not at all
2
3
4
5
Perfectly
284
Q
  1. A 33-year-old female with SLE presents to the fertility clinic complaining of repeated miscarriages. She has been desperately trying to start a family but has
    unfortunately suffered 3 miscarriages over the last 7 years. Her past medical history includes an appendicectomy (aged 12) and two DVTs. Given the likely diagnosis, which of the following antibodies is associated with this disease?
    A Anti-CCP antibody
    B Anti-Jo-1 antibody
    C Anti-centromere antibody
    D Anti-cardiolipin antibody
    E Anti-smooth muscle antibody
A

D

How well did you know this?
1
Not at all
2
3
4
5
Perfectly
285
Q
Which of the following is not associated with infective endocarditis? 
A  Clubbing 
B  Janeway lesions 
C  Rose spots 
D  New pansystolic murmur 
E  Splinter haemorrhages
A

C

How well did you know this?
1
Not at all
2
3
4
5
Perfectly
286
Q
A 43-year-old woman presents with a ‘rather embarrassing’ problem. Since the birth of her fourth child, 3 months ago, she has wet herself several times. She has noticed that whenever she laughs or coughs, a little bit of urine leaks out without her control. What is the name of this type of incontinence? 
A  Functional incontinence 
B  Stress incontinence 
C  Urge incontinence 
D  Overflow incontinence 
E  Double incontinence
A

B

How well did you know this?
1
Not at all
2
3
4
5
Perfectly
287
Q
A 44-year-old man has been suffering from constant, nagging headaches for the past 2 years. In this time, he has also noticed that his hands and feet appear to have grown as he has changed shoe size 3 times, and had to get his wedding ring cut off. He has been relatively healthy throughout his life except for undergoing surgery for carpal tunnel syndrome last year. What is the most likely diagnosis? 
A Cushing’s disease 
B Acromegaly
C Hypothyroidism
D Gigantism 
E Prolactinoma
A

B

How well did you know this?
1
Not at all
2
3
4
5
Perfectly
288
Q
Which of the following is used to predict the severity of acute pancreatitis? 
A  Alvarado score 
B  Rockall score 
C  Modified Glasgow score 
D  Glasgow-Blatchford score 
E  Child-Pugh score
A

C

How well did you know this?
1
Not at all
2
3
4
5
Perfectly
289
Q
A 31-year-old scuba diving instructor, living in the Maldives, had a seizure three days ago. He has no history of epilepsy, however, he has had persistent headaches over the past 5 months. He adds that the headaches are particularly bad when he goes to bed. On examination, a dark, irregular skin lesion is found on the back of his neck. An MRI scan shows multiple lesions across both cerebral hemispheres. What is the most likely diagnosis? 
A  Glioblastoma multiforme 
B  Metastases 
C  Neurofibromatosis Type 1 
D  Acoustic neuroma 
E  Meningioma
A

B

How well did you know this?
1
Not at all
2
3
4
5
Perfectly
290
Q
Which of the following examination findings is not associated with COPD? 
A  Use of accessory muscles 
B  Breathing through pursed lips 
C  Peripheral cyanosis 
D  Clubbing 
E  Bounding pulse
A

D

How well did you know this?
1
Not at all
2
3
4
5
Perfectly
291
Q
A 56-year-old man comes to A&amp;E with a very swollen glans. He went to the toilet to urinate last night, however, once he had finished, he was unable to replace his foreskin back over his glans. Since then, his glans has gradually become very painful and inflamed. What is the name of this condition? 
A  Phimosis 
B  Paraphimosis 
C  Balanitis 
D  Priapism 
E  Peyronie’s disease
A

B

How well did you know this?
1
Not at all
2
3
4
5
Perfectly
292
Q

A 59-year-old man presents to A&E with fatigue and shortness of breath, that gets worse when lying down. He has also been coughing up pink, frothy sputum. An echocardiogram is performed, which shows aortic regurgitation. On closer inspection

of the patient’s hands, his nail beds appear to be pulsating. What is the name of this sign? 
A  de Musset’s sign 
B  Quincke’s sign 
C  Traube’s sign 
D  Corrigan’s sign 
E  Becker’s sign
A

B

How well did you know this?
1
Not at all
2
3
4
5
Perfectly
293
Q

A 61-year-old man attends an outpatient clinic appointment complaining of epigastric pain that gets better soon after eating. A urease breath test confirms the presence of H. pylori. A duodenal ulcer is suspected. What is the most appropriate management option?
A 1 week of once daily omeprazole, amoxicillin and clarithromycin
B 1 week of twice daily omeprazole, amoxicillin and clarithromycin
C 1 week of once daily ranitidine, amoxicillin and clarithromycin
D 1 week of twice daily ranitidine, amoxicillin and clarithromycin
E 1 week of once daily omeprazole, ranitidine and amoxicillin

A

B

How well did you know this?
1
Not at all
2
3
4
5
Perfectly
294
Q
Which of the following is a major consequence of folate deficiency in pregnancy? 
A  Incomplete limb development 
B  Neural tube defects 
C  Congenital cardiac abnormalities 
D  High birth weight 
E  Cleft palate
A

B

How well did you know this?
1
Not at all
2
3
4
5
Perfectly
295
Q
Which of the following is true about the target population group and frequency of breast cancer screening in the UK? 
A  Women aged 35-65 every 3 years 
B  Women aged 40-60 every 3 years 
C  Women aged 40-70 every 5 years 
D  Women aged 50-75 every 5 years 
E  Women aged 50-70 every 3 years
A

E

How well did you know this?
1
Not at all
2
3
4
5
Perfectly
296
Q
A 46-year-old woman is brought to A&amp;E complaining of severe right upper quadrant pain and a high fever with rigors. On examination, she is jaundiced, febrile and tachycardic. Vital Signs: RR = 24 breaths per minute, HR = 112 bpm, Temp = 38.9°C. 
A full blood count is requested: 
Hb = 142 g/L (115-160) 
WBC = 14.7 x 109/L (4-11) 
Platelets = 370 x 109/L (150-400) 
She is diagnosed with ascending cholangitis and her disease is managed using ‘The Sepsis Six’ protocol. Which of the following is not part of ‘The Sepsis Six’? 
A  Give high-flow oxygen 
B  Take blood smear 
C  Give IV antibiotics 
D  Measure urine output 
E  Measure serum lactate
A

B

How well did you know this?
1
Not at all
2
3
4
5
Perfectly
297
Q
A 15-year-old girl is brought, by her mother, to see her GP. She is concerned that her daughter has been acting ‘very weird’ over the past few months. Having previously been quite shy and reserved, she has recently had several rude outbursts towards her parents. In addition, her performance at school has deteriorated. On examination, she appears slightly jaundiced. Closer inspection of her eyes using a slit-lamp shows dark rings around her iris. Given the most likely diagnosis, which of the following would you expect to see in the diagnostic work up? 
A  Low serum caeruloplasmin 
B  High serum copper 
C  Low AST 
D  Low ALP 
E  High transferrin saturation
A

A

How well did you know this?
1
Not at all
2
3
4
5
Perfectly
298
Q
A 4-year-old boy, who has recently moved to the UK from Cameroon, has been suffering from frequent infections and breathing difficulties since he was born. His mother tells you that he always has a cough and regularly suffers from chest infections. He has also had some bowel problems – his stools are often loose and irregular. On examination, he appears small for his age, his fingers are clubbed and bilateral coarse crackles are heard on auscultation. Cystic fibrosis is suspected. Which investigation should be requested to confirm the diagnosis? 
A Chest X-ray
B  Sweat test 
C  Faecal elastase 
D  Sputum culture 
E  Stool culture
A

B

How well did you know this?
1
Not at all
2
3
4
5
Perfectly
299
Q
Which of the following is not a risk factor for the formation of a DVT? 
A  Factor V Leiden 
B  Malignancy 
C  Nephrotic syndrome 
D  Antiphospholipid syndrome 
E  Alcohol
A

E

How well did you know this?
1
Not at all
2
3
4
5
Perfectly
300
Q
A 77-year-old retired ship-builder presents to his GP complaining of a 4-month history of right-sided chest pain, shortness of breath and 4 kg of weight loss. A chest X-ray is requested, which shows an ill-defined mass at the right pleural margin. What is the most likely diagnosis? 
A  Small cell lung cancer 
B  Adenocarcinoma 
C  Mesothelioma 
D  Squamous cell lung cancer 
E  Large cell lung cancer
A

C

How well did you know this?
1
Not at all
2
3
4
5
Perfectly
301
Q

In the treatment of acute coronary syndrome (ACS), which of the following statements is FALSE?
A Aspirin and clopidogrel do not provide enough anticoagulation; heparin should also be given
B Give 75 mg aspirin stat
3. C Give 300 mg clopidogrel in addition to aspirin
D Hypotension, asthma and bradycardia are the main contraindicators to
beta blockade
E Patients will likely continue taking a statin, beta-blocker and angiotensin-
converting enzyme (ACE) inhibitor on discharge home

A

B

How well did you know this?
1
Not at all
2
3
4
5
Perfectly
302
Q

Which of the following is not a potential cause of obstructive renal impairment?
A Benign prostatic hypertrophy B Recurrent kidney stones
C Retroperitoneal fibrosis
D Schistosomiasis
E Systemic sclerosis

A

E

How well did you know this?
1
Not at all
2
3
4
5
Perfectly
303
Q

A 60-year-old man who works for an oil company presents with a lesion on the temple that is bothering him as it is growing. It bled once when he knocked it. On examination, the lesion is 8 mm in diameter and is a flat, mildly erythematous patch with a few scales and a larger keratotic horn in the centre. There are no other lesions on inspection of his skin and no personal or family history of skin cancer.
Which of the following is the most appropriate management plan?
A Cryotherapy
B Curettage
C Excisional biopsy
D Topical 5-fluorouracil
E Wide local excision

A

A

How well did you know this?
1
Not at all
2
3
4
5
Perfectly
304
Q

A 68-year-old man who is recently diagnosed with lung cancer is admitted to the emergency department with acute shortness of breath. A chest X-ray shows a right upper zone (RUZ) collapse.
What do you expect to find on examination?
Trachea
A Deviated to right Dull RUZ Reduced breath sounds
B Deviated to right Dull RUZ Wheeze
C Deviated to right Resonant RUZ Reduced breath sounds
D Deviated to left Dull RUZ Reduced breath sounds
E Deviated to left Resonant RUZ Wheeze

A

A

How well did you know this?
1
Not at all
2
3
4
5
Perfectly
305
Q
A 68-year-old woman
disturbance. She has also noticed that she gets an itchy rash when she gets out of a hot bath. On examination she has a ruddy complexion and a palpable spleen. Her only previous medical history is gout. Initial blood tests reveal a raised packed red cell volume with a raised red cell mass, along with a raised white cell count and thrombocytosis. 
What is the most likely diagnosis? 
A Chronic myeloid leukaemia B Lymphoma
C Migraine with aura
D Polycythaemia rubra vera 
E Soap allergy
A

D

How well did you know this?
1
Not at all
2
3
4
5
Perfectly
306
Q

A 45-year-old man who is a heavy smoker is recently diagnosed with chronic obstructive pulmonary disease (COPD). He has no documented acute exacerbations in the past.
Which of the following treatment is NOT suitable in the management of COPD in this patient?
A Annual influenza and pneumococcal vaccination B Inhaled corticosteroids
C Short-acting 2-agonist
D Short-acting anti-cholinergic
E Smoking cessation

A

B

How well did you know this?
1
Not at all
2
3
4
5
Perfectly
307
Q

A 35-year-old woman is admitted to hospital with quick-onset shortness of breath. She has a past medical history of asthma. Her observations include a pulse rate 120 bpm, blood pressure 100/72 mmHg, respiratory rate 30/ min and SaO2 88% on room air. On examination, she appears to be drowsy and exhausted. Her chest is quiet on auscultation. Arterial blood gases show: pH 7.35, PaO2 5.2 kPa, PaCO2 4.9 kPa and bicarbonate 24 mmol/L.
Which of the following would NOT be appropriate in the management of this case?
A High-flow oxygen
B High-dose nebulised beta-2 agonists C Intravenous magnesium sulphate
D Leukotriene receptor antagonists
E Steroids

A

D

How well did you know this?
1
Not at all
2
3
4
5
Perfectly
308
Q

A 19-year-old footballer has collapsed on the pitch. His airway is clear and he is brought to the emergency department, where he begins to recover and denies that he has chest pain. He has never had anything like this before.
Which of the following is the most likely diagnosis?
A Carotid stenosis
B Hypertrophic obstructive cardiomyopathy (HOCM) C Myocardial infarction
D Rheumatic fever
E Thyrotoxicosis

A

B

How well did you know this?
1
Not at all
2
3
4
5
Perfectly
309
Q

A 31-year-old man presents to your clinic with a year-long history of itchy red scaly lesions.
Which of the following would make you more likely to diagnose eczema rather than psoriasis?
A Associated nail changes
B History of distal interphalangeal joint pain and swelling C Localised to flexures rather than extensors
D Well-demarcated lesions
E Worsening in winter months

A

C

How well did you know this?
1
Not at all
2
3
4
5
Perfectly
310
Q

Which of the following best describes the MRI findings in multiple sclerosis?
A Cortical grey matter inflammatory lesions
B Longitudinally extensive transverse myelitis (more than three spinal
segments)
C Periventricular white matter lesions matching the clinical picture
D Periventricular white matter lesions not necessarily matching the clinical
picture
E White matter lesions exclusively in the cerebellum and brainstem

A

D

How well did you know this?
1
Not at all
2
3
4
5
Perfectly
311
Q

A 67-year-old man is discharged from hospital following an incision and drainage of a large abdominal wall abscess. He needs someone to help change his wound packing regularly, however he is immobile and lives alone.
Which member of the multidisciplinary team would be most appropriate to help?
A District nurse
B Health visitor
C Occupational therapist D Orthotist
E Social worker

A

A

How well did you know this?
1
Not at all
2
3
4
5
Perfectly
312
Q

A 69-year-old man presents to the emergency department with ongoing chest pain. He has a past medical history of intermittent claudication and hypertension. He is an overweight smoker and heavy drinker of alcohol. On analysing the electrocardiogram (ECG), you notice broad S-waves in the right-hand chest leads, two R-waves per complex in the left-hand chest leads and ST-segment elevation. He asks if he has had a heart attack.
What is the best answer to this question?
A No – but we need to do more tests to find the true cause B No – it’s just right bundle branch block
C No – it’s just angina
D Yes
E I’m not sure; we need to do more tests

A

E

How well did you know this?
1
Not at all
2
3
4
5
Perfectly
313
Q

Which of the following is not a preventable risk factor for coronary artery disease?
A Five cigarettes per day smoking history
B High low-density lipoprotein (LDL) cholesterol levels C Hypertension
D Obesity
E 12 U/week alcohol history

A

E

How well did you know this?
1
Not at all
2
3
4
5
Perfectly
314
Q

A 29-year-old man presents to the emergency department with a 1-week history of non-productive cough, muscle aches, fever, vomiting and diarrhoea. His observations include temperature 38.4°C, pulse rate 105 bpm, blood pressure 110/76 mmHg and respiratory rate 22/min. On
examination, his chest is clear to both auscultation and percussion. A chest X-ray shows bilateral lung basal infiltrates. The blood results show Na+ 128 mmol/L, K+ 4.0 mmol/L, urea 5.9 mmol/L, creatinine 130 mol/L, albumin 26 g/L, ALT 106 IU/L, ALP 230 IU/L.
What is the most likely causative organism?
A Chlamydiapneumoniae
B Mycoplasma pneumoniae
C Legionellapneumophila
D Staphylococcus aureus
E Streptococcus pneumoniae

A

C

How well did you know this?
1
Not at all
2
3
4
5
Perfectly
315
Q

A 60-year-old man presents with a history of recurrent dizzy spells for the past 4 months, which occur daily. The dizzy spells last a few minutes and seem to occur if he moves his head, as a result of which he keeps his head as still as possible. The attacks are not associated with any deafness or tinnitus and a neurological examination is entirely normal. You favour a diagnosis of benign paroxysmal positional vertigo.
Which of the following descriptions of findings on Hallpike’s manoeuvre would confirm this diagnosis?
A Delayed onset (a few seconds) torsional nystagmus on descent facing both sides
B Delayed onset (a few seconds) torsional nystagmus on descent facing one side only
C Immediate torsional nystagmus on descent facing both sides
D Immediate torsional nystagmus on descent facing one side only
E No nystagmus on descent facing either side

A

B

How well did you know this?
1
Not at all
2
3
4
5
Perfectly
316
Q

A 43-year-old man presents to his GP with a 3-month history of recurrent nose bleeds, mucosal bleeding, haemoptysis and recurrent sinusitis. Besides that, he also noticed that he has increasingly become short of breath. On examination, he had a nasal deformity and chest auscultation revealed crackles in the left lower zone. A urine dipstick test showed microscopic haematuria.
Which of the following is the most likely diagnosis?
A Chronic myeloid leukaemia
B Chronic lymphocytic leukaemia
C Churg–Strauss syndrome
D Goodpasture syndrome
E Wegener granulomatosis

A

E

How well did you know this?
1
Not at all
2
3
4
5
Perfectly
317
Q
Following a protracted stay in hospital following a severe chest infection, an 83-year-old man develops bloody diarrhoea. 
What is the most likely cause? 
A Adenocarcinoma of the bowel 
B Clostridium difficile infection 
C Norovirus infection
D Salmonella infection 
E Shigella infection
A

B

How well did you know this?
1
Not at all
2
3
4
5
Perfectly
318
Q

A 21-year-old student on an internship with The Guardian travel section has recently returned from a backpacking holiday in West Africa. For the last few days he has been having headaches, flu-like symptoms and muscle aches, and now he has started rigoring.
Which investigation should be performed to rule out malaria?
A Blood cultures
B Falciparum antigen dipstick test
C Liver biopsy
D One blood film
E Three thick and thin blood films on consecutive days

A

E

How well did you know this?
1
Not at all
2
3
4
5
Perfectly
319
Q

You are asked to review an electrocardiogram (ECG) in the emergency department. Helpfully, a summary of details is printed at the top as follows: rate 88/min, regular rhythm, axis –20°, PR duration 0.26 seconds (constant), QRS complex 0.08 seconds, QT interval 0.2 seconds. You note that P-waves are only present before each QRS and that the rhythm is regular.
Which of the following would be the best summary?
A First-degree heart block
B Left axis deviation
C Left bundle branch block
D Refuse to summarise until it can be compared with an old ECG
E Ventricular tachycardia

A

A

How well did you know this?
1
Not at all
2
3
4
5
Perfectly
320
Q

A 65-year-old man with a longstanding diagnosis of chronic obstructive pulmonary disease has been reviewed by his GP for deteriorating liver function tests and clinical signs and symptoms of cirrhosis.
What investigation should the GP arrange?
A Alpha-1-antitrypsin serum levels
B Alpha-feto protein levels
C Anti-smooth muscle antibodies
D Gamma GT levels
E Hepatitis screen

A

A

How well did you know this?
1
Not at all
2
3
4
5
Perfectly
321
Q

A 44-year-old woman presents to the emergency department with pain. The pain is epigastric, sharp in nature, worse on lying flat and during inspiration. She has recently suffered a chest infection. She is not a smoker. On examination, she has diffuse inspiratory crepitations. Her oxygen saturation is 98% on room air. Her ECG shows widespread saddle-shaped ST elevation.
Which of the following is the most likely diagnosis?
A Acute pericarditis
B Angina
C Myocardial infarction D Pleurisy
E Pulmonary embolism

A

A

How well did you know this?
1
Not at all
2
3
4
5
Perfectly
322
Q

A 55-year-old overweight pub landlord presents with a several-year history of episodic acute painful joint swelling that started in his left big toe and now affects his knees. Symptoms improve with use of diclofenac. Gout was diagnosed on his first hospital visit, however this now appears recurrent. He developed an acute attack in his left knee 2 days ago.
Which of the following represents the best plan for prophylaxis?
A Keep on long-term diclofenac with gastric protection
B Start allopurinol now with non-steroidal anti-inflammatory drugs
(NSAIDs) cover and increase until his urate is below 300 mol/L
C Start allopurinol at least 2 weeks after the acute attack has settled with
NSAID cover and increase until his urate level is below 300 mol/L
D Switch to long-term colchicine
E Switch to use of depot steroid injections

A

C

How well did you know this?
1
Not at all
2
3
4
5
Perfectly
323
Q

Which of the following conditions does not classically cause hepatomegaly?
A End-stage cirrhosis
B Fatty liver
C Hepatocellular carcinoma D Myeloproliferative disease E Right-sided heart failure

A

A

How well did you know this?
1
Not at all
2
3
4
5
Perfectly
324
Q

Which of the following conditions is associated with sclerosing cholangitis?
A Autoimmune hepatitis
B Coeliac disease
C Irritable bowel syndrome D Pernicious anaemia
E Ulcerative colitis (UC)

A

E

How well did you know this?
1
Not at all
2
3
4
5
Perfectly
325
Q

A 35-year-old homeless man presents to the emergency department in a state of unconsciousness. He was fitting when the ambulance crew got to him 20 minutes ago, and a friend at the scene estimated that he had started fitting “around 15 minutes before”. His friend informed the ambulance crew that he is a known epileptic and you find a pack of phenytoin on him. He looks dishevelled and smells of alcohol. He has a blood pressure of 170/95 mmHg and temperature 37.9°C. On examination there is a quiet systolic murmur, though it is difficult to fully characterise.
Which of the following investigations will be most useful at this stage?
A Computed tomography (CT) of the brain
B Echocardiogram
C Electroencephalogram (EEG)
D Magnetic resonance imaging (MRI) of the brain
E Phenytoin levels

A

E

How well did you know this?
1
Not at all
2
3
4
5
Perfectly
326
Q

A 74-year-old male ex-comedian can no longer perform at smoky open- microphone nights due to shortness of breath. He is coughing up frothy white sputum, which recently has contained a small amount of blood. On examination, his chest demonstrates diffuse crackles on inspiration through which you can just discern a mid-diastolic murmur and a loud first heart sound. His chest X-ray confirms pulmonary oedema.
What is the most likely underlying cause for his symptoms?
A Lower respiratory tract infection
B Mitral stenosis
C Non-small cell carcinoma of the lung D Pulmonary embolism
E Small cell carcinoma of the lung

A

B

How well did you know this?
1
Not at all
2
3
4
5
Perfectly
327
Q

In clinic, a retired 62-year-old man presents with shortness of breath on exertion. You find a collapsing pulse and subsequent echocardiography confirms aortic regurgitation.
Which of the following is NOT associated with aortic regurgitation?
A Ankylosing spondylosis B Aortic dissection
C Marfan syndrome
D Rheumatic fever
E Systemic lupus erythematosus (SLE)

A

E

How well did you know this?
1
Not at all
2
3
4
5
Perfectly
328
Q

A previously fit and well 70-year-old woman has been admitted due to a fractured neck of femur, and she has recently returned to the ward after a cemented hemiarthroplasty. You are bleeped to the ward to see her, as it is noted that she has had only 30 ml urine output in the last 3 hours. She is asleep on the ward, with a patient-controlled analgesic device in situ. Her airway is intact and her respiratory rate is 12/min with normal saturations and good air entry bilaterally. Her pulse is 125 bpm with a blood pressure of 95/68 mmHg and she has delayed capillary refill. She has pale conjunctiva and a temperature of 37.3°C. She has a 12-hourly bag of normal saline running. Her catheter is draining concentrated urine. An abdominal examination is normal.
Which of the following interventions would you try first to increase the urine output?
A Fluid challenge of 500 ml 5% dextrose over 10 minutes
B Fluid challenge of 500 ml normal saline over 10 minutes
C Flush the catheter
D Start antibiotics for presumed sepsis
E Stop the patient-controlled analgesic device

A

B

How well did you know this?
1
Not at all
2
3
4
5
Perfectly
329
Q
A 23-year-old woman comes to see you about her stools, which over the last couple of months have become extremely foul smelling, pale in colour and difficult to flush. This has been associated with vague abdominal pains and a bloating sensation. She has found this very embarrassing as she lives in a shared house. She is normally fit and well. 
What is the most likely diagnosis? 
A Chronic pancreatitis
B Coeliac disease
C Common bile duct obstruction 
D Cystic fibrosis
E Giardia infection
A

B

How well did you know this?
1
Not at all
2
3
4
5
Perfectly
330
Q

An 83-year-old man who was diagnosed as having Parkinson’s disease 3 years ago has been treated with levodopa (L-DOPA). Whilst he initially responded well to therapy, he has started to be increasingly still, and has fallen more in the last 4 months despite no intercurrent illness or change in L-DOPA therapy.
Which is the best management option?
A Add a dopamine agonist (e.g. ropinerole)
B Add a peripheral dopamine antagonist (e.g. domperidone) C Decrease L-DOPA therapy
D Increase L-DOPA therapy
E Stop L-DOPA therapy

A

A

How well did you know this?
1
Not at all
2
3
4
5
Perfectly
331
Q
Which antibody can you expect to see in primary biliary cirrhosis (PBC)? 
A ANA
B ANCA
C Anti-mitochondrial antibody 
D Anti-phospholipid antibodies 
E Anti-smooth muscle antibodies
A

C

How well did you know this?
1
Not at all
2
3
4
5
Perfectly
332
Q

The left anterior descending coronary artery usually supplies:
A The anterior wall of the left ventricle and the atrio-ventricular node
B The anterior wall of the left ventricle and the inter-ventricular septum
C The anterior wall of the left ventricle, atrio-ventricular node and the
inter-ventricular septum
D The inter-ventricular septum and the inferior part of the left ventricle
E The sino-atrial node, the atrio-ventricular node and the inferior part of
the left ventricle

A

B

How well did you know this?
1
Not at all
2
3
4
5
Perfectly
333
Q

A 42-year-old woman with menorrhagia is complaining of tiredness. The GP does some blood tests, which reveal hypochromic microcytic anaemia, a decreased ferritin level and a raised total iron binding capacity. Platelets were slightly raised.
Which of the following is the best treatment for this anaemia?
A Erythropoietin
B Iron chelators
C Iron supplementation
D Regular transfusion
E Regular venesection

A

C

How well did you know this?
1
Not at all
2
3
4
5
Perfectly
334
Q

In which of the following circumstances should angiotensin-converting enzyme (ACE) inhibitors be avoided where possible?
A Glomerulonephritis
B Lupus nephritis
C Renal artery stenosis
D Systemic sclerosis with renal involvement
E All of the above

A

C

How well did you know this?
1
Not at all
2
3
4
5
Perfectly
335
Q

Which of the following correctly describes Staphylococcus aureus?
A Anaerobic rod
B Gram-negative coccus C Gram-negative rod
D Gram-positive coccus
E Gram-positive rod

A

D

How well did you know this?
1
Not at all
2
3
4
5
Perfectly
336
Q

A 78-year-old retired groundskeeper presents with a 2 cm skin lump on his temple. He is unsure how long it has been there. It appears to have a rolled, shiny edge with telangiectasia and a central ulcerated area.
Which of the following is the most likely diagnosis?
A Actinic keratosis
B Basal cell carcinoma
C Keratoacanthoma
D Malignant melanoma
E Squamous cell carcinoma

A

B

How well did you know this?
1
Not at all
2
3
4
5
Perfectly
337
Q
A 54-year-old woman presents to the emergency department with a 2-month history of intermittent right upper quadrant pain. The pain is sharp in nature and radiates round to the back. On examination there is no jaundice, no hepatomegaly and she is apyrexial. Liver function tests and an amylase are normal. She has no history of recent foreign travel. 
What is the most likely diagnosis? 
A Biliary colic 
B Cholangitis 
C Hepatitis A 
D Hepatitis C 
E Pancreatitis
A

A

How well did you know this?
1
Not at all
2
3
4
5
Perfectly
338
Q

A 23-year-old man presents with a several-months history of lower back pain and stiffness.
Which of the following symptoms would make you think of ankylosing spondylitis (AS) as the diagnosis?
A Asymmetrical tenderness on palpation over the lumbosacral spine
B HLA-DR4 genotype
C Pain present on waking in the early morning
D Scoliosis present on examination
E Worse after heavy lifting

A

C

How well did you know this?
1
Not at all
2
3
4
5
Perfectly
339
Q

A 75-year-old woman in the pre-assessment clinic tells you she has mitral stenosis.
Which of the following is not a sign of mitral stenosis?
A Bifid P-wave
B Diastolic opening snap heart sound C Double impulse apex beat
D Mid-diastolic murmur
E Peripheral cyanosis

A

C

How well did you know this?
1
Not at all
2
3
4
5
Perfectly
340
Q

A 35-year-old man presents with a 2-day history of right-sided facial weakness.Heisotherwisefitandwell.Thereisnopasthistoryofneurological symptoms. There is no history of preceding infection. On examination, the middle ear is normal, the salivary glands are not enlarged, and there are no other cranial nerves affected. The forehead is not spared. Neurological examination of the limbs is unremarkable. Routine investigations are all normal.
Which of the following represents the most reasonable management plan?
A Aspirin, dipyridamole, a statin and an angiotensin-converting enzyme (ACE) inhibitor
B Penicillin-based antibiotic therapy and antiviral therapy
C Steroids
D Steroids and penicillin-based antibiotic therapy
E Steroids, antiviral therapy and eye protection

A

E

How well did you know this?
1
Not at all
2
3
4
5
Perfectly
341
Q
A 51-year-old man is found to have a pathological fracture of his femur. Investigations reveal immunoglobulin light chains in the urine. 
What is his diagnosis? 
A Benign monoclonal gammopathy 
B Bone metastases
C Multiple myeloma
D Osteoporosis 
E Vitamin D deficiency
A

C

342
Q

A 35-year-old man is diagnosed with human immunodeficiency virus (HIV) infection.
Which of the following indicates that highly active antiretroviral therapy (HAART) should be commenced?
A Fever and weight loss >10 kg
B Viral load >50 copies/ml
C Viral load >100 copies/ml
D CD4 count <200 cells/mm3
E CD4 count <500 cells/mm3

A

D

343
Q

An anxious mum has read on the internet about tetralogy of Fallot as she is convinced her little boy may have it.
Which of the following does not fit the diagnosis?
A Her child is small for his age
B Her child is cyanotic
C His pulse exhibits a radio-femoral delay
D Her child exhibits a loud systolic murmur
E Her child can relive symptoms just by squatting

A

C

344
Q

A 45-year-old man presents with intermittent difficulty in swallowing for the last 4 months. This is associated with severe retrosternal pain and regurgitation. He has no risk factors or sinister signs for malignancy.
What is the most important investigation in this case?
A Barium swallow
B Chest X-ray
C CT of the chest
D Endoscopy
E Iron studies

A

D

345
Q
A 55-year-old Asian man with known thalassaemia trait registers with a new GP and is found to have a mild microcytic anaemia on routine testing. He does not complain of any symptoms. 
What is the most appropriate treatment? 
A Blood transfusion
B Folate supplementation 
C Iron chelators
D Iron supplementation 
E No treatment required
A

E

346
Q

A 72-year-old man is on warfarin for atrial fibrillation. Following a recent chest infection his international normalised ratio (INR) rockets up to 5.2.
What was the most likely cause for this?
A Codeine phosphate
B Erythromycin
C Inappropriate high doses of warfarin
D International normalised ratio (INR) increased in concomitant infection
E Steroid inhalers

A

B

347
Q

A 40-year-old man, previously fit and well, limps in to the emergency department with an acutely red, hot, swollen, exquisitely tender knee, which he holds rigid. He is tachycardic and has a temperature of 38.3°C.
Which of the following represents the best approach to diagnosis and management?
A Aspirate a small amount of joint fluid and send it for microscopy under polarised light
B Aspirate a small amount of joint fluid, send the fluid for urgent Gram stain and culture, take blood cultures, and start antibiotics only when you know the sensitivities of any bacteria present
C Aspirate the joint fully, send the fluid for urgent Gram stain and culture, take blood cultures, and start antibiotics only if bacteria are detected on Gram stain of either fluid
D Aspirate the joint fully, send the fluid for urgent Gram stain and culture, take blood cultures, and start antibiotics only when you know sensitivities of any bacteria present
E Aspirate the joint fully, send the fluid for urgent Gram stain and culture, take blood cultures, and start empirical intravenous antibiotics immediately

A

E

348
Q

A 29-year-old man is brought to the emergency department having been found unresponsive on a park bench. On examination, his airway is patent and he has a spontaneous respiratory rate of 7, with a saturation rate of 92% on air. There is no abnormality on examination or auscultation of the chest. He has a pulse of 70 bpm and a blood pressure of 110/80 mmHg. The ECG is normal. He has a Glasgow Coma Score (GCS) of 3 and has pinpoint pupils. He has a temperature of 36.8°C and a blood sugar reading of 6. Basic initial management steps include high-flow oxygen administration and intravenous access.
Which of the following might you also implement?
A 500 ml stat intravenous fluid challenge
B 50 ml of 50% glucose intravenously stat
C Bair hugger
D Flumazenil
E Naloxone

A

E

349
Q
A first-time mother comes to visit you with her 10-month-old son. At least once every day her son vomits up his entire feed. The vomiting is not projectile but rather the feed returns to the mouth and spills over his top. She stopped breast-feeding him when he was 6 months old. He is otherwise well in himself, with a normal weight for his age. 
What is the most likely diagnosis? 
A Gastro-oesophageal reflux disease 
B Lactose intolerance
C Physiological posseting
D Pyloric stenosis 
E Viral gastroenteritis
A

C

350
Q
A 13-year-old girl, who is quiet and withdrawn, comes to see you with her mother. She has a 4-month history of weight loss and secondary amenorrhoea. She has no bowel symptoms. Her body mass index is 16. Apart from being very thin, the examination is otherwise normal. All blood results, including hormone assays, are normal. 
What is the most likely diagnosis? 
A Anorexia nervosa
B Coeliac disease
C Crohn’s disease
D Epstein–Barr virus infection 
E Irritable bowel syndrome
A

A

351
Q

Which of the following routine blood tests is most likely to indicate a cause of an elderly patient’s acute confusional state?
A Calcium
B C-reactive protein (CRP) C Liver function tests
D Sodium
E Urea

A

B

352
Q
Which of the following conditions is not associated with HLA-B27? 
A Crohn’s disease 
B Psoriasis
C Scleritis
D Ulcerative colitis 
E Uveitis
A

C

353
Q

A 69-year-old man presents with a range of signs and symptoms that give the impression of heart failure.
Which of the following is not a feature of heart failure?
A Hepatomegaly
B Non-pitting oedema
C Pulsus alternans
D Raised jugular venous pressure
E Tricuspid regurgitation

A

B

354
Q

A 57-year-old man with a history of alcohol abuse presents with a 12-hour history of severe central epigastric pain radiating through to the back. Blood tests reveal a normal white cell count, amylase 2250 U/ml, LDH 530 IU/L, calcium 2.2 mmol/L, albumin 38 g/dl, urea 12 mmol/L and blood glucose 14 mmol/L. An arterial blood gas reading demonstrates a pO2 of 10.2 kPa on room air.

What is his Glasgow score? 
A0 
B1
 C2 
D3 
E4
A

C

355
Q

A 60-year-old man presents with total visual loss in his left eye that developed over 1 day. He says he had a headache on the left side in the 5 days that preceded the visual loss, which was constant and severe, and worse on talking or eating. On further questioning he said he has been feeling lethargic for the last 2 months and has been having some shoulder aches. There is no relevant past medical history and he has never smoked. On examination, there is no vision in the left eye, there is a relative afferent papillary defect, and the optic disc appears red and swollen on ophthalmoscopy. There is no other cranial nerve defect or neurological defect.
Which investigation would confirm the most likely diagnosis?
A Computed tomography (CT)
B Inflammatory markers (C-reactive protein (CRP), erythrocyte
sedimentation rate (ESR))
C Lumbar puncture
D Temporal artery biopsy
E Visual evoked potentials

A

D

356
Q

A 70-year-old man presents to the emergency department with a cough productive of blood-stained green sputum and shortness of breath. A chest X-ray demonstrates a suspicious lesion in the right lower zone associated with consolidation. On further questioning, he admits to recent weight loss and back pain. Blood tests show:
Na+ 137 mmol/L K+ 3.8 mmol/L Urea 14.5 mmol/L Creatinine 160 μmol/L
Corrected Ca2+ PO4
Alk Phos
3.0 mmol/L 0.33 mmol/L 450 mmol/L
Which of the following is NOT appropriate in the management of this patient?
A Check patient’s urea and electrolyte level and serum calcium level twice daily
B Consider starting intravenous bisphosphonate
C Consider starting loop diuretics
D Rehydration with intravenous normal saline
E Request an urgent isotope bone scan

A

E

357
Q

A 45-year-old woman with type 1 diabetes and rheumatoid arthritis has been admitted after a massive overdose of ibuprofen and has been noted to have acute renal failure with a creatinine level of around 350 μmol/L. The next day, you are called to see her as she is becoming breathless. Her airway is clear. She looks anxious. Her respiratory rate is 28/min and her saturations are 86% on air. You hear bibasal crepitations in her chest, which is slightly dull to percussion at both bases. She has a pulse of 110 bpm and blood pressure of 125/80 mmHg. Her urine output has been 120 ml in the last 10 hours, and you note that her jugular venous pressure is high. It appears from the notes that the night house officer was asked to see her due to her low urine output and prescribed aggressive fluid resuscitation.
What is the most likely reason for her breathlessness?
A Acute respiratory distress syndrome
B Aspiration
C Pleural effusion
D Pneumonia
E Pulmonary oedema

A

E

358
Q
A 55-year-old man is referred to haematology with repeated nosebleeds, malaise, weight loss and night sweats. Investigations show a raised white cell count, mainly neutrophils and myelocytes, anaemia, increased urate and increased alkaline phosphatase. On blood film there are no blast cells. Genetic studies show a t(9:22) translocation encoding for the BCR-ABL gene. 
What is his diagnosis? 
A Acute lymphoblastic leukaemia
B Acute myeloid leukaemia
C Chronic lymphoblastic leukaemia 
D Chronic myeloid leukaemia
E Non-Hodgkin lymphoma
A

D

359
Q

A 48-year-old man is admitted to the emergency department vomiting blood. He has a blood pressure of 80/45 mmHg with a heart rate of 135 bpm. He is cool and clammy to the touch.
Which of the following should NOT be in your immediate management of this patient?
A Alert the endoscopy suite
B Contact senior support
C Cross match 6–8 U blood urgently
D Insert two large-bore cannulae
E Transfer to the ward

A

E

360
Q

A 5-month-old girl presents with failure to thrive, pallor and mild jaundice. The child has frontal bossing of the skull, prominent maxillae and hepatosplenomegaly.
Which of the following is the most likely diagnosis?
A Congenital biliary atresia
B Congenital hypothyroid
C Hydrops fetalis
D Lead poisoning
E Thalassaemia major

A

E

361
Q

Which of the following findings on hand examination are suggestive of osteoarthritis?
A Dactylitis
B Osteophytes at the distal and/or proximal interphalangeal joints
C Prominent nail fold capillary loops
D Prominent ulnar styloid
E Trigger finger

A

B

362
Q

Several trials have been published regarding the role of therapeutic statin use in bowel cancer. The published results are disparate and you wish to find the best answer from the available literature.
Which of the following study designs would you use?
A Case series
B Delphi method
C Meta-analysis
D Non-systematic review
E Randomised, controlled trial

A

C

363
Q

A 52-year-old man presents to his GP with chest pain and is afraid he is having a heart attack. His most significant symptom toward a diagnosis of acute coronary syndrome (ACS) is:
A Chest pain located just under the left nipple, near the apex beat
B A past medical history of controlled angina
C A sharp chest pain
D Shortness of breath
E Tachycardia

A

B

364
Q
A 29-year-old woman is concerned about recurrent ulcers she gets in her mouth. Occurring every few months, they are small, grey, shallow ulcers that disappear spontaneously. She has no other gastrointestinal symptoms or weight loss. 
What is the most likely cause? 
A Behçet disease
B Crohn’s disease
C Herpes simplex
D Idiopathic aphthous ulcers 
E Oral carcinoma
A

D

365
Q

From the following list of antibiotics and their predominant cover, which is incorrect?
A Co-amoxiclav – broad-spectrum Gram-positive and -negative cover B Flucloxacillin – Gram-positive cover
C Gentamicin – Gram-negative cover
D Metronidazole – anaerobic cover
E Vancomycin – Gram-negative cover

A

E

366
Q

A 62-year-old Asian woman presents to the emergency department with increasing shortness of breath for the past 2 weeks. She has a past medical history of breast cancer, ischaemic heart disease, previous myocardial infarction with a stent in her left anterior descending artery, and osteoarthritis. On examination, there are reduced breath sounds on her lung bases, and dullness to percussion. Both legs are swollen. A chest X-ray demonstrates a right-sided pleural effusion. A simple aspiration of the pleural cavity showed a pleural fluid protein:serum protein ratio <0.5.
What is the most likely underlying diagnosis?
A Congestive cardiac failure
B Metastatic lung cancer
C Rheumatoid disease
D Systemic lupus erythematosus
E Tuberculosis

A

A

367
Q

A 25-year-old man presents with a severe outbreak of a dry erythematous itchy rash that is now widespread, despite having used a steroid cream prescribed by his GP. It appears to be eczema, and along with regular emollients for the dryness, and antihistamines for the itch, you would like to prescribe some very potent topical steroids for a brief period to attempt get on top of the outbreak.
Which of the following is classed as a very potent topical steroid?
A Betnovate
B Dermovate
C Eumovate
D Hydrocortisone
E Prednisolone

A

B

368
Q

A 7-year-old boy presents with multiple erythematous patches, over both sides of his face, head, neck, upper chest and left arm and shoulder, which appear to be covered in a honey-coloured crust. His mother says that the lesions have spread, over about a week, starting at the left neck and radiating outwards. The child appears to be upset and the lesions are itchy. There is no past medical history and no history of recent infections.
Which of the following diagnoses is most likely?
A Eczema
B Erysipelas
C Impetigo
D Psoriasis
E Staphylococcal scalded skin syndrome

A

C

369
Q

A 75-year-old man has been recently diagnosed with Parkinson’s disease and started on levodopa (L-DOPA), which has helped his tremor and bradykinesia significantly. However, in the past week he had a fall shortly after getting out of his chair, during which he briefly blacked out, and afterwards was sweating and felt cold.
Which of the following side effects most likely explains the fall?
A Dyskinesia
B Hallucinations
C Nausea
D None, he actually has multiple system atrophy and not Parkinson’s disease E Postural hypotension

A

E

370
Q

You are reviewing the medication of a 62-year-old man suffering from depression and cardiac arrhythmias. He was recently discharged from hospital for chest pain. During his admission, he was noted to have “torsades de pointes”.
What is torsades de pointes?
A A clinical sign associated with acute pericarditis
B A congenital facial sign associated with congenital heart defects
C A form of ventricular tachycardia that can self-correct but can also lead
to sudden death
D A variation in the QRS that changes in magnitude with inspiration and
expiration
E Alternate T-wave inversion

A

C

371
Q

A 69-year-old man recently diagnosed with metastatic prostate cancer presents with weakness in his legs and urinary retention. He has had back pain for years but in the last 24 hours this has become very severe in his lower back. On examination he has a sensory deficit, loss of anal tone and poor sensation in the skin around the anus. When catheterised he has a residual volume of 1.5 L.
Which of the following is the most informative initial investigation?
A Computed tomography (CT) of the abdomen/pelvis
B Lumbar X-rays
C Magnetic resonance imaging (MRI) of the lumbar spine
D Serum calcium
E Ultrasound scan (USS) of the renal tract

A

C

372
Q
A 6-year-old boy has a 1-month history of aching bones and muscle pains. On examination he appears pale and has multiple bruises along his legs. He has extensive lymphadenopathy. Blast cells are seen on the blood film. 
What is the most likely diagnosis? 
A Acute lymphoblastic leukaemia 
B Child abuse
C Henoch–Schönlein purpura
D Juvenile idiopathic arthritis 
E Sickle cell disease
A

A

373
Q

A 76-year-old man presents with a vesicular eruption on the left side of his forehead only. It is severely painful and the vesicles have started to crust over. On examination, the area affected is well-demarcated. You also note a red eye with apparent conjunctivitis.
Given the most likely diagnosis, which of the following treatments is the most appropriate?
A Intravenous aciclovir
B Oral aciclovir
C Topical aciclovir
D Topical antibiotic
E Topical steroids

A

B

374
Q

You find a loud systolic murmur in a 14-year-old girl during a preoperative assessment for an appendicectomy.
Which of the following signs or investigations do NOT help confirm a diagnosis of coarctation of the aorta?
A Ankle brachial pressure index
B Electrocardiogram
C Magnetic resonance angiography
D Radial pulse asymmetry
E Radio-femoral delay

A

B

375
Q

A 28-year-old woman with a past medical history of asthma has been using her inhaled salbutamol more frequently of late. She is currently on an inhaled short-acting beta-2 agonist and an inhaled steroid 800 μg/day. She has been compliant.
What should you do?
A Add inhaled long-acting beta-2 agonist
B Add leukotriene receptor antagonist
C Add oral steroid
D Increase dose of inhaled steroid
E Prescribe inhaled short-acting beta-2 agonist as regular therapy

A

A

376
Q
A 62-year-old woman presents to the emergency department with collapse. She felt dizzy when she tried to stand up from a sitting position. She did not lose consciousness. She denied any visual disturbances, headache or head injury. She also complained of fatigue over the past month. She takes only omeprazole and paracetamol. An electrocardiogram (ECG) showed normal sinus rhythm. Her blood pressure was 102/50 mmHg. A blood test revealed the following: Na+ 126 mmol/L, K+ 6.5 mmol/L, urea 10.0 mmol/L, and creatinine 139 μmol/L. 
What is the most likely diagnosis? 
A Acromegaly
B Adrenal insufficiency 
C Cushing’s disease
D Hypocalcaemia
E Hypothyroidism
A

B

377
Q

A 56-year-old man with a long history of alcohol abuse presents to the emergency department with abdominal pain. On examination he has a distended abdomen with shifting dullness and has a temperature of 38.2°C.
What is the most likely diagnosis?
A Bowel obstruction
B Liver cirrhosis
C Mallory–Weiss syndrome
D Perforated peptic ulcer
E Spontaneous bacterial peritonitis (SBP)

A

E

378
Q

A 52-year-old man with hyperthyroidism, vitiligo and a 30 pack/year smoking history, presents to hospital with an acute clumsiness of his right hand. Neurological examination reveals normal cranial nerves, and the only abnormal feature on the limb examination is some past pointing and dysdiadochokinesis in the right hand. Diffusion-weighted magnetic resonance imaging (MRI) reveals a small right-sided cerebellar infarct.
Which of the following investigations is unlikely to be helpful?
A Carotid Doppler
B Electrocardiogram (ECG)
C Erythrocyte sedimentation rate (ESR)
D Full blood count
E Magnetic resonance angiography

A

A

379
Q
A 32-year-old woman presents to the GP with a 3-month history of non- productive cough and breathlessness on exertion. She also complains of fatigue, weight loss and joint pain. She smokes 15 cigarettes/day. She has not travelled to any foreign countries recently. On examination, fine interstitial crackles are heard on the anterior chest wall. There are multiple tender red lumps on both shins. 
What is the most likely diagnosis? 
A Idiopathic pulmonary fibrosis 
B Lung cancer
C Sarcoidosis
D Systemic lupus erythematosus 
E Wegener granulomatosis
A

C

380
Q

Which of the following findings on lumbar puncture is most commonly used to suggest a diagnosis of multiple sclerosis (given a correlative clinical picture)?
A Autoantibodies to myelin basic protein
B Oligoclonal bands on electrophoresis
C Presence of red blood cells
D Raised lymphocytes
E Raised protein

A

B

381
Q

A 59-year-old woman with known polycythaemia vera presents to the emergency department with right upper quadrant pain, tender hepatomegaly and gross ascites, which has come on suddenly. There is no jaundice.
What is the next most appropriate investigation?
A Cytomegalovirus (CMV) screen
B Gamma-glutamyl transferase (GGT) levels
C Hepatitis serology
D Hepatic vein Doppler ultrasound scan (USS)
E Human immunodeficiency virus (HIV) testing

A

D

382
Q
A 60-year-old Chinese man presents to the GP with a 4-month history of weight loss. He has lost around 1 stone in weight. He also has a cough productive of green sputum without blood. He has a reduced appetite, insomnia and night sweats. His past medical history includes hypertension and ischaemic heart disease. He is a retired managing director who has just emigrated from Hong Kong to England. He denies a smoking history. His observations include temperature 36.4°C, pulse 67 bpm, blood pressure 152/92 mmHg and respiratory rate 15/min. On examination, there is dullness to percussion over the left upper lung zone. 
What is the most likely diagnosis? 
A Lung cancer
B Lung abscess
C Pulmonary infarction
D Tuberculosis
E Wegener granulomatosis
A

D

383
Q

A 27-year-old man presents with a 3-month history of cough with some blood streaking, loss of weight and night sweats. You suspect tuberculosis.
Which of the following chest X-ray findings is not consistent with tuberculosis?
A Cavitating lesion
B Consolidation of a lobe
C Diffuse 1–2 mm spots of increased opacity
D Perihilar ground-glass changes
E Pleural effusion

A

D

384
Q

A 74-year-old man is admitted to hospital with an acute-on-chronically ischaemic leg and started on intravenous heparin. Blood tests taken 5 days
9 later show platelet levels have dropped from 250 to 54 × 10 .
What is the most likely cause for thrombocytopenia in this patient?
A Chronic lymphocytic leukaemia
B Disseminated intravascular coagulation
C Evans syndrome
D Heparin-induced thrombocytopenia (HIT)
E Thrombotic thrombocytopenic purpura

A

D

385
Q

A 40-year-old man presents to the emergency department after 8 hours of a severe frontal headache, photophobia and two episodes of vomiting. On examination his Glasgow Coma Score (GCS) is 13 (eyes open to voice, verbal response with confused sentences, and moving freely), he has a temperature of 38.5°C, a BP of 95/65 mmHg and pulse rate of 105 bpm. He is photophobic and has neck stiffness. There is no rash and no focal neurological deficit as far as you can ascertain. His wife is present and is able to elaborate on the history. He has hit the bottle quite hard since losing his job 7 months ago and has been drinking 8–10 cans of lager per night. He has recently been coughing a lot and feeling unwell. He still lives at home and has not appeared to lose weight recently.
What are you most concerned that this might be?
A Listeria meningitis
B Meningococcal meningitis C Pneumococcal meningitis D Tubercular meningitis
E Viral meningitis

A

C

386
Q

A 63-year-old male smoker, on diuretics for essential hypertension, presents to the emergency department with chest pain. His ECG is presented to you. Amongst other signs, you notice T-wave inversion.
Which of the following does not cause T-wave inversion?
A Hyperkalaemia
B Left bundle branch block
C Left ventricular hypertrophy
D Myocardial infarction
E Myocardial ischaemia

A

A

387
Q
A 72-year-old man is bed-bound following a severe stroke several years previously and lives in a nursing home with full-time care. The staff noticed that over the last 8 days he didn’t pass any stool motions until this morning, when there was profuse, offensive liquid stool. They are concerned this may be infective diarrhoea. On examination there is hard stool in the rectum. 
What is the most likely diagnosis? 
A Campylobacter infection
B Carcinoma of the rectum
C Inflammatory bowel disease 
D Norovirus
E Overflow diarrhoea
A

E

388
Q

A 24-year-old man presents with two episodes of involuntary clonic beating of the right arm followed by loss of consciousness with urinary and faecal incontinence. He then had a headache and confusion for some hours (no- one witnessed any episode). A diagnosis of motor partial seizures with secondary generalisation is made. He is started on valproate.
Which of the following would you NOT tell him (as it is untrue)?
A Computed tomography (CT)/magnetic resonance imaging (MRI) is indicated to look for a cause
B No driving for 1 year after the last seizure
C On valproate, blood tests will need to be monitored for liver enzymes and
full blood count
D Valproate may cause gum hypertrophy
E Valproate may cause hair loss

A

D

389
Q

A 42-year-old woman with known HIV presents with a 3-week history of dry cough and increasing breathlessness. She is now unable to walk 100 yards without becoming short of breath. You suspect pneumocystis pneumonia.
Which of the following tests is the best way to confirm the diagnosis?
A Bronchoalveolar lavage
B Chest X-ray
C Computed tomography (CT) of the chest
D Cytology of sputum induced by nebulised hypertonic saline
E Hypoxaemia on arterial blood gas

A

A

390
Q

A 65-year-old woman presents to her GP complaining of tiredness, shortness of breath and loss of sensation in her feet. On examination she is visibly pale, has mild jaundice and glossitis. She has loss of joint position sense and vibration distally, with extensor plantar reflexes, absent ankle jerks and brisk knee jerks bilaterally.
What is the most likely cause for her symptoms?
A Chronic sickle cell disease
B G6PD deficiency
C Heart failure
D Renal failure
E Vitamin B12 deficiency

A

E

391
Q

A 17-year-old girl shuffles awkwardly into clinic with her mother, who explains that her daughter has suffered from severe acne for the last few years and nothing that the GP has tried has shifted it. On examination, as well as several large pustules on her face and comedones, some scarring is beginning to develop. You would like to start oral isotretinoin (roaccutane).
When counselling about side effects, which of the following would you NOT advise the patient?
A Blood tests should be taken to monitor for hyperlipidaemia
B Blood tests should be taken to monitor for raised liver function tests
C Most people feel depressed on isotretinoin
D Parts of the skin and lips often get very dry
E Pregnancy should be avoided as teratogenicity is a risk

A

C

392
Q

A 68-year-old man presents with a 3-month history of epigastric pain and weight loss. He has a history of acid reflux, which his GP has been managing.
What is the next most appropriate step?
A Computed tomography (CT) scan of the abdomen
B Dietician review
C Increase his proton pump inhibitor (PPI) and ask him to return in 1
month if symptoms don’t settle
D Treat empirically for H. pylori infection and review in 1 month
E Upper gastrointestinal (GI) endoscopy

A

E

393
Q

A 23-year-old man develops a urethral discharge and dysuria after a recent change of sexual partner and urethral swabs are positive for Chlamydia.
Which of the following statements about antibiotic treatment is TRUE?
A His partners should be asked about symptoms and tested only if symptomatic
B His partners should be tested for Chlamydia and treated only if positive
C His partners should be tested for Chlamydia and treated with antibiotics
regardless of outcome
D No antibiotic treatment is necessary provided he abstains from having
sexual intercourse for 2 weeks
E Only the patient who has presented needs antibiotic therapy

A

C

394
Q

A 75-year-old woman is suffering from osteoarthritis of the hips and knees and paracetamol is not touching the pain. You want to consider use of a non-steroidal anti-inflammatory drug (NSAID).
Which of the following is NOT a relative or absolute contraindication to NSAID use?
A Asthma
B Concomitant aspirin use
C Concomitant steroid use D Congestive cardiac failure E Previous gastric ulcers

A

C

395
Q

Which of the following is NOT a contraindication to renal transplantation?
A Active tuberculosis
B High-pressure urinary tract, e.g. posterior urethral valves
C Malignancy
D Severe arterial disease with stenosed iliac vessels
E Severe ischaemic heart disease with unstable angina and congestive
cardiac failure

A

B

396
Q

Which of the following patients is eligible for thrombolysis with intravenous recombinant tissue plasminogen activator?
A 2 hours post onset, BP 150/80 mmHg, GCS 11, MRI shows infarct
B 2 hours post onset, BP 160/90 mmHg, GCS 15, MRI shows haemorrhage
C 2 hours post onset, BP 160/95 mmHg, GCS 15, MRI shows infarct
D 2 hours post onset, BP 195/115 mmHg, GCS 15, MRI shows infarct
E 7 hours post onset, BP 135/80 mmHg, GCS 15, MRI shows infarct

A

C

397
Q

A 43-year-old woman presents with an acute sharp, central chest pain radiating to the left arm that is worse on inspiration and at night. She reports that it is preventing her coughing which, in turn, is prolonging a recent chest infection. On examination, her chest exhibits vesicular breath sounds with bi-basal crackles. Her electrocardiogram (ECG) shows widespread concave ST elevation in sinus rhythm.
Which of the following is the most likely diagnosis?
A Acute pericarditis
B Cardiac tamponade
C Pulmonary embolism
D ST-elevated myocardial infarction
E Unstable angina

A

A

398
Q

An 87-year-old man with a background of Alzheimer’s disease, a previous stroke, and atrial fibrillation (for which he is on warfarin), is admitted with increasing amounts of painless haematuria. He is catheterised and the warfarin stopped. The haematuria reduces and then seems to stop. Problems with his residential home are delaying discharge. You note 2 days later that his creatinine level has risen to 250 μmol/L. When you go to see him, he seems stable, as do his observations. He does seem slightly dry with reduced skin turgor and delayed capillary refill. The urine output has not been carefully documented in the last few days, and by your calculations, 100 ml have been passed in the last 24 hours. On examination he appears to have a mass in his central lower abdomen.
What should be your next step?
A Fluid challenge of 500 ml normal saline over 10 minutes
B Flush the catheter with 50 ml warm saline and then aspirate
C Insert a suprapubic catheter
D Remove the catheter
E Request an ultrasound of the urinary tract

A

B

399
Q

A 70-year-old woman presents with hip pain following a fall. The fall appears to have been related to alcohol ingestion and, whilst the history is vague, she denies loss of consciousness and does not seem grossly confused, nor is there evidence of infection. She is a smoker and has a history of ischaemic heart disease and depression. A hip fracture is ruled out and she is admitted for rehabilitation purposes. On day 3, however, the nurses report that she is increasingly sleepy and muddled. On examination, observations are stable, she is apyrexial, her Glasgow Coma Score (GCS) is 12, and on neurological examination, she appears to have some mild left arm and leg weakness with normal or brisk reflexes. There is no hemianopia or other neurological deficit.
Which of the following is most likely to explain the changes?
A Bacterial meningitis
B Intracerebral haemorrhage
C Lacunar infarct
D Subarachnoid haemorrhage
E Subdural haematoma

A

E

400
Q
A 40-year-old woman presents to the GP with a history of weight loss and irritability. She has lost 1 stone in the past 2 weeks. She claims that she has a normal appetite. She also complains of blurring of the vision for the past month. On examination, she has a “staring look” with lid lag and lid retraction. 
What is the most likely diagnosis? 
A Graves’ disease
B Hashimoto thyroiditis 
C Multinodular goitre
D Myasthenia gravis
E Toxic thyroid adenoma
A

A

401
Q

A 55-year-old man has presented to the medical admissions unit with chest pain. You have diagnosed acute coronary syndrome (ACS) and are about to write up his drug chart.
Your most suitable prescription would be:
A 75 mg aspirin, clopidogrel, treatment dose heparin stat with glyceryl trinitrite (GTN) spray and morphine PRN
B 75 mg aspirin, clopidogrel, treatment dose heparin and beta-blocker stat with GTN spray and morphine PRN
C 300 mg aspirin, clopidogrel, prophylactic dose heparin stat with GTN spray and morphine PRN
D 300 mg aspirin, clopidogrel, treatment dose heparin and beta-blocker stat with GTN spray and morphine PRN
E 300 mg aspirin, prophylactic heparin and beta-blocker stat with GTN spray and morphine PRN

A

D

402
Q

A 32-year-old man presents to the GP 4 days after an episode of painless haematuria, 2 weeks following a sore throat. He says he now feels he is producing less urine than usual and that it is brown. He denies any weight loss or fatigue, and has no family history of urological malignancy. On examination, he has a blood pressure of 155/90 mmHg, +++ blood on urine dipstick, and blood tests reveal a creatinine of 170 μmol/L and normal electrolytes.
What is the clinical picture most consistent with?
A Acute tubular necrosis
B Nephritic syndrome
C Nephrotic syndrome
D Renal calculi
E Transitional cell carcinoma of the bladder

A

B

403
Q

In the treatment of heart failure with atrial fibrillation, which of the following statements about digoxin is false?
A ACE inhibitors and beta-blockers are more effective at extending survival
B Digoxin has a secondary vagal effect that slows heart rate
C Digoxin is effective at treating the symptoms
D Digoxin is obtained from foxgloves
E Digoxin is primarily a chronotropic drug

A

E

404
Q
A 32-year-old man is diagnosed with Hodgkin’s lymphoma following a recent history of weight loss and night sweats. Computed tomography (CT) staging scan shows disease in the mediastinum bilaterally and some abdominal lymphadenopathy, including the spleen, but no evidence of disease in extranodal sites. 
What is his stage of disease? 
A Stage IIA 
B Stage IIB 
C Stage IIIA 
D Stage IIIB 
E Stage IVB
A

D

405
Q

A 35-year-old woman with a 1-year history of plaque psoriasis is having difficulty with management. Along with emollients, she has tried tar and dithranol, which she did not get on well with (due to the smell and burning, respectively), and has been using a combination of vitamin D analogues and BetnovateTM, a potent topical steroid. She has difficulty finding time with her lifestyle to apply the creams often enough and as a result the lesions have become more widespread and she is now extremely concerned about her appearance, both in the workplace and at her sister’s wedding, which is coming up soon.
Which of the following would be the best treatment to try next?
A Ciclosporin
B Infliximab
C Methotrexate
D Prednisolone
E Ultraviolet therapy

A

E

406
Q

Which of the following statements about subarachnoid haemorrhage is NOT true?
A Focal weakness may ensue
B Lumbar puncture (LP) immediately on admission is likely to show
xanthochromia
C Subarachnoid haemorrhage may present as a new onset of seizures
D Subarachnoid haemorrhages may be provoked by exertion or coitus
E Unenhanced CT within 48 hours is 95% sensitive

A

B

407
Q

A 32-year-old woman presents to the emergency department having reportedly taken a large dose of heroin. She is unresponsive, has small pupils and has a respiratory rate of 6 breaths/min.
Which of the following should be administered?
A Atropine
B Ethanol
C Flumazenil
D N-acetylcysteine
E Naloxone

A

E

408
Q

A 44-year-old man presents to the emergency department with a 3-day history of shortness of breath and a productive cough. His observations include temperature 38.2°C, pulse rate 90 bpm, blood pressure 130/86 mmHg, respiratory rate 24/min and saturations 94% on room air. Blood results include white cell count of 13.5 × 106/L, CRP 45 mmol/L, Na+136 mmol/L, K+4.3 mmol/L, urea 9.2 mmol/L and creatinine 110 μmol/L. On examination, he appears to be alert, and there are dull sounds on percussion over the right lower lobe. A chest X-ray confirms a right lower lung lobe consolidation.
What is the most appropriate management?
A Admission into hospital with empirical antibiotic treatment
B Admission into hospital for nebulised salbutamol and empirical antibiotic
treatment
C Admission into intensive care unit
D Home with inhaled salbutamol when clinically stable
E Home with empirical antibiotic treatment when clinically stable

A

E

409
Q

A 68-year-old obese woman is not coping at home and presents to the emergency department following a fall. On examination, she has a weak, regular pulse and an ejection systolic murmur. You try to lean her forward and palpate her apex beat in expiration but she gets flustered and complains of a sore back and shortness of breath. A neurological examination is unremarkable and her haemoglobin level is 13.2 g/dl.
What is most likely wrong with her?
A Aortic stenosis
B Atrial fibrillation
C Cerebrovascular accident D Gastrointestinal bleed
E Mitral regurgitation

A

A

410
Q
A 70-year-old man presents with multiple widespread tense blisters measuring between 0.5 cm and 5 cm in diameter. They are localised mainly to the arms and legs, with some lesions on the chest. They appeared over weeks, preceded by itchy urticarial lesions. A few lesions have burst and have left not much behind. There is a history of osteoarthritis of the knees for which he takes diclofenac, and he has not changed any medication in the past year. A subsequent biopsy of a bulla shows splitting at the dermoepidermal junction. 
What is the diagnosis? 
A Insect bites
B Pemphigoid
C Pemphigus
D Stevens–Johnson syndrome
 E Toxic epidermal necrolysis
A

B

411
Q
Which of the following antibiotic and side-effect pairings is FALSE? 
A Ceftriaxone and tendon rupture
B Co-amoxiclav and cholestatic jaundice 
C Erythromycin and diarrhoea
D Gentamicin and nephrotoxicity
E Nitrofurantoin and pulmonary fibrosis
A

A

412
Q

Which of the following scoring systems should be used to assess a patient’s risk of developing a pressure score?
A Breslow score
B Confusion, Urea, Respiratory rate, Blood pressure, Age (CURB) score
C Ranson’s criteria
D Rockall score
E Waterlow score

A

E

413
Q

A 78-year-old woman, who has recently been feeling under the weather and losing some weight, develops a severe left-sided headache with jaw pain on eating. She also complains of blurred vision on the left side.
Which of the following investigations is needed to give a definitive diagnosis?
A Computed tomography (CT) of the head
B Erythrocyte sedimentation rate (ESR)
C Magnetic resonance imaging (MRI) of the head
D Temporal artery biopsy
E Ultrasound of the temporal arteries

A

D

414
Q

A 48-year-old man presents to the emergency department in a reduced state of consciousness. He was brought in by his son, who says he has been more confused over the past few days. A collateral history suggests 2 weeks of polyuria and polydipsia. There was no history of head injury, trauma or ingestion of illegal drugs. He has no other significant past medical history. He has a 21 U/week alcohol history. On examination, he is unresponsive to pain. The liver edge is felt on abdominal examination. His blood tests show:
Na+ K+
Glucose
Serum osmolality Urea
Creatinine
168 mmol/L 3.8 mmol/L 68 mmol/L 350 mmol/kg 14.3 mmol/L 203 μmol/L
A urine dipstick test revealed glycosuria but no ketones.
What is the most appropriate approach to the management of this patient?
A Aim for blood glucose concentration fall by 10 mmol/L per hour
B Fluid restriction
C Give treatment dose of low-molecular-weight heparin
D Infuse 1 L of 0.9% NaCl fluid infusion with added 40 mmol potassium at
a rate of 1 L/hour
E Start insulin infusion at 3 U/hour initially

A

E

415
Q

An 81-year-old retired farmer presents with a 1.5 cm raised lesion on his left temple that has slowly grown over the past year and occasionally bleeds. He has had a basal cell carcinoma removed from his right temple previously, and he has had cryotherapy for actinic keratoses on his temples and head several times at his GP surgery. On examination, the lesion has a rolled edge and is translucent with telangiectasia.
Which of the following treatment options is NOT recommended for this lesion?
A Chemotherapy
B Excision
C Mohs micrographic surgery
D Radiotherapy
E Topical 5-fluorouracil

A

A

416
Q

A 35-year-old man presents to the GP with increasing fatigue for 3 months. He finds himself waking up choking a few times at night. He was told by his partner that he snores quite loudly most nights. He denies any weight loss. He smokes 30 cigarettes/day, drinks 2 pints of beer every night and has a body mass index of 32.
What is the most likely differential diagnosis?
A Central sleep apnoea
B Chronic fatigue syndrome
C Idiopathic hypersomnolence
D Narcolepsy
E Obstructive sleep apnoea

A

E

417
Q

A 73-year-old man has been admitted with severe sepsis and acute renal failure secondary to a urinary tract infection. He has been treated with intravenous antibiotics and fluid resuscitation but the following day you are called to see him as he is worse. He looks extremely unwell. His airway is patent and he has laboured breathing at a rate of 22 breaths/min. His saturations are 98% on air and his chest sounds clear. His pulse is 120 bpm and blood pressure 85/55 mmHg. His capillary refill rate is 4 seconds. His urine output has been 40 ml in the last 5 hours. You see from his blood tests that his creatinine level has increased slightly. You take an arterial blood gas reading.
Which of the following is the most likely result?
A pH 7.22, pO8, pCO 8.3, HCO – 24
B pH 7.22, pO18, pCO 2.3, HCO – 10
C pH 7.22, pO18, pCO 8.3, HCO – 10
D pH 7.40, pO8, pCO 8.3, HCO – 38
E pH 7.51, pO18,pCO 2.3,HCO–24

A

B

418
Q
A 65-year-old man presents to the GP with chest pain, increasing shortness of breath and weight loss. He describes the pain as dull and worse on inhalation. He notices that he has lost 1 stone in 3 weeks. He has never smoked and has no other significant past medical history. He used to work in a shipyard. Observations include temperature 36.8°C, pulse rate 80 bpm, blood pressure 140/95 mmHg and respiratory rate 18/ min. On examination, there is dullness to percussion over the left lower lung zone. 
What is the most likely diagnosis? 
A Bronchocarcinoma
B Mesothelioma
C Pneumoconiosis
D Pulmonary embolism 
E Tuberculosis
A

B

419
Q

A 61-year-old woman presents to the walk-in clinic with a history of tight, sternal chest pain “appearing out of the blue”. Such episodes have occurred at different times throughout the day and rarely last longer than a few minutes. They do not correspond to hard exercise. The patient has no cardiac history to date. Her electrocardiogram (ECG) has not detected any abnormalities and 2 days have passed since the last episode.
Which of the following is the next best step in her management?
A 24-hour ECG
B Admit to a ward
C Chest X-ray
D Echocardiogram
E Prescribe home oxygen

A

A

420
Q
A 37-year-old woman is being investigated for weight loss and diarrhoea. Barium enema shows “cobble-stoning”, rose thorn ulcers and colonic strictures at intermittent points throughout the colon. 
What is the most likely diagnosis? 
A Coeliac disease
B Colonic carcinoma
C Crohn’s disease
D Inflammatory bowel disease 
E Ulcerative colitis
A

C

421
Q

You are asked by your registrar to see a 40-year-old woman and report back your findings. On examination, you struggle to find an apex beat although heart sounds 1 and 2 were audible with no murmur. On inspection, her electrocardiogram (ECG) is normal except for inverted P-waves.
What is the most likely reason for these findings?
A Dextrocardia
B Cardiomyopathy
C Mitral stenosis
D Myocardial ischaemia
E Pulmonary hypertension

A

A

422
Q

A 53-year-old man, who has had some lower back pain and sciatica into his right leg for the past year, presents with a 2-day history of leg weakness and severe pain.
On examination, which of the following signs does NOT make you think of cauda equina syndrome?
A Distended bladder
B Reduced anal tone
C Reduced reflexes in the ankles D Saddle anaesthesia
E Upgoing plantars

A

E

423
Q

A 35-year-old man presents with a 3-week history of pain and swelling in the tips of his fingers. He has no history of bowel problems, recent infection or skin disease, but his brother has Crohn’s disease. On examination you find several swollen, red, tender distal interphalangeal joints, and the nails have separated from the nailbed and have small pockmarks covering them. The rest of the examination is unremarkable.
Which of the following diagnoses is most likely?
A Enteropathic arthritis
B Osteoarthritis
C Psoriatic arthropathy
D Reactive arthritis
E Rheumatoid arthritis

A

C

424
Q
Which of the following is a recognised complication of coeliac disease? 
A Fistulae
B Intestinal lymphoma
C Primary sclerosing cholangitis 
D Toxic megacolon
E Uveitis
A

B

425
Q
You are called in the middle of the night to see a 49-year-old man with known alcoholic liver disease admitted 2 days previously. He is shaking, sweating, tachycardic, apyrexial and believes he is seeing spiders crawling across the ceiling. 
What is the most likely diagnosis? 
A Alcohol intoxication
B Delirium tremens
C Bacterial peritonitis
D Fulminant hepatic necrosis 
E Urinary tract infection
A

B

426
Q
A 59-year-old woman with advanced metastatic breast cancer presents to the emergency department with severe abdominal pain. She has not opened her bowels for 7 days and feels constipated. She has also noticed that she has been passing water more frequently but has not been incontinent. On rectal examination there is no loss of anal tone and normal sensation. 
What is the most likely diagnosis? 
A Hypercalcaemia
B Hypocalcaemia
C Metastatic spread to the bowel 
D Opiate-induced constipation 
E Spinal cord compression
A

A

427
Q

A 19-year-old female presents to the emergency department with a severe headache, photophobia, neck stiffness and a temperature. She is treated for bacterial meningitis with intravenous ceftriaxone. Blood cultures grow Neisseria meningitides. The next day, she starts bleeding from around her intravenous cannula and venepuncture sites.
Which of the following investigation results would you NOT expect in disseminated intravascular coagulation?
A Increased activated partial thromboplastin time (APTT)
B Increased fibrinogen
C Increased international normalised ratio (INR)
D Decreased haemoglobin
E Decreased platelets

A

B

428
Q

A 67-year-old man presents with painless rectal bleeding for the past 2 months. It occurs mainly on defecation, and is bright red in the pan and on the toilet paper. A digital rectal examination is unremarkable.
What should your management be?
A Abdominal X-rays
B Review patient in 1 month to see if bleeding has stopped
C Routine bloods
D Routine outpatient appointment in 4–6 weeks
E Urgent 2-week outpatient referral

A

E

429
Q
A 23-year-old final year law student presents to the GP with a cough productive of foul green sputum. He also complains of breathlessness on exertion. He has had an average of five respiratory tract infections per year for the past 2 years. As a child, he could not tolerate dairy products. On examination, finger clubbing is present and there is dullness to percussion of the right upper lung zone with widespread bilateral wheeze. 
What is the most probable diagnosis? 
A Asthma
B Coeliac disease
C Cystic fibrosis
D Immotile cilia syndrome 
E Pneumonia
A

C

430
Q

A 50-year-old overweight pub landlord presents with an acutely painful swollen red hot first metatarsophalangeal joint on the left. You suspect gout.
Which of the following would confirm this on microscopy of a joint aspirate?
A Calcium pyrophosphate crystals
B Crystals of a rhomboid shape
C Crystals showing apple green birefringence under polarised light
D Crystals showing negative birefringence under polarised light
E Crystals showing positive birefringence under polarised light

A

D

431
Q

A 56-year-old man who has been recently diagnosed with sigmoid cancer and had a Hartmann’s procedure is admitted to hospital with acute-onset shortness of breath. He denies any collapse or fainting. He has a past medical history of chronic obstructive pulmonary disease, hypertension, stroke and gout. His observations include temperature 37.2°C, pulse rate 106 bpm, blood pressure 110/74 mmHg, respiratory rate 25/min and saturations 87% on room air. There is no significant finding on chest examination. An electrocardiogram (ECG) shows sinus tachycardia with no ST elevation. A plain chest X-ray shows a small left pleural effusion.
Which of the following is the most appropriate diagnostic investigation?
A Magnetic resonance imaging (MRI) of the chest
B D-dimer level
C Computed tomographic pulmonary angiography
D Ventilation-perfusion scanning
E Echocardiography

A

C

432
Q

A 27-year-old woman who works in the city as an investment banker presents with difficulty walking, and a tremor in the right hand. It has come on over the last 4 days and she is now unable to get to work safely. She denies any preceding respiratory or gastrointestinal infection or any abnormal stress at work or in her personal life; however, she says that 5 months ago she had some pain and blurring in the left eye for about 10 days, but she put that down to “sinusitis”, kept working, and things returned to normal. Her grandfather, who died 2 years ago, had Parkinson’s disease for some time, and she is concerned that this could be the same thing. On examination, in her arms, there is normal tone, power and sensation, but there is indeed a tremor in the right hand on testing coordination, which gets worse on approaching the target, and is absent at rest. In the legs, there is normal power, but decreased light touch sensation bilaterally and a loss of vibration sense and proprioception up to the ankles.
What are you most concerned is the diagnosis?
A Acute disseminated encephalomyelitis
B Familial early-onset Parkinson disease
C Guillain–Barré syndrome
D Multiple sclerosis
E Psychogenic neurological symptoms

A

D

433
Q

A 33-year-old woman with multiple sclerosis (MS) is having problems with painful spasms and disabling spasticity in her left leg.
Which of the following medications is most likely to help?
A Baclofen
B Lactulose
C Modafinil
D Oxybutinin
E Propanolol

A

A

434
Q
A 65-year-old woman presents to hospital with left-sided weakness of sudden onset. She is a type II diabetic, smokes 20 cigarettes/day and has high blood pressure. On examination, power in the left arm is 0 throughout although the left leg shows a power of 3 at the hip and knee and 4 at the foot. Her reflexes are reduced throughout, and sensation is absent in the arm and reduced in the leg. In the cranial nerve examination she is unable to see on her left-hand side, and the lower half of her left face is weak (she can raise her eyebrow). There is no dysphasia. 
Which vascular territory is affected? 
A Left anterior cerebral artery
B Right anterior cerebral artery 
C Right carotid artery
D Right middle cerebral artery
E Right posterior cerebral artery
A

D

435
Q

A 45-year-old man presents to the GP with a 4-month history of a productive cough with exertional breathlessness. He denies haemoptysis or weight loss. He has a 20 pack/year smoking history. On examination, fine crackles are heard throughout the whole lung field.
What is most likely differential diagnosis?
A Asthma
B Chronic obstructive pulmonary disease
C Cystic fibrosis
D Idiopathic pulmonary fibrosis
E Lung cancer

A

B

436
Q

A 70-year-old woman who has been in hospital for 5 days with severe pneumonia and treated successfully with intravenous co-amoxiclav and oral clarithromycin develops profuse watery diarrhoea. Her pulse rises to 110 bpm and she develops a temperature of 38.3°C. She is also taking omeprazole for reflux disease. Stool samples are positive for Clostridium difficile toxin.
Which one of the following is the best treatment plan?
A Barrier nursing in a side room
B Continue present antibiotic therapy
C Intravenous vancomycin
D Oral vancomycin
E Stop co-amoxiclav and omeprazole

A

D

437
Q
A 54-year-old woman presents to her GP complaining of repeated incidents of burning central chest pain. It mainly occurs when she lies down to go to bed at night. She is overweight with a body mass index of 40. She uses glyceryl trinitrate (GTN) occasionally but it doesn’t always relieve her symptoms. She doesn’t report any shortness of breath or palpitations and examination is unremarkable. 
What is the most likely diagnosis? 
A Angina
B Gastro-oesophageal reflux disease 
C Myocardial infarction
D Pancreatitis
E Sleep apnoea
A

B

438
Q

Which of the following patients meets the Modified Duke criteria for having definite endocarditis?
A One blood culture positive for a Gram-negative bacillus, pyrexia of 38.5°C and splinter haemorrhages and Osler nodes
B One blood culture positive for Streptococcus viridans and a new soft pansystolic murmur loudest at the apex
C Pyrexia of 38.5°C, intravenous drug abuse, Osler nodes and splinter haemorrhages
D Two blood cultures positive for Strep. viridans and a new crescendo– decrescendo ejection systolic murmur
E Two blood cultures positive for Strep. viridans, pyrexia of 38.5°C, splinter haemorrhages and Osler nodes, and new small cerebral abscesses

A

E

439
Q

clammy to the touch. Whilst she is clutching your hand and telling you her past medical history, you check her observations on the bedside monitor. You notice that her heart rate is 230 bpm and regular. The QRS complexes are narrow and no P-waves are noted.
What is the most likely arrhythmia?
A Atrial fibrillation
B Atrial flutter
C Atrio-ventricular node re-entry tachycardia
D Sinus tachycardia
E Ventricular tachycardia

A

C

440
Q

A 4-year-old boy swallowed a button battery while playing with his older siblings. He and his mother present to you, the GP, after a few hours. He is otherwise well in himself and has no previous medical history.
What should you do?
A Attempt gastric lavage
B Give laxatives to speed up the passage through the gut
C Manage expectantly and wait for it to pass naturally
D Refer immediately to hospital for evaluation and possible endoscopic
retrieval
E Refer to social services

A

D

441
Q

A 16-year-old boy presents with a 1-year history of acne affecting his face and upper back. He has tried over-the-counter creams and “a few creams prescribed by his GP”, which turn out to be benzoyl peroxide and an antibiotic cream. None have worked and the lesions have been getting worse. His friend had treatment with roaccutane (isotretinoin), which sorted it out, and he is keen to try this. On examination he has acne on his face and upper back but without large pustules or scarring.
Which therapy might you recommend trying before isotretinoin?
A Oral cyproterone acetate B Oral flucloxacillin
C Oral lymecycline
D Oral prednisolone
E Topical steroids

A

C

442
Q
A new blood test is being developed to help diagnose sarcoidosis. In a trial 100 patients have been tested. The trial produces 20 positive results and 80 negative results. Of the 20 positive results, 10 of them are false positives. Of the 80 negative results, 10 of them are false negatives. 
What is the sensitivity of this test? 
A 10% 
B 25% 
C 50% 
D 75% 
E 90%
A

C

443
Q
A 34-year-old woman, who is known to be human immunodeficiency virus (HIV) positive, presents with multiple lesions on her face. The lesions are raised and shiny, non-tender and around 3 mm in diameter. They have an umbilicated centre. 
What is the most likely diagnosis? 
A Herpes simplex
B Kaposi sarcoma
C Molluscum contagiosum 
D Scabies
E Syphilis
A

C

444
Q

A 49-year-old man presents to the emergency department with acute- onset right-sided pleuritic chest pain and shortness of breath. There was no associated trauma. He has a past medical history of chronic obstructive pulmonary disease, hypertension and diabetes mellitus. He smokes 40 cigarettes per day. His observations include temperature 37.2°C, pulse rate 95 bpm, blood pressure 150/92 mmHg, respiratory rate 24/min and saturations 92% on room air. A chest X-ray (CXR) shows a small rim of air (less than 2 cm) between the right lung edge and the chest wall.
Which of the following is the most appropriate first-line management?
A Admit to be observed only
B Intercostal tube drainage
C Medical pleurodesis
D Percutaneous needle aspiration
E Surgical pleurodesis

A

B

445
Q

A 52-year-old man with a history of alcohol excess presents to his GP because he cannot fully extend his ring finger on his right hand (neither actively nor passively). Since the finger is permanently partially flexed he can no longer place his hand flat on a flat surface.
Which of the following is the most likely diagnosis?
A Asterixis
B Dupuytren contracture
C Palmar erythema
D Trigger finger
E Xanthelasma

A

B

446
Q

A 63-year-old man is admitted to the emergency department with severe dyspnoea. He is a longstanding smoker of 40 years. His observations are pulse rate 130 bpm, respiratory rate 36/min, oxygen saturation of 85% on 24% oxygen via a venturi mask and temperature 38.2°C. Chest examination reveals reduced air entry on both lungs with coarse crackles on his left lower lobe. Despite immediate maximum standard medical treatment on controlled oxygen therapy, his arterial blood gas still shows persistent acidosis with a PaCO2 of 8 kPa.
Which of the following would NOT be a required inclusion criterion for non-invasive ventilation?
A Ability to protect his airway
B Home oxygen use
C Patient’s wishes considered
D Potential for recovery to quality of life acceptable to the patient
E Primary diagnosis of chronic pulmonary obstructive disease

A

B

447
Q
A 34-year-old woman with systemic lupus erythematosus (SLE) has had multiple miscarriages and now presents with a painful right swollen leg. A compression ultrasound scan confirms deep vein thrombosis. 
Which blood test may now be indicated? 
A Anti-phospholipid antibodies 
B Clotting factors
C Haemoglobin
D Pregnancy test 
E Tumour markers
A

A

448
Q

A 74-year-old man with a history of hypertension, ischaemic heart disease and moderate congestive cardiac failure is admitted with a urinary tract infection and has developed acute renal failure. His potassium level is noted to be 5.5 mmol/L.
Which of his following medications may be contributing to the raised serum potassium level?
A Amlodipine
B Bendroflumethiazide
C Furosemide
D Simvastatin
E Spironolactone

A

E

449
Q

A 42-year-old woman attends to her GP complaining of non-specific abdominal pain and an increasing abdominal girth. She is found to have a large mass in her right lower abdomen and ascites on transvaginal ultrasound imaging.
Which of the following tumour markers would be most useful?
A CA 125
B Ca 15-3
C Ca 19-9
D CEA
E Beta-hCG

A

A

450
Q

Which of the following best describes the four cardinal signs of Parkinson’s disease?
A Bradykinesia, mask-like facies, rigidity, and tremor ~10–12 Hz
B Bradykinesia, micrographia, rigidity, and tremor ~3–4 Hz
C Bradykinesia, postural hypotension, rigidity, and tremor ~3–4 Hz
D Bradykinesia, postural instability, rigidity, and tremor ~3–4 Hz
E Bradykinesia, postural instability, rigidity, and tremor ~10–12 Hz

A

D

451
Q

You find an 80-year-old man collapsed in the street. He is unresponsive and is making a snoring sound. An ambulance has been called but has yet to arrive.
Which of the following is the best course of action?
A Cricothyroidotomy
B Do nothing till the ambulance arrives
C Finger sweep
D Head tilt chin lift
E Place in the recovery position

A

D

452
Q

A 26-year-old woman presents to the emergency department with an oxygen saturation reading of 80% on air with a background history of increased breathlessness on exertion over the course of a week. She has not travelled abroad recently and does not have a family history of thromboembolism. On examination, there is elevation of the jugular venous pressure and accentuation of the pulmonary component of the second heart sound. A chest examination is unremarkable. A chest X-ray is normal and a subsequent CT angiography demonstrated a pulmonary embolism.
Which of the following management is NOT appropriate?
A Request an echocardiogram
B Request a thrombophilia screen
C Request tumour markers
D Start treatment dose low-molecular-weight heparin
E Start warfarin treatment

A

C

453
Q

A 34-year-old woman presents to the GP complaining of a new rash. The skin under her arms and on the back of her neck is dark and velvety in texture. She has a past medical history of diabetes for which she takes insulin.

What is the most likely diagnosis? 
A Acanthosis nigricans 
B Diabetic dermopathy 
C Erythema ab igne
D Hyperhidrosis 
E Xanthelasma
A

A

454
Q

Which of the following scoring systems can be used to assess the risk of an adverse outcome following an upper gastrointestinal bleed?
A Breslow score
B CURB (Confusion, Urea, Respiratory rate, Blood pressure, Age) score
C Ranson’s criteria
D Rockall score
E Waterlow score

A

D

455
Q
A 64-year-old man presents to the emergency department with a large rectal bleed and left-sided abdominal pain. He has a long history of constipation. He has a temperature of 38.4°C and a heart rate of 110 bpm. 
What is the most likely cause? 
A Angiodysplasia
B Bowel malignancy
C Diverticulitis
D Haemorrhoids
E Pseudomembranous colitis
A

C

456
Q

A 55-year-old man is being investigated for irregular heart rhythms. He has a medical history of diabetes mellitus. He explains that exercise is difficult for him due to joint pains. During the examination it is noted that he has tan skin pigmentation and hepatomegaly.
Which of the following investigations could reveal the aetiology of his symptoms?
A Haematinics
B Serum caeruloplasmin
C Short synacthen test
D Alpha-1 antitrypsin
E Gamma-GT

A

A

457
Q

A 64-year-old woman is an inpatient on a surgical ward following an above- knee amputation. The nursing staff said she vomited earlier and she has been unresponsive since her operation a few hours ago. On examination, you noticed her pupils are small and she has a respiratory rate of 5 breaths/ min.
Which of the following is the most likely reason for this clinical picture?
A Alcohol withdrawal
B Opiate use
C Opiate withdrawal
D Sedative use
E Sedative withdrawal

A

B

458
Q

A 66-year-old man with a 10-year history of chronic obstructive pulmonary disease is assessed in the respiratory clinic for eligibility for long-term domiciliary oxygen therapy.
Which of the following is NOT a criterion for prescription of long-term oxygen therapy?
A No exacerbation of chronic obstructive pulmonary disease (COPD) for the previous 5 weeks
B Patient has stopped smoking
C Patient has chronic hypoxaemia with PaO2 <7.3 kPa
D Presence of pulmonary hypertension with PaO2 <8.0 kPa
E Two arterial blood gases showing PaO2 <7.3 kPa within 7 days

A

E

459
Q
A 68-year-old woman is recently not coping at home. She is now experiencing episodes of central, choking chest pain and shortness of breath on exertion. Her ECG is normal, as are her bloods and chest X-ray. Which of the following will NOT help her during her next episode? 
A Bisoprolol
B Glyceryl trinitrate (GTN) spray 
C Morphine
D Oxygen
E Salbutamol nebulisers
A

E

460
Q

A 33-year-old woman who is 10 weeks’ pregnant develops increased frequency of urination and dysuria. Urine dip is positive for nitrites.
Which of the following antibiotics could be used to empirically treat the urinary tract infection?
A Ciprofloxacin
B Co-amoxiclav
C Doxycycline
D Trimethoprim
E Vancomycin

A

B

461
Q

An 80-year-old man attends the general practice for an annual check-up of his hypertension. He gingerly tells you all about his “left ventricular hyperthingummy”. You check him and his records for confirmatory signs and symptoms.
Which of the following is NOT a sign of left ventricular hypertrophy?
A Inverted T-waves
B Left axis deviation
C Pansystolic murmur
D R-wave in V6 >25 mm
E The sum of the magnitude of the S-wave in V1 and R-wave in V5 >35mm

A

C

462
Q
A 79-year-old woman who was admitted to hospital with a fractured right neck of femur 1 week ago suddenly becomes acutely unwell on the ward 6 days after her operation. She complains of sudden-onset of shortness of breath and chest tightness. The pain is exacerbated by deep breathing. She has a past medical history of hypertension, hypercholesterolaemia and asthma. Her observations include temperature 37.8°C, pulse rate 108 bpm, blood pressure 96/66 mmHg, respiratory rate 26/min and saturations 89% on room air. On examination, her chest is clear to both auscultation and percussion. An electrocardiogram (ECG) shows sinus tachycardia without bundle branch block. 
What is the most likely diagnosis? 
A Acute exacerbation of asthma 
B Myocardial infarction
C Pneumonia
D Pneumothorax 
E Pulmonary embolism (PE)
A

E

463
Q

A 43-year-old man presents with intermittent haematuria. On examination of the abdomen, bilateral masses are felt and an ultrasound reveals polycystic kidneys. You explain the syndrome to him. He is concerned that his son may develop the disease. He does not believe his wife suffers from the condition.
What is the probability that his son will develop the disease?
A Negligible
B One in two
C One in four
D One in a thousand
E One in two thousand

A

B

464
Q

A 35-year-old homeless man presents to the emergency department unconscious and fitting. You estimate that he has been fitting now for 35 minutes. He smells of alcohol and looks dishevelled. He has an adequate airway and is breathing 10 L/min O2. His pulse and blood pressure are within normal range, and his capillary glucose is normal.
What should your immediate management be?
A Diazepam 2 mg intravenously
B Diazepam 2 mg intravenously and Pabrinex intravenously
C Lorazepam 4 mg intravenously and Pabrinex intravenously
D Lorazepam 8 mg intravenously
E Lorazepam 8 mg intravenously and Pabrinex intravenously

A

C

465
Q

A 17-year-old female has taken 20 paracetamol tablets 3 hours ago in an attempt to end her life after an argument with her mum. She has now attended hospital with regret and is asking for treatment.
Which of the following should be administered?
A Atropine
B Desferrioxamine
C Digibind
D N-acetylcysteine
E Sodium bicarbonate

A

D

466
Q

You are examining a tearful young child who has fractured her arm. On auscultation you hear an ejection systolic murmur. There is no cyanosis.
Which of the following prevents you reassuring her calm dad that his daughter has no serious heart problem?
A A history of cyanosis
B The fracture needs to heal first
C The second heart sound is split
D The murmur disappears on sitting up
E The murmur is soft

A

A

467
Q

A 60-year-old man presents to the emergency department with shortness of breath for 3 hours and chest pain. He also complains of a right calf pain that he has had for 2 weeks. He has a past medical history of hypertension, recent stroke and disseminated lung cancer. His observations include temperature 37.0°C, pulse rate 112 bpm, blood pressure 100/54 mmHg, respiratory rate 26/min and saturations 87% on room air. There is no significant finding on chest examination. An electrocardiogram (ECG) shows sinus tachycardia and new right bundle branch block. A computed tomography (CT) pulmonary angiography confirmed massive pulmonary embolism.
What is the most appropriate initial treatment?
A Intravenous caval filters
B Oral warfarin
C Prophylactic dose of low-molecular-weight heparin
D Thrombolysis
E Treatment dose of low-molecular-weight heparin

A

E

468
Q

A 55-year-old man is found to have a gastric ulcer following an endoscopy for left upper quadrant pain. A rapid urease test is positive for H. pylori infection.
What treatment does this man need?
A H2 antagonist plus metronidazole and clarithromycin
B Metronidazole, clarithromycin and amoxicillin
C PPI plus metronidazole and clarithromycin
D PPI plus H2 antagonist plus an antibiotic
E Steroids + PPI + metronidazole

A

C

469
Q

A 55-year-old man presents to his GP with increasing lethargy and polyuria. He has a past medical history of ischaemic heart disease and congestive cardiac failure. He smokes 30 cigarettes per day and drinks alcohol occasionally. He has a body mass index (BMI) of 32. His random blood glucose is 14.0 mmol/L and fasting blood glucose level is 9.0 mmol/L.
Which of the following management is NOT appropriate in this patient?
A Advise the patient to change his diet and stop smoking
B Metformin should be considered as the first-line oral treatment option
for overweight patients
C Sulphonylureas and metformin could be considered as a combined
therapy if glycaemic control is not optimal
D Sulphonylureas should be considered if patient is intolerant to metformin
E Thiazolidinediones can be added to metformin and sulphonylurea

A

E

470
Q

A 79-year-old woman is started on high-dose steroids for giant cell arteritis. As therapy will likely be maintained for at least a year, she needs bone protection. The decision is made to start alendronate 70 mg once weekly.
How would you advise her to take the tablet?
A Take it in bed at least half an hour before rising for breakfast
B Take it whilst sitting or standing just before breakfast
C Take it whilst sitting or standing upright at least half an hour after
breakfast with minimal sips of water
D Take it with plenty of water whilst sitting or standing upright at least half
an hour after breakfast
E Take it whilst sitting or standing upright at least half an hour before
breakfast with plenty of water

A

E

471
Q

A 59-year-old man is admitted to the emergency department following a fall. He complains of increased tiredness and jerking movement of his legs, which led to his fall from the staircase. He is a smoker. On arrival, he has an oxygen saturation level of 76% in air, and an arterial blood gas reading demonstrates: pH 7.40, PaO 6 kPa, PaCO 9.3 kPa, HCO − 35 mmol/L on
223 room air. His respiratory rate was 20/min.
What does this blood gas result show?
A Acute type 1 respiratory failure
B Acute type 2 respiratory failure
C Compensated chronic type 2 respiratory failure
D Metabolic acidosis
E Respiratory alkalosis

A

C

472
Q

A 55-year old man fractures his ankle attempting to replicate the latest dance fad he has seen his children do. He is otherwise fit and well and is haemodynamically stable. The fracture requires open reduction and internal fixation. The trauma registrar has said that he needs to be nil-by- mouth from midnight, however the operating list is always changing and new emergencies may come in. There are at least three cases that are likely to get done before him. Your request that he have breakfast as he is likely to be done later in the day is met with derision. He will need physiological fluid replacement when he is nil-by-mouth. He weighs 70 kg.
Which of the following regimens is closest to physiological needs?
A 1 L 0.9% normal saline with 20 mmol potassium and 2 × 1 L 5% dextrose in 24 hours
B 1 L 0.9% normal saline with 20 mmol potassium and 2 × 1 L 5% dextrose with 20 mmol potassium in 24 hours
C 2 × 1 L 0.9% normal saline with 20 mmol potassium and 1 L 5% dextrose in 24 hours
D 2 × 1 L 0.9% normal saline with 20 mmol potassium and 1 L 5% dextrose with 20 mmol potassium in 24 hours
E 3 L Hartmann’s in 24 hours

A

B

473
Q

In haemolytic anaemia which is the correct pattern of investigations that you would expect to see?
A Low haemoglobin, low unconjugated bilirubin, high haptoglobin
B Low haemoglobin, high unconjugated bilirubin, high urinary
urobilinogen, and low haptoglobin
C Microcytic anaemia, normal bilirubin, high transferrin, low ferritin
D Normocytic anaemia, normal bilirubin, low iron, low TIBC, normal
ferritin
E Normal haemoglobin raised conjugated bilirubin, normal haptoglobin

A

B

474
Q

You are the designated driver on Saturday night and are soberly walking to your car with your friends. You spot an elderly man lying on the ground. He is not breathing and has no pulse although he is warm. You ask a friend to call 999. His airway is clear. Your praecordial thump fails.
The most likely way for his heart to resume beating is:
A Adrenaline
B Defibrillation
C Cerebral reoxygenation
D Chest compressions
E Recovery position

A

B

475
Q
Which of the following diseases is the most common reason for misdiagnosis of Parkinson’s disease? 
A Corticobasal degeneration
B Essential tremor/familial tremor 
C Progressive supranuclear palsy 
D Pugilist encephalopathy
E Wilson’s disease
A

B

476
Q

A 46-year-old woman has been diagnosed with rheumatoid arthritis and has been on methotrexate for over a year, but sadly her disease is not under control. She wants to know about the new drugs used for rheumatoid arthritis that she heard about at a patients’ association.
Which of the following statements about biological disease-modifying anti-rheumatic drugs (DMARDs) is NOT true?
A AchestX-rayshouldbetakenbeforestartingtoruleoutlatenttuberculosis (which could be reactivated)
B If the patient’s disease gets worse whilst on biological therapy, switching from one biological DMARD to another is unlikely to produce an improvement
C Injection site reactions for subcutaneously injected agents are common
D The more commonly used agents target and inhibit tumour necrosis
factor alpha (TNF-alpha)
E They can be used in combination with methotrexate

A

B

477
Q

A 55-year-old woman presents with mild cognitive impairment and disinhibition without significant mood change. She also has falls and urinary incontinence. She has no major past medical history or medications, and has never smoked or drank much alcohol. On examination, there are no cranial nerve or arm defects, however the legs appear to have increased tone, upgoing plantars and clonus bilaterally with some weakness of hip and knee flexion bilaterally.
Which of the following diagnoses may explain the picture?
A Alzheimer’s disease
B Benign intracranial hypertension
C Lewy body dementia
D Normal pressure hydrocephalus (NPH)
E Vascular dementia

A

D

478
Q

A 45-year-old man is having his second transfusion following blood loss from a road traffic collision. During the transfusion he developed a fever of 38.5°C and rigors.
What should your immediate action be?
A Contact haematology for urgent advice
B Give steroids and antihistamines and continue transfusion
C Start antibiotics
D Stop the transfusion, take blood cultures and provide supportive measures
E Take blood cultures and continue transfusion

A

D

479
Q
A 32-year-old woman presents to her GP complaining of tingling and numbness around her mouth for 1 week. Occasionally, she also develops painful carpal spasm. She has a past medical history of Graves disease for which she just recently had subtotal thyroidectomy. 
What is the most likely diagnosis? 
A Hypercalcaemia
B Hypermagnesaemia 
C Hypocalcaemia
D Hypophosphataemia 
E Hypothyroidism
A

C

480
Q

A 62-year-old woman presents to the emergency department with collapse. She felt dizzy when she tried to stand up from a sitting position. She did not lose consciousness. She denied any visual disturbances, headache or head injury. She also complained of fatigue over the past month. She takes only omeprazole and paracetamol. An ECG showed normal sinus rhythm. Blood pressure was 102/50 mmHg with a postural systolic drop of 30 mmHg. Blood test showed revealed Na+ 126 mmol/L, K+ 6.5 mmol/L, urea 10.0 mmol/L, creatinine 139 μmol/L.
Which of the following will be most useful in establishing the diagnosis?
A Low-dose dexamethasone suppression test
B Random cortisol level
C Renal ultrasound scan
D Short synacthen test
E Water deprivation test

A

D

481
Q

A 70-year-old man, admitted two days previously with a mechanical fall and needing rehabilitation, becomes increasingly anxious and agitated on the ward. He has been pestering the nurses for some “moonshine”, and he is now disorientated in place and time. He claims to have seen little gnomes dashing between patients’ beds. You suspect delirium tremens.
Which of the following represents best management?
A Intravenous Pabrinex
B Oral Pabrinex
C Reducing dose chlordiazepoxide
D Reducing dose chlordiazepoxide and intravenous Pabrinex
E Reducing dose chlordiazepoxide and oral Pabrinex

A

D

482
Q
Which of the below results is the best indicator of poor liver function? 
A Raised alanine transferase 
B Raised albumin
C Raised alkaline phosphatase 
D Raised bilirubin 
E Prolonged prothrombin time
A

E

483
Q

A 60-year-old woman presents to her GP with a chronic cough associated with thick, yellow sputum for the past year. Sometimes, the sputum is blood-tinged. She had been prescribed multiple courses of antibiotics but they did not seem to help. She had a past medical history of severe pneumonia that required admission to the intensive care unit for 20 days. On chest examination, there are inspiratory crackles throughout the lung fields, with normal vesicular breath sounds.
Which of the following is the most likely differential diagnosis?
A Bronchial carcinoma
B Bronchiectasis
C Chronic pulmonary obstructive disease
D Idiopathic pulmonary fibrosis
E Pneumonia

A

B

484
Q

A 31-year-old man presents with a 1-month history of a growing round, flat, erythematous lesion on the left thigh. The border is slightly more erythematous than the rest of the lesion and has some flakiness of the skin. The lesion itches. The patient also has had long-standing itchy scales between his toes.
Which of the following investigations would help confirm the most likely diagnosis?
A Excisional biopsy
B Incisional biopsy
C Punch biopsy
D Skin scrapings to be sent for microscopy culture and sensitivities
E Skin swab to be sent for microscopy culture and sensitivities

A

D

485
Q

A 56-year old man has a myocardial infarction the day after a hernia repair. You request an ECG and then compare it to his pre-admission trace. You notice ST-segment changes in leads II, III and aVF.
In which part of the myocardium is the infarct?
A Anterior
B Anterolateral
C Anteroseptal
D Inferior
E Posterior

A

D

486
Q
A 19-year-old female university student presents with problems in both arms and difficulty walking. Since starting her course she has had difficulty writing, typing, and other such activities, and thus has struggled to keep up with the workload. She says the arms have “felt unusual” for around 6 months. She describes finding small burns and blisters on her arms that she can’t remember getting in the first place. In the last few weeks she has also fallen twice when walking up the stairs. The only other thing she describes is some mild occipital headaches that have increased in frequency lately. She is otherwise fit and well. 
On examination, there is a loss of pinprick sensation found throughout the arms and on the back to around T3 level. Her hands appear to have some element of small muscle wasting and there is some loss of power throughout the arms. Reflexes are hard to elicit or possibly absent. In the legs, there is some mild symmetrical weakness, and the knee reflexes appear brisk. Plantars appear equivocal. 
What is the most likely diagnosis? 
A Cervical spondylosis
B Multiple sclerosis
C Psychogenic neurological symptoms 
D Syringomyelia
E Viral transverse myelitis
A

D

487
Q

A 74-year-old woman presents to hospital with an acute right-sided hemiparesis, and is found to have a left middle cerebral artery infarct on diffusion-weighted magnetic resonance imaging (MRI). It is her first stroke. Her past medical history is unremarkable. Her blood pressure is normal and her electrocardiogram (ECG) shows sinus rhythm with occasional ventricular ectopics. Blood tests show normal cholesterol and normal glucose. On carotid Doppler she is found to have an 85% stenosis of the left carotid.
Which of the following treatments will NOT benefit her?
A ACE-inhibitor
B Aspirin
C Left carotid endarterectomy
D Statin
E Warfarin

A

E

488
Q

A 24-year-old woman who has been travelling to India on a gap year presents to clinic as she is concerned about a “funny-looking mole” on her leg. She is unsure how long it has been there.
Which of the following is not a concerning feature of a mole when considering a diagnosis of malignant melanoma?
A Asymmetry
B Bleeding
C Border irregularity
D Colour different to that of other moles on patient
E Itching

A

D

489
Q

A 42-year-old woman presents to her GP with a 2-month history of nipple discharge from her left breast. The discharge is milky in colour. No blood is noted. She has a past medical history of hypertension, type 2 diabetes mellitus and depression. She has been taking regular medication for these conditions. She has been amenorrhoeic for 6 months. A provisional diagnosis of hyperprolactinaemia is made.
What of the following is NOT appropriate management?
A Breast examination
B Serum prolactin
C Stop antidepressants
D Thyroid function tests
E Urgent computed tomography (CT) of the brain

A

E

490
Q

A 52-year-old woman presents to the GP with intermittent palpitations and breathlessness that has occurred over the last few weeks. She denies chest pain. Her electrocardiogram (ECG) shows sinus tachycardia and she does not appear to be anaemic.
Which blood test would be of most use in confirming the diagnosis?
A Haematinics
B Lactate
C No blood test required
D Thyroid function tests
E Troponin

A

D

491
Q

A 33-year-old man presents to his GP complaining of worsening headaches and tiredness for the last 2 months. He denies drinking alcohol and smokes occasionally. He has a good appetite and has gained 3 pounds over the past month. He also noticed that he has difficulty in biting his food, and has a reduced libido. On examination, he is tall, with an oval-shaped face, large hands and prominent lower jaw.
Which of the following tests is the most useful in the initial diagnosis?
A Growth hormone levels during an oral glucose tolerance test
B IGF-1 levels
C MRI scan of pituitary fossa
D Pituitary function test
E Random growth hormone levels

A

A

492
Q
In which of the following circumstances would use of emollients be ill- advised? 
A Acne vulgaris
B Contact dermatitis 
C Eczema
D Psoriasis
E Wound healing
A

A

493
Q

An 83-year-old man is admitted with acute confusion. He has an extensive medical history including atrial fibrillation, type 2 diabetes, osteoarthritis, hypertension and some mild congestive cardiac failure, for which he takes several medications. He appears clinically dry, with a pulse of 115/min, dry mucous membranes and a capillary refill rate of 4 seconds. He is noted to have a reduced urine output with concentrated urine. His creatinine is 235 μmol/L.
Which of the following medications does not need be reduced or stopped?
A Amlodipine
B Diclofenac
C Digoxin
D Furosemide
E Metformin

A

A

494
Q
A 27-year-old woman presents to the GP complaining of intense sharp pain on passing stools, accompanied by bright red bleeding noticed on the toilet 
paper. She has no fever and is otherwise well in herself. She recently had a course of codeine phosphate following a sprain injury to her ankle. 
What is the most likely diagnosis? 
A Anal abscess
B Anal carcinoma 
C Anal fissure
D Crohn’s disease 
E Haemorrhoids
A

C

495
Q
A 15-year-old girl is being investigated for behavioural changes and is found to have a green–yellow discolouration around her iris on slit lamp examination. 
What is the name of this feature? 
A Arcus senilis
B Caput medusae
C Kayser–Fleischer rings 
D Lens dislocation
E Xanthoma
A

C

496
Q

Which of the following is NOT an indication for haemodialysis in patients with acute renal failure?
A Acidosis with a pH <7.2
B Hypertension >220 mmHg systolic or 160 mmHg diastolic
C Persistent potassium of >7 mmol/L
D Refractory pulmonary oedema
E Uraemic pericarditis

A

B

497
Q
A 43-year-old woman attends the GP with a 3-month history of a grey–white vaginal discharge which she says has a “fishy” odour. She is systemically well and has no menstrual abnormalities. 
What is the most likely diagnosis? 
A Bacterial vaginosis (BV) 
B Candida
C Chlamydia
D Gonorrhoea 
E Syphilis
A

A

498
Q

A 35-year-old woman has had a diffuse mild arthralgia with a fluctuating time course for the last year. There is never much synovitis to be seen on examination and inflammatory markers are only mildly raised. ANA is positive.
Which of the following would make a diagnosis of systemic lupus erythematosus (SLE)?
A Anterior uveitis
B Anti-ribonucleoprotein antibodies
C Erosions on X-rays of affected joints
D Painful palpable purple lumps on the shins
E Red rash across the cheeks, worse in summer

A

E

499
Q
Which of the following is NOT a sign of uraemia? 
A Clouding of consciousness 
B Hiccoughs
C Lemon yellow skin tone
D Spider naevi 
E Twitching
A

D

500
Q

A 30-year-old man presents to the medical assessment unit with a history of excessive drinking and urination. He has been going to the toilet about 7–8 times per day for 1 month. His random blood glucose is 9.3 mmol. His results on admission show:
Na+ K+
Urea
Creatinine Corrected Ca2+ Plasma osmolality Urine osmolality
130 mmol/L 3.5 mmol/L 2.4 mmol/L 80 μmol/L 2.34 mmol/L 609 mOsm/kg 145 mOsm/kg
Water deprivation test – urine osmolality 296 mOsm/kg after DDAVP 2 μg administered intramuscularly
What is the most likely diagnosis?
A Acute tubular necrosis
B Cranial diabetes insipidus
C Nephrogenic diabetes insipidus
D Primary polydipsia
E Type 2 diabetes mellitus

A

B

501
Q

Which of the following statements regarding Alzheimer’s disease is FALSE?
A Cholinesterase inhibitors may halt progression
B Computed tomography (CT) or magnetic resonance imaging (MRI) may
show medial temporal atrophy
C Pathologically, extracellular beta-amyloid plaques are seen
D Pathologically, intracellular tau-protein neurofibrillary tangles are seen
E Short-term memory impairment is an early symptom

A

A

502
Q

A 78-year-old man taking warfarin for a metallic aortic valve presents with 5 days of cough productive of purulent sputum, fever, anorexia and confusion. He is diagnosed with community-acquired pneumonia and is admitted for antibiotic therapy.
Which of the following antibiotics does NOT interact with warfarin?
A Ciprofloxacin
B Clarithromycin
C Gentamicin
D Metronidazole
E Rifampicin

A

C

503
Q

A 32-year-old man with known type I diabetes mellitus presents to the emergency department with nausea and vomiting. His blood glucose level was 24 mmol/L, with high blood ketones and K+ 4.5 mmol/L.
Which of the following management is NOT appropriate in this patient on the first day?
A Commence intravenous 0.9% normal saline of 1 L over 1 hour
B Start intravenous insulin infusion at a rate of 6 U/hour
C Start intravenous K+ supplement 40 mmol/L in the first bag of 0.9%
normal saline
D Monitor blood glucose hourly
E Consider urinary catheter

A

C

504
Q

Your consultant has heroically stented 72-year-old Mrs Harris’ coronary arteries at 0200 hours over the weekend. Whilst reviewing her medication, she asks you what you think she could do to prevent this happening again and “spare these lovely nurses all this bother”.
The best course of action would be to:
A Advise her that alcohol is not a cause of heart disease
B Ask her if she smokes and encourage her to cut down on salt intake
C Ask her if she smokes and review her long-term blood pressure and blood
glucose
D Ask her if she smokes, check her lipid profile, review her long-term blood
pressure and blood glucose and assess her pre-morbid mobility
E Encourage her to stop smoking and get out more

A

D

505
Q

A 43-year-old woman presents to her GP with a 3-month history of lethargy and low mood. She noticed that she had gained 2 stones over the past 2 months, despite having a normal appetite. On examination, she looked pale with periorbital puffiness, with a bitemporal hemianopia. Her thyroid gland was not palpable. Her thyroid function tests showed TSH 0.3 (range 0.5–5.5) mU/L , T4 3 (range 4.5–12.5) g/dl.
Which investigation would be most useful in the diagnosis?
A 99m technetium (99mTc)scintigraphy scans
B Magnetic resonance imaging (MRI) of the brain
C Thyroid peroxidise antibodies
D Thyroid-stimulating hormone (TSH) receptor-blocking antibodies
E Ultrasound of the thyroid gland

A

B

506
Q

A 16-year-old boy with known sickle cell disease presents with painful hands bilaterally after being out early in the morning for a run. He is otherwise systemically well.
What should be your immediate management?
A Joint aspiration
B Malaria screen
C Referral to rheumatology D Rehydration and analgesia
E X-rays

A

D

507
Q

A 70-year-old woman has recently been admitted to hospital with a chest infection and mild confusion. Unfortunately no informant was available, hence a full history could not be taken. You are called to see her 2 days after her admission because she was sleeping poorly, complaining of nausea and sweating, and went on to have a seizure.
Which of the following is the most likely reason for this clinical picture?
A Alcohol withdrawal
B Opiate use
C Opiate withdrawal
D Sedative use
E Sedative withdrawal

A

E

508
Q
A 16-year-old girl presents to the GP with an itchy rash. This is located on both forearms and examination reveals evidence of blistering. She has a history of coeliac disease but is otherwise systemically well. 
What is the most likely diagnosis? 
A Acrodermatitis enteropathica 
B Candidiasis
C Dermatitis herpetiformis
D Leukoplakia 
E Linea nigra
A

C

509
Q

A 32-year-old woman undergoes a contrast enhanced CT scan of her abdomen to look for a cause of lower abdominal pain. Although no obvious cause of the pain is found, there is a small cavernous haemangioma of the liver.
What is the next step in management of this lesion?
A Anticoagulation with lifelong aspirin
B Chemotherapy
C Head CT to rule out cerebral haemangiomas
D No treatment necessary
E Surgical resection

A

D

510
Q

A 65-year-old man with a longstanding history of chronic obstructive pulmonary disease was admitted to the hospital with an acute exacerbation. His observations include temperature 37.5°C, pulse rate 120 bpm, blood pressure 140/90 mmHg, respiratory rate 28/min and saturations 84% on room air. An arterial blood gas shows pH 7.31, PaO2 4.6 kPa, PaCO2 8.3 kPa and bicarbonate 25.2 mmol/L.
What is the most appropriate percentage of O2 to be given initially?
A 28%
B 35%
C 40%
D 60%
E 100%

A

A

511
Q

A 43-year-old man has just been diagnosed with polycystic kidney disease and is keen to know about potential risks or associated health problems.
Which of the following is not an extrarenal complication of polycystic kidney disease?
A Bladder diverticuli
B Cerebral aneurysms
C Hepatic cysts
D Mitral valve prolapse
E Pancreatic cysts

A

A

512
Q
A 75-year-old woman presents with weight loss, anorexia and fatigue. She has lost 2 stone in weight in the last 6 months and she has been investigated for presumed dysphagia (she is not an excellent historian), which has found nothing. Her daughter said the first thing she noticed was a reduced walking ability and shoulder pain. You witness her struggle to stand up out of the chair. She also comments brushing the back of her hair is now difficult. She has tender shoulders to palpation. The blood tests reveal an erythrocyte sedimentation rate (ESR) of 60 mm/hour and a CRP of 30 mmol/L. 
What is the most likely diagnosis? 
A Fibromyalgia
B Inclusion body myositis 
C Myasthenia gravis
D Polymyalgia rheumatica 
E Polymyositis
A

D

513
Q

During a routine medical for a work placement a GP discovered that a 33-year-old man who was otherwise fit and healthy had a raised bilirubin level. He otherwise had entirely normal liver function tests and a full blood count. He had no history of foreign travel or drug or alcohol abuse. Repeated blood tests 1 month later showed a similar picture.
What is the most likely diagnosis?
A Crigler–Najjar syndrome B Gilbert’s disease
C Haemolytic anaemia
D Hepatitis B
E Pancreatic cancer

A

B

514
Q
Which of the following drugs is used during cardiac arrest to improve the coronary perfusion pressure? 
A Adrenaline
B Amioderone
C Atropine
D Calcium gluconate
E Magnesium sulphate
A

A

515
Q

A 35-year-old woman presents to the GP with increased weight loss over the past 2 months, with lid retraction, exophthalmos, and double vision. Her thyroid-stimulating hormone (TSH) receptor antibody level is raised.
Which of the following is the first-line treatment in this patient?
A Beta-blockers
B Carbimazole therapy
C Intravenous hydrocortisone
D Radioactive iodine therapy
E Subtotal thyroidectomy

A

B

516
Q

A 26-year-old woman from sub-Saharan Africa presents with two months of haemoptysis and lethargy. In the last month she has lost half a stone in weight and she frequently soaks the bedsheets with sweat. She has a negligible smoking history.
What is the most likely diagnosis?
A Lung cancer
B Lymphoma
C Pneumocystis pneumonia D Pulmonary embolism
E Tuberculosis

A

E

517
Q

A 33-year-old man presents to the emergency department with acute shortness of breath. He appears exhausted and unable to provide a clear history. An arterial blood gas reading shows pH 7.33, PaO2 7.3 kPa, PaCO2 6.9 kPa and bicarbonate 26 mmol/L on room air.
What abnormality do these results represent?
A Metabolic acidosis
B Metabolic alkalosis
C Respiratory acidosis
D Respiratory alkalosis
E Type 1 respiratory failure

A

C

518
Q

A 38-year-old woman, with a history of autoimmune thyroid disease, presents with persistently dry eyes for over a year. She denies any arthralgia, rash, fatigue or any other symptoms. Schirmer’s test reveals <10 mm wetting of filter paper in 5 minutes. Antibodies reveal positive anti-nuclear antibodies, with positive anti-Ro and anti-La antibodies.
Which of the following represents the best course of management?
A Artificial tears
B Hydroxychloroquine
C Steroids
D Surveillance
E Tear duct surgery

A

A

519
Q

A 21-year-old woman is admitted to the emergency department with increasing shortness of breath after taking oral amoxicillin, which was started by her GP for a chest infection. Chest auscultation reveals widespread wheeze bilaterally. Vascular access is secured.
Which of the following management options is NOT appropriate?
A Discontinue administration of the suspected drug
B Give intramuscular adrenaline immediately
C Give intravenous 5% dextrose immediately
D Give intravenous chlorphenamine
E Give intravenous hydrocortisone

A

B

520
Q

A 29-year-old man with known type 2 diabetes mellitus and excess alcohol intake was admitted to the emergency department with reduced consciousness. He is on oral anti-diabetic agents (intermediate and long- acting sulphonylureas) for his diabetes. His friend who brought him said that he had been drinking excessive quantities of alcohol that evening.
Which of the following managements is NOT appropriate in this case?
A Consider CT of the head
B Continue intravenous dextrose after the blood glucose level has returned
to within normal range
C Intravenous dextrose
D Intramuscular glucagon
E Send random blood glucose sample to the laboratory

A

D

521
Q
Which clotting factor is deficient in haemophilia A? 
A VII 
B VIII 
C IX 
D XI
E vWF
A

B

522
Q

A histology report for a specimen taken from the colon of a 24-year-old woman reports “patchy areas of transmural non-caseating granulomatous inflammation”.
What disease process does this suggest?
A Colonic adenocarcinoma B Crohn’s disease
C Irritable bowel syndrome D Tuberculosis
E Ulcerative colitis

A

B

523
Q
Which of the following conditions do not cause a leucoerythroblastic picture on blood film? 
A Chronic myeloid leukaemia
B Megaloblastic anaemia
C Metastatic spread to the bone marrow 
D Myeloma
E Tuberculosis
A

B

524
Q

A 55-year-old woman presents to the emergency department with a 1-month history of shortness of breath and weakness in both lower limbs. She has a 15 pack/year smoking history and has recently been diagnosed with diabetes by her GP. She has no other significant past medical history. On examination, there is reduced air entry on the left lower lung zone. Her chest X-ray shows a mass lesion on the left lower lobe.
An arterial blood gas reading shows: pH 7.53, PaO2 8.6 kPa, PaCO2 5.3 kPa, bicarbonate 30.2 mmol/L.
The blood result shows: Na+ 137mmol/L, K+ 2.3, urea 4.8 mmol/L, creatinine 108 μmol/L.
A CT of the chest reveals a tumour invading the left lung and a percutaneous lung biopsy confirms the diagnosis of lung cancer.
What biochemical abnormalities are shown and what is the most likely histology of the lung cancer?
A Metabolic acidosis, non-small cell carcinoma
B Metabolic alkalosis, small cell carcinoma
C Metabolic alkalosis, non-small cell carcinoma
D Respiratory alkalosis, small cell carcinoma
E Respiratory alkalosis, non-small cell carcinoma

A

B

525
Q

Which of the following cranial nerves would be affected by pathology in the cavernous sinus?
A II, III, IV and VI
B III and V
C III, IV and VI
D III, IV, VI and ophthalmic division of V
E III, IV, VI and V

A

D

526
Q

A 75-year-old man presents with a history of episodes of cognitive impairment and memory deficits. His daughter seems perplexed that at some times he seems entirely normal, however some days he seems muddled and can’t remember anything. He occasionally claims to see faces where there are none. He is a smoker and has chronic obstructive pulmonary disease (COPD), but there is nothing else in his past medical history. On examination he has a slow tremor present in his left thumb when relaxed or distracted, but not present when he is moving his arm. The arms appear to have a rigid tone, more on the left than the right. The rest of the neurological and general examination appears normal.
Which of the following is the most likely cause?
A Depressive pseudodementia
B Frontotemporal dementia C Korsakoff psychosis
D Lewy body dementia
E Vascular dementia

A

D

527
Q

One year after bone marrow transplantation from a matched donor to treat non-Hodgkin’s lymphoma a patient re-presents complaining of widespread skin irritation. The skin is exfoliated in areas and there is a maculopapular rash, while the fingers are becoming sclerotic. He has not been feeling himself for a while with episodes of diarrhoea and recurrent chest infections.
What is the most likely reason for this?
A Acute graft rejection
B Chronic graft-versus-host disease
C Herpes zoster infection
D New-onset scleroderma
E Side effect of immunosuppressants

A

B

528
Q
A 24-year-old woman presents with a 3-month history of vague right upper quadrant pains. She has also noticed an abnormal vaginal discharge since starting a new sexual relationship 4 months ago. On examination there is a hepatic friction rub. 
What is the most likely diagnosis? 
A Fitz-Hugh–Curtis syndrome 
B Hepatitis C
C HIV
D Liver abscess 
E Liver infarct
A

A

529
Q

A 36-year-old man has a 1-month history of a non-productive dry cough. He has also become more short of breath on exertion for the past 2 weeks. He was a known intravenous drug abuser. Chest auscultation is unremarkable. He is referred to the medical assessment unit for a chest X-ray, which shows bilateral symmetrical interstitial infiltrates. A subsequent HIV test is positive.
Which of the following is the most likely causative agent?
A Aspergillus fumigates
B Cytomegalovirus
C Histoplasma capsulatum D Pneumocystis jirovecii
E Staphyloccocus aureus

A

D

530
Q
A 28-year-old woman with known neurofibromatosis type I presents to her GP with palpitations and recurrent headaches that are not relieved by paracetamol. She also feels dizzy and becomes sweaty during these episodes. Her blood pressure is found to be 220/120 mmHg. 
What is the most likely diagnosis? 
A Acromegaly
B Carcinoid syndrome 
C Cushing’s disease
D Thyrotoxicosis
E Phaeochromocytoma
A

E

531
Q

A 28-year-old Afro-Caribbean pregnant woman presents with a 2-month history of tingling in her feet and difficulty walking, which has progressed so that she is barely able to walk. She came to Britain 3 years ago from West Africa and has struggled to find work since, has a 2-year-old son, and lives in a flat with her boyfriend. She has been pregnant for 5 months. She has no significant past medical history other than various diarrhoeal illnesses when living in Africa – in particular she denies HIV. On examination, she looks thin and the conjunctiva are pale. There are no cranial nerve abnormalities, or abnormalities on the arms. In the legs, there is an increase in tone in the legs, with a marked weakness to about grade 3 or 4 throughout both legs, which is worse in the ankles. Knee jerks appear brisk, although the ankle jerks are hard to elicit and appear absent. Plantar reflexes are upgoing. There is a loss of light touch to just above the ankles, and there is absent proprioception and vibration sense in the big toe joints.
Which of the following is the most likely diagnosis?
A Diabetic polyneuropathy
B Guillain–Barré syndrome
C HIV-related polyneuropathy
D Subacute combined degeneration of the cord
E Tabes dorsalis

A

D

532
Q

A 60-year-old man presents with a 1.5 cm raised pigmented lesion on his right arm. He is unsure how long it has been there.
Which feature on dermoscopy would suggest a malignant melanoma rather than a benign naevus or a pigmented seborrhoeic keratosis?
A Erythema of border
B Granular surface
C Irregular pigment network
D Stuck-on appearance
E Telangectasia

A

C

533
Q

A 60-year-old Caucasian woman who recently moved from Zimbabwe presents with a growing pigmented lesion about 1 cm across with an irregular border and a raised, darker patch within. You suspect melanoma and excise the lesion.
Which of the following features of the tumour is the best prognosis indicator?
A Colour
B Diameter
C Grade
D Invasive depth
E Weight

A

D

534
Q

Whilst on call, you are asked to review an ECG. It shows a long QT interval followed by a burst of QRS complexes at 300 bpm with an overlying waxing and waning R-wave amplitude, after which the ECG reverts back to its previous pattern. The patient feels well.
You conclude that this was an episode of torsade de pointes and should now:
A Advise the nurses to perform hourly observations
B Check the drug chart for possible causes
C Check the fluid balance chart
D Give anti-arrhythmics
E Send out a crash call

A

B

535
Q

A 75-year-old man presents with a 5-day history of cough productive of green sputum, mild right-sided chest pain, worsening breathlessness, and in the last day or so, confusion. He is previously fit and well with no background of dementia. He was in hospital 2 months ago for an elective hernia repair. On examination he has coarse crepitations at the right lung base, he has a respiratory rate of 34/min, a temperature of 38°C, and has an abbreviated mental test score of 6/10. Chest X-ray shows right lower zone consolidation and a small degree of blunting of the right costophrenic angle.
Which of the following is the likely diagnosis?
A Aspiration pneumonia
B Empyema
C Hospital acquired pneumonia
D Mild or moderate community-acquired pneumonia
E Severe community-acquired pneumonia

A

E

536
Q

A 16-year-old girl is seen by her general practitioner. She is lethargic and “not feeling her usual self ” according to her parents. On examination, she has a stiff neck and a non-blanching purpuric rash on her trunk and legs.
Which of the following is the best course of action?
A Expectant management until there is deterioration
B Intramuscular benzylpenicillin and call ambulance
C Intramuscular benzylpenicillin and review in 12 hours
D Intravenous lorazepam and call ambulance
E Oral penciliin and review in 24 hours

A

B

537
Q

A 53-year-old retired truck driver presents to the general practitioner with a 3-month history of productive cough. He also complains of gradual onset of shortness of breath on exertion. He has to stop for breath after walking about 100 m on level ground. He is a heavy smoker, but is otherwise fit and well.
Which of the following investigations would be most useful in establishing the cause of his symptoms?
A Chest X-ray
B Electrocardiogram
C Peak flow meter
D Spirometry
E Sputum cytology

A

D

538
Q
A 54-year-old woman presents to her GP complaining of repeated incidents of burning central chest pain. It mainly occurs when she lies down to go to bed at night. She is overweight with a body mass index of 40. She uses GTN occasionally but it doesn’t always relieve her symptoms. She doesn’t report any shortness of breath or palpitations and examination is unremarkable. 
What is the most likely diagnosis? 
A Angina
B Gastro-oesophageal reflux disease 
C Myocardial infarction
D Pancreatitis
E Sleep apnoea
A

B

539
Q

You are called to see a 92-year-old woman on the ward. The patient has been recovering from an exacerbation of heart failure. On arrival, the patient is on 15 L/min oxygen and an intravenous drip. You find the patient’s oxygen saturation is reading 88% on pulse oximetry. She is having prolonged bouts of coughing, is reported by the sister as seeming physically weak for her, and is delirious.
Which of the following would be the most appropriate initial course of action?
A Check her fluid balance chart
B Check she is on regular paracetamol
C Keep checking the pulse oximeter hourly for deterioration
D Request a computed tomography (CT) pulmonary angiography
E Start her on antibiotics for hospital-acquired pneumonia

A

A

540
Q

A 46-year-old woman presents to the emergency department in a reduced state of consciousness. Her blood glucose level was 9 mmol/L. On examination, her body temperature was 33°C. She has non-pitting oedema on the skin of her hands, feet and eyelids. No obvious head injury is noticed. She has a thyroidectomy scar on her neck.
Bearing in mind the likely diagnosis, which of these is NOT appropriate in the initial management?
A High-flow oxygen if the patient is cyanosed
B Intravenous hydrocortisone
C Intravenous lorazepam
D Parenteral triiodothyronine, given slowly
E Warm blanket

A

C

541
Q

A 45-year-old man presents to his GP with lethargy and polyuria for the past month. He has a strong family history of diabetes mellitus. A provisional diagnosis of diabetes mellitus is made.
Which of the following investigations is NOT useful in establishing the diagnosis?
A Fasting glucose level
B HbA1C level
C Oral glucose tolerance test
D Random glucose level
E Urine ketones

A

B

542
Q

An obese 48-year-old woman presented direct from work with fatigue and shortness of breath. She has been started on treatment for acute coronary syndrome but her oxygen saturation is now dropping and on review, you feel this is iatrogenic.
Which medication is most likely to be at fault?
A Aspirin
B Atenolol
C Clopidogrel
D Glyceryl trinitrate
E Heparin

A

B

543
Q

A 32-year-old woman is started on methotrexate for her newly diagnosed rheumatoid arthritis.
Which of the following is NOT true regarding side effects and contraindications of methotrexate?
A A baseline chest X-ray should be taken as pulmonary fibrosis is a potential side effect
B Alcohol should be absolutely avoided whilst taking methotrexate
C Full blood count should be monitored as neutropenia and
myelosuppression are potential side effects
D Liver function tests should be monitored to watch for hepatic fibrosis
E Pregnancy should be avoided and if considering having a baby,
methotrexate should be switched to an alternative disease-modifying anti-rheumatic drug

A

B

544
Q

A 55-year-old man presents to the hospital with a 4-month history of headaches, which usually occur in the mornings. These are becoming more frequent and severe.
Which of the following would suggest increased intracranial pressure?
A Jaw claudication
B Kernig’s sign
C Low opening pressure on lumbar puncture
D Relief of symptoms when lying down
E Transient bilateral visual loss

A

E

545
Q

A 53-year-old man is seen in the medical assessment unit with lethargy, nausea, polyuria and polydipsia. He also has a weight loss of 2 stones over the period of 1 month. He smokes 40 cigarettes per day. A chest X-ray shows a lung mass in right lower lobe. His blood tests show a corrected calcium level of 3.1 mmol/L. Other causes of hypercalcaemia were excluded.
Which of the following cell types of bronchial carcinoma is most likely in this case?
A Adenocarcinoma
B Sarcomatoid carcinoma
C Small cell carcinoma
D Squamous cell carcinoma E Large cell carcinoma

A

D

546
Q
A 44-year-old woman who works in an office presents with a 2-month history of pains in her right hand and thumb weakness. The pains would shoot into her first three fingers, and initially she thought she was using bad typing technique, however the pains progressed despite ergonomic keyboards and then she became weak in her thumb. She also says her first three fingers feel numb. On examination there is wasting of the thenar eminence, weakness of thumb abduction, and reduced sensation in the thumb, index and middle finger and the lateral half of her fourth finger. 
Which nerve or nerve root is affected? 
A C6
B C8
C Median nerve 
D Radial nerve 
E Ulnar nerve
A

C

547
Q

A 55-year-old woman presents to clinic with joint pain. She is concerned that it might be rheumatoid arthritis.
Which of the following clinical features is NOT used in the American College of Rheumatology diagnostic criteria to diagnose rheumatoid arthritis and differentiate it from other causes of arthritis?
A Morning stiffness for 1 or more hours for more than 6 weeks
B Simultaneous symmetrical joint swelling for more than 6 weeks
C Three or more joints affected for more than 6 weeks
D Two hand joints (either metacarpophalangeal (MCP) or proximal
interphalangeal (PIP)) involved for more than 6 weeks
E Ulnar deviation of one or more fingers for more than 6 weeks

A

E

548
Q
A new tumour marker is being tested for use in cholangiocarcinomas. In a trial 40 patients have been tested. The trial produces 10 positive results and 30 negative results. Of the 10 positive results, 5 of them are false positives. Of the 30 negative results, 5 of them are false negatives. 
What is the specificity of this test? 
A 17% 
B 50% 
C 70% 
D 83% 
E 100%
A

D

549
Q

A 43-year-old woman presents with a 1-week history of loin pain, suprapubic pain and dysuria. On examination she looks unwell, has a pulse of 110 bpm, blood pressure of 120/80 mmHg, respiratory rate of 24/min and a temperature of 38.5°C. Bloods are taken for routine tests and blood cultures, a catheter passed, and empirical antibiotics started. Later that day, blood cultures come back showing a Gram-negative rod. You note that in the last 3 hours she has only passed 30 ml of urine and she is starting to show signs of confusion. Observations remain as they were earlier.
Which of the following terms best describes the patient’s physiological status?
A Sepsis
B Septic shock
C Septicaemia
D Severe sepsis
E Systemic inflammatory response syndrome

A

D

550
Q

A 38-year-old man presents with fatigue, swelling of the eyelids and legs, and frothy urine. His bloods appear normal except for an albumin level of 26 g/L (his albumin was normal on blood tests taken 1 year ago). You suspect nephrotic syndrome.
How would you confirm the diagnosis?
A Proteinuria on urine dipstick
B >1 g of protein lost in the urine over 24 hours
C >1 g of protein lost in the urine over 72 hours
D >3.5 g of protein lost in the urine over 24 hours
E >3.5 g of protein lost in the urine over 72 hours

A

D

551
Q

A 16-year-old female attends her GP complaining of a persistent lack of periods. She has a short stature, low-set ears and broad square-shaped chest. She has a short fourth metacarpal. There is radio-radial delay on examination.
What would the most appropriate diagnostic test be in this case?
A Growth hormone level
B Karyotype analysis
C Magnetic resonance angiography
D Thyroid function test
E Ultrasound of the ovaries

A

B

552
Q

Which of the following investigation findings would you NOT expect in myasthenia gravis?
A Decrementing response on electromyography
B Hyperplasia of the thymus gland or thymoma on computed tomography
(CT) of the chest
C Positive acetylcholine receptor antibodies
D Rimmed vacuoles on muscle biopsy
E Tensilon test reduces weakness

A

D

553
Q

A 78-year-old man admitted with severe sepsis secondary to a chest infection develops worsening acute renal failure. An arterial blood gas reading, other than showing metabolic acidosis, also reveals a potassium level of 7.2 mmol/L. You ask the nurse for an electrocardiogram (ECG).
Which of the following changes is NOT associated with hyperkalaemia?
A Broadening of the QRS complex
B Deepening Q-waves
C Flattening of the P-waves
D Tented T-waves
E Ventricular fibrillation

A

B

554
Q

A 75-year-old man with known heart failure is likely to have had a myocardial infarction. He is now haemodynamically stable on the ward. A 12-hour troponin level is raised.
Which investigation will confirm your diagnosis?
A 24-hour ECG
B Exercise ECG
C Troponin level at 48 hours
D Nuclear studies of the myocardium
E None of the above

A

D

555
Q

A 45-year-old woman presents with a 5-month history of worsening generalised itching. There is nothing remarkable in the woman’s medical history and there is no sign of any skin lesion or inflammatory disorder of the skin. You take some blood tests for a pruritus screen.
Which of the following tests would NOT reveal a cause for pruritus and thus is not indicated?
A Electrolytes
B Full blood count
C Iron studies
D Thyroid function tests
E Urea

A

A

556
Q

Which of the following symptoms would NOT be caused by complex partial seizures?
A Déjà-vu
B Fumbling or rubbing, slightly odd limb movements
C Shooting/electric shock-like pains in the limbs
D Smell and/or taste hallucinations
E Subtly impaired conscious level

A

C

557
Q
A 70-year-old woman with a background of bowel cancer, which was resected 2 years ago, presents with a 3-week history of back pain, progressive clumsiness and heaviness of the legs with paraesthesia. On examination, the arms are normal, but there is weakness throughout the legs, brisk reflexes, upgoing plantars and diminished sensation bilaterally up to the belly button. 
Where is the lesion likely to be? 
A C5 
B L2 
C T8 
D T10 
E T12
A

D

558
Q

A 50-year-old man being treated for inoperable gastric cancer with chemotherapy presents with a sore throat and cough. On examination, he has a temperature of 38.2°C.
Which of the following statements indicates the most severe life-threatening illness?
A Cough of >2 weeks duration
B Neutrophils >15 × 109/L
C Neutrophils <0.5 × 109/L
D Severe vomiting
E Weight loss

A

C

559
Q

A 22-year-old man with a family history of psoriasis attends clinic complaining of lower back and buttock pains on and off for the past 6 months. This is worse in the mornings and associated with stiffness.
Which of the following findings on magnetic resonance imaging (MRI) would be first to appear and thus diagnostic of early ankylosing spondylitis?
A Bamboo spine
B Fusion of the sacroiliac joints
C Osteophyte formation
D Sacroiliitis
E Syndesmophyte formation

A

D

560
Q
An 80-year-old woman is admitted to hospital with a right neck of femur fracture. She was found to become drowsy with a respiratory rate of 8/ min after a dose of intravenous morphine. There are pinpoint pupils on examination 
What should you do? 
A Give intravenous flumazenil
B Give intravenous magnesium sulphate 
C Give intravenous N-acetylcysteine
D Give intravenous naloxone
E Give intravenous sodium bicarbonate
A

D

561
Q

Which of the following statements is FALSE about squamous cell carcinoma?
A Curettage is the first line of management
B Sun exposure is a risk factor
C They are ill-defined lesions and often ulcerate
D They are more common in kidney transplant patients
E They can metastasise to lymph nodes

A

A

562
Q

A 28-year-old pregnant woman has a routine mid-stream urine sample taken for dipstick testing and microscopy culture and sensitivities. She is asymptomatic.
Which of the following results does not warrant a course of antibiotics?
A All of the results below require treatment
B Leukocyte esterase negative, nitrites negative, but >105/ml Gram-negative
rods on microscopy
C Leukocyte esterase negative, nitrites positive
D Leukocyte esterase positive, nitrites negative
E Leukocyte esterase positive, nitrites positive

A

A

563
Q

A 35-year-old homeless man diagnosed with new tuberculosis 1 month previously and started on treatment presents with a loss of colour vision.
Which of the following medications is responsible?
A Ethambutol
B Isoniazid
C Pyrazinamide
D Rifampicin
E Streptomycin

A

A

564
Q

A 64-year-old woman with known heart disease presents to her GP with weight gain, weakness and bilateral leg swelling. On examination she has slow-relaxing elbow reflexes. Thyroid function tests show an elevated TSH and low serum T4 level.
Which of the following medications might be contributing to her symptoms?
A Amiodarone
B Fluoxetine
C Lisinopril
D Prednisolone
E Simvastatin

A

A

565
Q
A 52-year-old woman presents with a 2-year history of painful fingers. In cold weather her fingers turn pale and then through blue to red, at which point she experiences severe pain in the fingers. Over the last few months, she has had some retrosternal pain on eating and experiences some reflux. On examination the fingers appear to be swollen and tense, with some dilated nail-fold capillary loops and a small crusted ulcer is present on a fingertip. 
What is the most likely diagnosis? 
A Dermatomyositis
B Limited cutaneous systemic sclerosis 
C Primary Raynaud phenomenon
D Sarcoid
E Systemic lupus erythematosus
A

B

566
Q

A 26-year-old woman presents to the GP with a history of palpitations. She also feels hot most of the time and has noticed some weight loss. On examination, she has a “startled” look with lid lag and lid retraction. There is a palpable neck lump on examination that moves up on swallowing.
Which of the following investigations will be LEAST appropriate?
A 24-hour electrocardiogram
B Technetium-99 scintigraphy C Fine-needle aspiration
D Thyroid function tests
E TSH-receptor antibodies

A

A

567
Q
A 15-year-old boy has recently started coughing up blood. He has had a non-productive cough for about 6 months with intermittent haemoptysis. Otherwise he is normally fit and well. On examination, finger clubbing is noted. Chest examination reveals basal crepitations with reduced air entry. Pulmonary function tests reveal a raised transfer factor. A blood test shows low haemoglobin, low mean cell volume, low iron and low ferritin. Autoantibody profiling is negative. 
What is the most likely diagnosis? 
A Goodpasture syndrome
B Idiopathic pulmonary haemosiderosis 
C Microscopic polyarteritis
D Systemic lupus erythromatosus
E Wegener granulomatosis
A

B

568
Q

A 64-year-old woman presents with a 3-week history of progressive painless haematuria, hypertension, breathlessness and haemoptysis. There is no evi- dence of epistaxis. Her blood pressure is now 190/110 mmHg, and her chest X-ray shows haemorrhagic pulmonary infiltrates in the lower zones. Her bloods reveal a creatinine level of 340 μmol/L and a raised C-reactive protein.
Which of the following antibody tests would confirm the likely diagnosis?
A Anti-double-stranded DNA antibodies (anti-dsDNA)
B Anti-glomerular basement membrane antibodies (anti-GBM)
C Anti-ribonucleoprotein antibodies (anti-RNP)
D Cytoplasmic anti-neutrophil cytoplasm antibodies (cANCA)
E Perinuclear anti-neutrophil cytoplasm antibodies (pANCA)

A

B

569
Q

breathlessness on exertion. He has a past medical history of hypertension, diabetes mellitus and chronic obstructive pulmonary disease. He has also complained of weight loss for the past 6 months. On examination, he was alert with a respiratory rate of 25/min and an oxygen saturation reading of 86% in air. He has distended neck veins, facial oedema and bilateral arm swelling.
Which of the following management options is NOT appropriate initially?
A Consider intravenous dexamethasone and radiotherapy
B Consider intubation if in respiratory compromise
C Consider percutaneous endovascular stenting
D Give oxygen to maintain oxygen saturation at 88–92%
E Request a chest X-ray

A

A

570
Q
Which of the following is NOT a cause for atrial fibrillation? 
A Abdominal aortic aneurysm 
B Acute infection
C Binge drinking
D Hypertension 
E Idiopathic
A

A

571
Q
Which of the following is the universal recipient in blood transfusion? 
A A+ 
B AB– 
C AB+ 
D B– 
EO
A

C

572
Q

A 43-year-old woman presents to the GP with generalised weakness. She notices that she has been gaining weight for the past 3 months despite a normal diet. She has a past medical history of asthma, which is poorly controlled, and she was admitted to the hospital twice last year due to exacerbations. On examination, her blood pressure is 160/90 mmHg. She has bruises on both shins, with wasting and weakness of her proximal thigh muscles.
Which of the following is the most likely diagnosis?
A Addison’s disease
B Cushing’s syndrome
C Hypothyroidism
D Hyperthyroidism
E Hypercalcaemia

A

B

573
Q

A 25-year-old female student, who is normally fit and well, presents to the emergency department 3 weeks before her final exams. She is drowsy and confused. On examination, she has a hepatic flap and is jaundiced. There is also tender hepatomegaly.
Which of the following is a pertinent first-line investigation?
A Ferritin levels
B Gamma-glutamyl transferase (GGT)
C Hepatitis screen
D Lumbar puncture
E Paracetamol levels

A

E

574
Q

An 83-year-old woman who lives in a care home presents with a 2-week history of itching. A few of the other residents have since complained of similar symptoms. You suspect scabies.
Which of the following statements about scabies is FALSE?
A All clothes, bedsheets and any upholstery contacted by the patient should be washed thoroughly
B Burrows are often in web spaces of the digits, wrists, elbows, and male genitals
C Diagnosis is by examining scraped-off burrows under a microscope
D Treatment is application of topical malathion or permethrin
E Treatment with topical agents can be one-off or can ideally be repeated 7
days later

A

A

575
Q

A 35-year-old woman presents with a history of weakness and paraesthesia for 2 weeks. It started with some unusual sensations in her feet and hands, then some days later she noticed her feet were dragging as she walked. Then over the next week she became unable to get out of her chair and developed arm weakness. She says she had some diarrhoea and abdominal cramps around 4 weeks ago. On examination there is weakness throughout the legs and arms in a patchy distribution varying between grades 2 and 4, reflexes are diminished throughout, and there is reduced sensation in the feet up to mid-shin level.
Which of the following investigation findings is suggestive of the likely diagnosis?
A Multifocal decreased motor conduction block only
B Multifocal decreased motor conduction speed with/without conduction
block
C Oligoclonal bands on lumbar puncture
D Positive antiviral or antibacterial antibodies to campylobacter
E Raised lymphocytes on lumbar puncture

A

B

576
Q

A 45-year-old woman presents with malaise, joint pains and jaundice. A hepatitis screen is sent and returns with the following results:
Hepatitis B surface antigen (HBsAg) – POSITIVE Hepatitis B ‘e’ antigen (HBeAg) – POSITIVE Hepatitis B core antibody (HBcAb) – POSITIVE
Which of the following is the most likely diagnosis?
A Acute hepatitis B infection
B Chronic hepatitis B infection
C Hepatitis B immunity following previous infection
D Hepatitis B immunity following previous vaccination
E No hepatitis B infection

A

A

577
Q

A lumbar puncture in a patient with headache shows the following results:
Cells
Predominant cell type Glucose
Protein
460/mm3 (normal range <4/mm3) Lymphocytes
1.3 mmol/L (normal range 2–9 mmol/L) 1200 mg/L (normal range 200–400 mg/L)

India ink staining after the culture is negative
Which one of the following are these findings consistent with?
A Bacterial meningitis
B Cryptococcal meningitis C Malignant meningitis
D Tubercular meningitis
E Viral meningitis

A

D

578
Q
Which of the following medications used in the management of severe hyperkalaemia does not actually lower serum potassium, but rather is cardioprotective by acting as a membrane stabiliser? 
A Calcium gluconate
B Calcium resonium
C Dextrose 50% with Actrapid insulin 
D Furosemide
E Salbutamol
A

A

579
Q

A 60-year-old man with known type 1 diabetes mellitus for the past 20 years has come to his GP for his annual diabetic check. His blood pressure is 160/90 mmHg today. He is found to have a urinary albumin/creatinine ratio (ACR) of >2.5 mg/mmol, suggestive of microalbuminuria.
Which of the following drugs is proven to reduce microalbuminuria?
A ACE inhibitors
B Alpha-blockers
C Beta-blockers
D Calcium channel blockers E Thiazide

A

A

580
Q

An 18-year-old female is newly diagnosed with coeliac disease following a prolonged history of diarrhoea, malaise and weight loss.
Which of the following is the most suitable initial management?
A Exclusion of gluten from the diet
B Exclusion of lactose from diet
C Immunosuppressive agents
D Non-steroidal anti-inflammatories
E Stool-forming drugs such as loperamide

A

A

581
Q

Which of the following patients is most likely to have a cerebellopontine angle tumour?
A A 30-year-old woman with unilateral lower motor neuron facial weakness progressing over days and covered in café au lait spots
B A40-year-oldwomanwithinsidiousonsetunilateraldeafness,onWeber’s testing sound is louder ipsilaterally
C A 50-year-old man with bilateral facial weakness, nasal voice and swallowing difficulty progressing over 3 months
D A 50-year-old woman with insidious-onset unilateral deafness, on Weber’s testing sound is louder contralaterally, and with some ipsilateral reduced facial sensation
E A 65-year-old man with insidious-onset mild bilateral sensorineural deafness and no other neurological abnormality

A

D

582
Q

An 85-year-old retired soldier defends a young woman from being mugged, only to find himself stabbed in the left side of the chest.
On arrival at the emergency department, which of the following features would be most relevant?
A Beck’s triad
B Kussmaul breathing
C Kussmaul’s sign
D Virchow’s triad
E Witnessed Stokes–Adams attack

A

A

583
Q

A 68-year-old man with chronic hepatitis B virus infection presents with a 1-month history of weight loss, malaise and right upper quadrant discomfort. On examination he has mild ascites and appears cachectic.
What is the most likely diagnosis?
A Cholangiocarcinoma
B Cirrhosis of the liver
C Hepatocellular carcinoma D Reactivation of hepatitis B E Right-heart failure

A

C

584
Q

You have been asked to explain the causes of hypertension to a patient recently diagnosed with Conn’s syndrome.
Which of the following is an incorrect statement that should NOT feature in your explanation?
A Angiotensin I is converted to angiotensin II mainly in the lungs
B Angiotensin II causes blood pressure to rise but will be low in this patient
C Conn’s syndrome causes excess aldosterone, the vasoconstriction from
which leads to hypertension
D Hypertensive patients are often vulnerable to hypokalaemia as a side
effect of their medication
E The patient is vulnerable to hypokalaemia as they are excreting too much
in their urine

A

C

585
Q
An 88-year-old man presents with a month-long history of difficulty swallowing solids. He has noticed his clothes are looser on him now and he has a significant smoking history. 
What is the most likely diagnosis? 
A Achalasia
B Lung carcinoma
C Motor neurone disease 
D Oesophageal Candida
E Oesophageal carcinoma
A

E

586
Q

A 71-year-old woman is admitted with 5 days of cough productive of purulent sputum and fevers, which has not improved with oral antibiotics prescribed by the GP. She is now confused and dehydrated with some renal impairment evident from the bloods (urea 9 mmol/L, creatinine 140 μmol/L). Chest X-ray now shows bilateral consolidation.
Urinary antigen testing should be done looking for which causative pathogen?
A Escherichia coli
B Haemophilus influenza
C Legionella spp.
D Mycoplasma pneumoniae
E Viruses

A

C

587
Q
A 40-year-old man presents to the GP with a cough. His cough has been present for about 2 months, and it has become worse over the past few weeks. Besides that, he also finds himself easily becoming short of breath when he exercises. The cough is dry and non-productive. He does not produce any blood-tinged sputum. He has not lost weight, never smokes and has no other significant past medical history. 
What is the most likely diagnosis? 
A Asthma
B Chronic obstructive pulmonary disease 
C Interstitial pulmonary fibrosis
D Lung cancer
E Pneumonia
A

A

588
Q

A 40-year-old woman presents to the GP with generalised weakness and weight loss. She noticed that she had become more tanned despite not travelling abroad. On examination, there were bruises on both of her shins. A provisional diagnosis of Cushing’s syndrome is made.
Which of the following test is NOT helpful in this case?
A 48-hour high-dose dexamethasone suppression test
B Computed tomography (CT) of the adrenals
C CT of the chest
D Overnight low-dose dexamethasone suppression test
E Plasma ACTH

A

B

589
Q

A 55-year-old man presents with 6 months of progressive weakness in his arms and legs. On further questioning he reveals that he has choked quite a few times recently on swallowing liquids. He is a smoker but has no significant past medical history. He is British and has not travelled widely. On examination of the cranial nerves, his palate does not rise much on the left on saying “aah”, and does not rise on the right at all. On examination of his arms, there is wasting of the dorsal interosseous muscles and fasciculations in the right forearm and left deltoid, there is diffuse weakness (grade 4), the reflexes are present or brisk, and there is no sensory deficit. On examination of the legs, there are fasciculations seen in the thighs, some proximal weakness on the left, and brisk knee jerks and upgoing plantar reflexes. Again there is no sensory deficit.
Which of the following diagnoses is most likely?
A Cervical spondylosis
B HIV-related motor neurone disease-like syndrome
C Motor neurone disease
D Multiple sclerosis
E Syringomyelia and syringobulbia

A

C

590
Q

A 43-year-old woman presents with a history of 5 months of symmetrical hand pain and swelling with morning stiffness. You suspect rheumatoid arthritis.
Which of the following tests is the most sensitive and specific for rheumatoid arthritis?
A ANCA
B Anti-CCP antibodies
C Anti-centromere antibodies
D Rheumatoid factor
E Soft tissue swelling and bony erosions on X-ray of the hands

A

B

591
Q
A 55-year-old woman presents with a 4-month history of an erythematous rash over her cheeks, nose and forehead. There appears to be multiple small papules and a few pustules on a background of erythematous skin. 
What is the most likely diagnosis? 
A Acne vulgaris
B Dermatomyositis
C Eczema
D Rosacea
E Systemic lupus erythematosus
A

D

592
Q

You are covering the wards overnight. You are bleeped by the critical care ward. An experienced nurse informs you that a recent admission, an 83-year-old woman, has second-degree heart block. You ask her to fax the ECG to you as you are so busy.
What is meant by second-degree heart block?
A A bradycardic QRS complex is dissociated from the P-wave
B Occasional P-waves are failing to conduct through and trigger QRS
complexes
C There is a prolonged P-R interval
D There is ST segment elevation throughout the chest leads
E The QRS complexes are irregularly irregular

A

B

593
Q

A 32-year-old man is brought to the emergency department whilst fitting. His airway has been secured using a Guedel airway. He has oxygen saturations above 96% on 15 L of oxygen and a capillary refill time of 2 seconds. Intravenous access has been gained.
Which of the following is the most appropriate next step?
A Jaw thrust
B Intravenous benzylpenicillin
C Intravenous lorazepam
D Intravenous phenytoin
E Rectal diazepam

A

C

594
Q

A pleasantly confused 92-year-old woman with a history of cardiac disease is an inpatient on your ward. You are called by a nurse as her observations are peculiar and she is not feeling well. Her heart rate is 150 bpm and her blood pressure 110/60 mmHg. You inspect her electrocardiogram (ECG) and diagnose a supraventricular tachycardia.
A sensible next step would be to:
A Administer adenosine
B Administer digoxin
C Book an echocardiogram for tomorrow
D Defibrillate
E Print off the ECG and put it in the notes – no further action is required

A

A

595
Q

A 68-year-old man presents to hospital with a 2-week history of confusion and he is found to have a pneumonia, for which he is treated. You note his plasma sodium is 119 mmol/L.
Which of the following investigation findings is consistent with a diagnosis of “syndrome of inappropriate anti-diuretic hormone secretion”?
(Normal values: urine osmolality 50–1200 mOsm/kg, plasma osmolality 280–300 mOsm/kg)
A Urine osmolality 45 mOsm/kg, plasma osmolality 250 mOsm/kg
B Urine osmolality 45 mOsm/kg, plasma osmolality 290 mOsm/kg
C Urine osmolality 45 mOsm/kg, plasma osmolality 305 mOsm/kg
D Urine osmolality 800 mOsm/kg, plasma osmolality 250 mOsm/kg
E Urine osmolality 800 mOsm/kg, plasma osmolality 305 mOsm/kg

A

D

596
Q

A 26-year-old woman presents to the GP with right-sided pleuritic chest pain and increasing exertional dyspnoea that has lasted for about 2 weeks. She also complains of fatigue, malaise, hair loss and arthralgia that has lasted for 3 months. On examination, she has a rash over her cheeks, and auscultation of the chest reveals a friction rub and dullness to percussion over the right lower zone. Autoantibody blood tests demonstrate a profile consistent with systemic lupus erythematosus (SLE).
What is the most likely pulmonary manifestation of SLE in this case?
A Acute pleuritis
B Acute pneumonitis
C Interstitial lung disease
D Pneumonia
E Pulmonary haemorrhage

A

A

597
Q

A 48-year-old man presents to the emergency department in a reduced state of consciousness. He was brought in by his son who says he has been more confused over the past few days. A collateral history suggests 2 weeks of polyuria and polydipsia. There was no history of head injury, trauma or ingestion of illegal drugs. He has no other significant past medical history. He has a 21 unit/week alcohol history. On examination, he is unresponsive to pain. The liver edge is felt on abdominal examination. His blood tests show:
Na+ K+
Glucose
Serum osmolality Urea
Creatinine
168 mmol/L 3.8 mmol/L 68 mmol/L 350 mmol/kg 14.3 mmol/L 203 μmol/L
A urine dipstick test revealed glycosuria but no ketones
What is the most likely differential diagnosis?
A Diabetic ketoacidosis
B Encephalitis
C Hepatic encephalopathy
D Hyperosmolar non-ketotic diabetic coma
E Uraemia

A

D

598
Q

A 24-year-old man diagnosed with Wilson’s disease was started on penicillamine. He calls his GP 1 month later to report that he has a temperature and a sore throat and has noticed marked bruising in the last week.
What advice should the GP give at this time?
A Advise the patient this is likely to be flu
B Ask the patient to come in for an urgent blood test
C Ask the patient to take some paracetamol and book an appointment the
following week
D Increase the dose of penicillamine
E Reconsider the diagnosis of Wilson’s disease

A

B

599
Q
Which of the following radiographic changes are NOT consistent with a diagnosis of osteoarthritis? 
A Loss of joint space
B Osteophytes
C Punched-out periarticular erosions 
D Subarticular sclerosis
E Subchondral cysts
A

C

600
Q

A 22-year-old male medical student returns from travelling across Southeast Asia with a 48-hour history of watery diarrhoea and abdominal pain. He is otherwise well in himself.
What is the commonest organism for a short history of traveller’s diarrhoea?
A Ameobic dysentery
B Escherichia coli
C Listeria
D Malaria
E Salmonella

A

B

601
Q

A 60-year-old man develops shortness of breath and a cough productive of purulent sputum three days after a right hemicolectomy. An arterial blood gas shows pH 7.42, pO2 7.1, pCO2 5.2, bicarbonate 25.
Reference ranges: pO2 >11.0 kPa, pCO2 4.6−6.0 kPa, bicarbonate 22−28 mmol/L, pH 7.35−7.45.
What blood gas picture does this represent?
A. Metabolic alkalosis
B. Respiratory acidosis
C. Respiratory alkalosis
D. Type I respiratory failure
E. Type II respiratory failure

A

D

602
Q

A 72-year-old woman is brought into hospital with profuse diarrhoea, abdominal pain and fever, which she has suffered for the last two days. On examination, she has generalized abdominal tenderness. Her observa- tions include heart rate 108/min, blood pressure 110/64 mmHg and tem- perature 38.2°C. Abdominal and erect chest X-rays are unremarkable. Her son tells you she was in hospital two weeks ago with a chest infection.
Which of the following would be the most appropriate treatment in the first instance?
A. Intravenous co-amoxiclav
B. Intravenous vancomycin
C. Oral metronidazole
D. Oral rehydration
E. Subtotal colectomy

A

C

603
Q

A 45-year-old man has walked into the emergency department follow- ing his involvement in a road traffic collision. On arrival at the hospital he is anxious, but otherwise fine. Later, he suddenly becomes faint. He is taken into the resuscitation room where he is found to have a heart rate
of 46/min and a blood pressure of 80/48 mmHg. Primary examination is unremarkable and the patient has warm peripheries.
Which of the following is the most likely cause of his symptoms?
A. Cardiogenic shock
B. Haemorrhagic shock
C. Hypovolaemic shock
D. Neurogenic shock
E. Spinal shock

A

D

604
Q

A 53-year-old man presents to the GP with a deep, painful ulcer over the big toe. He gives a 3-month history of severe calf pain on walking which is only eased on resting. Examination shows cool peripheries with reduced distal pulses.
Which ulcer does the patient most likely have?
A. Arterial ulcer
B. Curling ulcer
C. Marjolin ulcer
D. Neuropathic ulcer
E. Venous ulcer

A

A

605
Q

A mother brings her 5-week-old son to the paediatric outpatient clinic. She is concerned as he has been having episodes of forceful vomiting after feeds for the last 2 weeks. She says her son always seems hungry and now is beginning to appear lethargic. Examination of the child reveals mild dehydration and the presence of a smooth, firm, non-tender mass in the right upper quadrant of the abdomen. Blood tests are sent.
What biochemical abnormalities would you expect to find?
A. Hyperchloraemic, respiratory alkalosis
B. Hypernatraemic, hyperkalaemic, metabolic alkalosis C. Hypochloraemic, hyperkalaemic, metabolic alkalosis
D. Hypochloraemic, hypokalaemic, metabolic alkalosis E. Hyponatraemic, metabolic acidosis

A

D

606
Q

A 45-year-old woman is due to have an elective cholecystectomy following a recent bout of acute cholecystitis (which has fully resolved). She has a past medical history of high blood pressure, which is fully controlled with tablets.
Which of the following best describes her preoperative morbidity?
A. ASA grade I
B. ASA grade II
C. ASA grade III
D. ASA grade IV
E. ASA grade V

A

B

607
Q

A 56-year-old man is brought to hospital by his wife. He has been vomit- ing fresh blood since earlier that morning. She tells you he has a long his- tory of alcohol abuse and drinks at least a bottle of spirits a day.
Which of the following is the most likely cause of his symptoms?
A. Dieulafoy lesions
B. Duodenal ulcer
C. Gastric ulcer
D. Oesophageal varices
E. Oesophagitis

A

D

608
Q

A 28-year-old woman was previously admitted to the orthopaedic ward with an ankle fracture which was treated with a plaster cast. When the cast is removed, the patient is unable to dorsiflex her foot. All other leg movements are maintained.
Which nerve has most likely been affected?
A. Common peroneal nerve
B. Obturator nerve
C. Sciatic nerve
D. Sural nerve
E. Tibial nerve

A

A

609
Q
A 45-year-old woman presents with a 2-month history of worsening jaun- dice, itching and malaise. On examination, she has multiple localized areas of yellow pigmentation around her eyes. She has a past history of Sjögren syndrome, but is otherwise well.
What is the most likely diagnosis?
A. Acute cholecystitis
B. Ascending cholangitis
C. Biliary colic
D. Primary biliary cirrhosis
E. Primary sclerosing cholangiti
A

D

610
Q
A 28-year-old man presents with a 3-month history of pain in both calves which comes on with walking and is relieved by resting. He is otherwise fit and well, although he does smoke 30 cigarettes a day. On examination, the legs and feet are warm, but the pedal pulses are not palpable. The upper limbs are normal
What is the most likely diagnosis?
A. Buerger disease
B. Embolus
C. Intermittent claudication
D. Spinal stenosis
E. Takayasu arteritis
A

A

611
Q

A 50-year-old woman with known gallstones presents to the emergency department with severe epigastric pain radiating to the back together with nausea and vomiting. Examination reveals localized epigastric peri- tonitis and investigations reveal an amylase of 650 μ/L.
Which of the following would indicate a poor prognosis in this condition?
A. Amylase of 650 μ/L
B. Arterial pO2 of 7.0 kPa
C. Patient age of 50 years
D. Pyrexia of 38.5°C
E. White cell count of 10 × 103/μL

A

B

612
Q

A 6-year-old boy who had a fracture above the left elbow and was treated in a plaster cast is brought into the GP practice by his mother a few days after plaster removal. She is worried due to the abnormal positioning of his forearm. On examination, his left forearm appears to be shortened and held in flexion at the wrist and the fingers.
Which of the following complications has led to this appearance?
A. Brachial artery injury
B. Lack of physiotherapy
C. Malunion at fracture site
D. Median nerve injury
E. Ulnar nerve injury

A

A

613
Q

A 40-year-old man presents to the GP complaining of increasing diffi- culty in swallowing over the last few months. He tells you he has been working in Mexico for the last two years with a new business. He is other- wise well and denies any other symptoms.
Which of the following is the most likely cause of his symptoms?
A. Chagas disease
B. Gastro-oesophageal reflux disease
C. Myasthenia gravis
D. Plummer-Vinson syndrome
E. Zenker diverticulum

A

A

614
Q
A 30-year-old woman presents with multiple bilateral breast swell- ings which cause her discomfort, particularly just before her periods. They have been present for several years but appear to be getting worse.
What is the most likely diagnosis?
A. Duct ectasia 
B. Fat necrosis
C. Fibroadenoma
D. Fibrocystic disease
E. Peau d’orange
A

D

615
Q

You are called to see a 56-year-old man who is one day post appendicec- tomy because he became acutely short of breath. He has just been given his first dose of cyclizine to relieve nausea. On arrival, the patient is breath- less with the following observations: heart rate 122/min, blood pressure 86/48 mmHg and saturations 85% in air.
Which of the following would you administer first?
A. Adrenaline
B. Chlorphenamine
C. Fluids
D. Hydrocortisone
E. Salbutamol

A

A

616
Q
A 34-year-old man presents to the emergency department with a 10-hour history of abdominal pain associated with nausea and vomiting. On examination, the patient is lying still and has tenderness with guarding in the right iliac fossa. His temperature is 37.6°C. He has no significant past medical history.
What is the most likely diagnosis?
A. Appendicitis 
B. Crohn disease
C. Meckel diverticulitis 
D. Mesenteric adenitis
E. Renal colic
A

A

617
Q
A 43-year-old man presents to the GP with a 2-month history of wors- ening headaches. The headaches are almost constant and are worse in the morning. On examination, his skin is thick and greasy and he is hypertensive.
What is the most likely diagnosis?
A. Acromegaly
B. Addison disease
C. Congenital adrenal hyperplasia
D. Conn syndrome
E. Phaeochromocytoma
A

A

618
Q
A 55-year-old man presents to the emergency department with increasing itching and upper abdominal discomfort. His wife has noticed that he is looking ‘yellow’. On examination, there is a non-tender mass in the right upper quadrant. The patient has a history of ulcerative colitis, which is currently in remission.
What is the most likely diagnosis?
A. Cholangiocarcinoma B. Gallstones
C. Haemolysis 
D. Hepatitis
E. Pancreatic carcinoma
A

A

619
Q
A 27-year-old man presents to the emergency department after being stabbed in the back. He is now unable to move his right leg. On exami- nation, you note that he cannot feel pain on the left leg, although motor function in this limb is preserved.
What is the most likely diagnosis?
A. Anterior cord syndrome 
B. Brown-Séquard syndrome
C. Central cord syndrome 
D. Posterior cord syndrome
E. Syringomyelia
A

B

620
Q
A 12-year-old boy is brought to the GP practice by his mother with a 2-month history of malaise and a worsening yellowing of his skin. His mother says that he has been behaving differently the last few days. On examination, his abdomen is slightly distended and the liver is palpable.
What is the most likely diagnosis?
A. Haemochromatosis 
B. Hepatitis A
C. Infectious mononucleosis
 D. Riedel lobe
 E. Wilson disease
A

E

621
Q
A 70-year-old woman presents with a 2-month history of anal pain and itching. More recently she had been having some fresh bleeding and mucous discharge per rectum. On examination, there is an irregular ten- der ulceration at the anal margin which appears to be extending into the anal canal.
What is the most likely diagnosis?
A. Anal carcinoma B. Anal fissure
C. Anal warts
D. Fistula-in-ano
E. Primary syphilis
A

A

622
Q

A new form of CT scan is being piloted to help detect scaphoid fractures. A study looked at 600 people: 300 people with scaphoid fractures and 300 without. The trial produced 250 positive results and 350 negative results. Of the 250 positive results, there were no false positives, and of the 350 negative results, 50 were false negatives.
What is the sensitivity of CT scanning in detecting scaphoid fractures in this study?
A. 17%
B. 33%
C. 50%
D. 83%
E. 100%

A

D

623
Q

A newborn baby boy is found to have visible intestine emerging from his abdomen. There is no covering to the contents.
Which of the following is the most likely diagnosis?
A. Epigastric hernia B. Exomphalos
C. Gastroschisis
D. Paraumbilical hernia
E. Umbilical hernia

A

C

624
Q

A 27-year-old woman is undergoing a routine surveillance colonoscopy for ulcerative colitis. The endoscopist notes multiple small projections throughout the bowel that are of a similar colour to the normal bowel mucosa.
What is the most likely morphology of the polyps?
A. Adenomatous polyp
B. Hamartomatous polyp
C. Juvenile polyp
D. Metaplastic polyp
E. Pseudopolyp

A

E

625
Q

A 45-year-old woman presents to the orthopaedic clinic with a 6-week history of shooting pains in her right foot, radiating to her toes, only experienced when wearing her shoes. On examination, there is tenderness between the third and fourth metatarsal heads. Foot X-rays are reported as normal.
Which of the following is the most likely diagnosis?
A. Bunion
B. Gout
C. March fracture D. Morton neuroma
E. Plantar fasciitis

A

D

626
Q
A 69-year-old man presents with a swelling in his right groin. He other- wise feels well. On examination, he has a medial lying, minimally tender pulsatile lump. He tells you he recently suffered from a heart attack and had an angioplasty.
What is the most likely diagnosis?
A. False aneurysm B. Femoral hernia
C. Groin abscess D. Inguinal hernia
E. Saphena varix
A

A

627
Q

A 65-year-old woman presents with episodes of lower abdominal pain and intermittent vaginal bleeding. On examination, there is a palpable mass in the lower abdomen.
Which of the following tumour markers would be associated with this presentation?
A. Alpha fetoprotein B. Beta-hCG
C. CA 125
D. Calcitonin
E. Carcinoembryonic antigen

A

C

628
Q
A 38-year-old woman presents to the GP with pain in the subareolar region of the left breast associated with occasional blood-stained nipple discharge. Apart from being extremely anxious she has no other associ- ated symptoms. Examination is unremarkable.
What is the most likely diagnosis?
A. Duct ectasia 
B. Galactocoele
C. Intraductal papilloma 
D. Paget disease
E. Prolactinoma
A

C

629
Q
A 76-year-old man presents to the GP with a rapidly growing lump in his neck. It is now causing him difficulty swallowing. Examination reveals a 4 cm hard mass in the front of the neck that is fixed to the overlying skin.
What is the most likely diagnosis?
A. Anaplastic carcinoma 
B. Follicular carcinoma
C. Graves disease
D. Papillary carcinoma
E. Toxic multinodular goitre
A

A

630
Q

A 27-year-old man presents to the emergency department with a 6-hour history of upper abdominal pain radiating to the back and associated vomiting. On examination, he has marked epigastric tenderness and you notice a bluish discolouration around his umbilicus. His heart rate is 118/ min and blood pressure is 108/76 mmHg.
Which of the following blood analytes would be most useful in identi- fying a diagnosis?
A. Amylase
B. C-reactive protein
C. Haemoglobin
D. Sodium
E. Urea

A

A

631
Q

A 32-year-old woman presents to the GP with a 1-month history of panic attacks. She says the attacks are associated with sweating and the feeling of her heart thumping in her chest. She cannot always think of a precipita- tor to these attacks, but they are increasing in frequency. Examination is unremarkable.
Which of the following investigations will be the most helpful in con- firming the diagnosis?
A. 17-hydroxyprogesterone levels
B. 24-hour urinary vanillylmandelic acid
C. 24-hour urinary 5-hydroxyindole acetic acid
D. Serum calcitonin
E. Short synacthen test

A

B

632
Q

A 27-year-old man is brought into the resuscitation room having been hit by a car. The patient is stable and his only injury is a fractured right tibia with overlying tissue loss of around 3 cm. His leg is currently in a splint. His distal pulses are palpable.
What would be the most appropriate next step in management?
A. Dress wound
B. External fixation of fracture
C. Internal fixation of fracture
D. Intravenous antibiotics and debridement of tissue
E. Plaster immobilization of fracture

A

D

633
Q
A 22-year-old woman presents to the GP practice with a scaly, well- defined red rash on her cheeks. She has also noticed its appearance on her scalp. There is some associated hair loss.
What is the most likely diagnosis?
A. Acne vulgaris 
B. Discoid lupus
C. Erysipelas 
D. Impetigo
E. Rosacea
A

B

634
Q

An 18-year-old boy is brought into the emergency department follow- ing an epileptic seizure. On recovery he complains of pain in the right shoulder. Examination reveals the arm to be held adducted and internally rotated with a fullness at the posterior aspect of the shoulder. There is resistance to passive external rotation.
Which of the following injuries is the patient most likely to have sustained?
A. Acromioclavicular dislocation
B. Anterior dislocation of the shoulder
C. Inferior dislocation of the shoulder
D. Posterior dislocation of the shoulder
E. Sternoclavicular dislocation

A

D

635
Q

A 63-year-old woman with known varicose veins presents to the surgi- cal outpatients with a fever and a sloughy shallow ulcer above the medial aspect of the ankle with surrounding cellulitis.
Which of the following would be the best immediate treatment option?
A. Debridement and intravenous antibiotics
B. Debridement and topical antibiotics
C. Debridement and sclerotherapy D. Graded compression bandaging
E. Stripping of the long saphenous vein

A

A

636
Q

A 60-year-old Asian woman presents to the GP having noticed a dark brown lesion on the palm of her left hand which has been present for 2 months. She initially thought it was a bruise but it has been slowly enlarging.
What is the most likely diagnosis?
A. Acral lentiginous melanoma
B. Amelanocytic melanoma
C. Lentigo maligna melanoma
D. Nodular melanoma
E. Superficial spreading malignant melanoma

A

A

637
Q
A 12-year-old girl is brought to the GP by her mother having noticed a painless swelling on the left side of her neck following a recent cold. On examination, there is a smooth, fluctuant, non-tender swelling anterior to the sternocleidomastoid muscle.
What is the most likely diagnosis?
A. Branchial cyst 
B. Cervical rib
C. Cystic hygroma
D. Sternocleidomastoid tumour
E. Thyroglossal cyst
A

A

638
Q

A 27-year-old man presents to the emergency department complaining of severe nasal pain and a blocked nose. On examination, he has a bluish discoloured swelling over the bridge of the nose. His friend tells you he was assaulted four days ago.
What is the most likely cause of his symptoms?
A. Fracture of the orbit
B. Fracture of the cribriform plate
C. Intranasal foreign body
D. Maxillary sinusitis
E. Septal haematoma

A

E

639
Q

A 17-year-old boy presents to the emergency department at 1 am with a 3-hour history of pain in his right testicle. He denies any trauma but does a lot of long distance running. Examination is difficult due to pain; however the right testicle does appear to be swollen, slightly red and extremely tender.
Which of the following do you do next?
A. Admit for urgent ultrasound scan the next morning
B. Contact the urologist on call and organize urgent ultrasound scan
C. Contact the urologist on call and prepare for theatre
D. Obtain a urine sample and prescribe antibiotics
E. Prescribe strong analgesia and advise to stop running till symptoms resolve

A

C

640
Q

A 27-year-old man who was the driver of a car involved in a high speed col- lision with a truck is brought into the resuscitation room. On arrival, he is complaining of severe left-sided chest pain. You note that he is breathless, tachycardic and hypotensive. He has reduced air entry on the left side of the chest and the trachea is deviated to the right.
What would you do next?
A. Insert a chest drain into the fifth intercostal space
B. Insert a wide bore cannula into the second intercostal space
C. Request an urgent chest X-ray
D. Request an urgent ECG
E. Perform a pericardiocentesis

A

B

641
Q

A 54-year-old man presents to the emergency department in the middle of the night with acute pain in his left eye, blurred vision and vomiting. He tells you it happened suddenly while he was watching TV. On exami- nation his cornea is injected and the eyeball feels hard. His pupil is semi- dilated and fixed.
How would you manage this patient in the first instance?
A. Refer patient for iridectomy
B. Topical aciclovir
C. Topical antibiotics
D. Topical pilocarpine
E. Topical steroids

A

D

642
Q
A 6-year-old boy is brought into the emergency department by his father with a 1-day history of right hip pain. There is no history of trauma and the child is systemically well. On examination, the child is afebrile and there is generalized reduced range of movement of the hip. Blood tests and X-rays show no abnormality.
What is the most likely diagnosis?
A. Congenital dislocation of the hip 
B. Irritable hip
C. Perthes disease D. Septic arthritis
E. Slipped upper femoral epiphysis
A

B

643
Q

A 54-year-old woman presents to the GP complaining of generalized muscle weakness, difficulty swallowing and blurred vision, all of which are worse at the end of the day. Examination is unremarkable apart from ptosis of the right eye.
Which of the following investigations would be most useful in estab- lishing the cause of her symptoms?
A. Creatine kinase levels
B. Electromyography
C. Muscle biopsy
D. Tensilon test
E. Troponin levels

A

D

644
Q

A 72-year-old man comes to see you in the GP practice complaining of a 6-month history of urinary hesitancy, poor stream and occasional incon- tinence. He is increasingly troubled by his symptoms. Blood test results show a prostate specific antigen of 30 ng/mL.
Which of the following is the most appropriate next step?
A. Antibiotics
B. Medical management with alpha-blockers
C. Referral for transurethral resection of prostate
D. Urgent referral for renal tract ultrasound
E. Urgent referral for transrectal ultrasound and prostate biopsy

A

E

645
Q

A 64-year-old woman presents with a 2-day history of increasing left-sided abdominal pain with fever. On examination, she has localized peritonism
in the left iliac fossa. Her blood tests reveal a raised white cell count and C-reactive protein.
Which of the following is the most likely diagnosis?
A. Constipation
B. Diverticular disease
C. Diverticulitis
D. Diverticulosis
E. Irritable bowel syndrome

A

C

646
Q
A 14-year-old boy who has been complaining of pain and a localized ten- der swelling above his right knee for a month has been sent for X-rays by his GP. The X-ray shows an ill-defined breach in the lower end of the cor- tex of the femur with periosteal elevation and calcification.
What is the most likely diagnosis?
A. Chondrosarcoma B. Ewing sarcoma
C. Lipoma 
D. Osteoma
E. Osteosarcoma
A

E

647
Q

A 63-year-old man presents to the GP practice following two episodes of passing blood in his urine. These episodes were not associated with pain. He has no past medical history of note but is a lifelong smoker.
Which of the following is the most likely cause?
A. Cystitis
B. Diverticulitis
C. Renal calculi
D. Transitional cell carcinoma
E. Urethral injury

A

D

648
Q
A 4-week-old baby girl is brought to the paediatrics clinic with swelling of both legs which the parents feel has been present since birth. On exami- nation there is oedema, which is non-pitting and firm to touch, of both lower limbs to the knee.
What is the most likely diagnosis?
A. Elephantiasis
B. Hereditary angioedema
C. Lymphoedema praecox 
D. Lymphoedema tarda
 E. Milroy disease
A

E

649
Q

An 8-year-old boy is brought to the emergency department with a 3-day history of left-sided flank and abdominal pain, fevers and reduced appe- tite. On examination, he has minimal left-sided flank and lower abdomi- nal tenderness, and there is a fluctuant, non-tender swelling in the child’s groin. You note that the child is walking with a limp and there is pain in the hip region on straight leg raising against resistance.
How would this child be best managed?
A. Appendicectomy
B. Arthroscopic hip washout
C. Exploration of testes
D. Incision and drainage with intravenous antibiotics
E. Intravenous antibiotics alone

A

D

650
Q

A 33-year-old woman has been referred to the orthopaedic clinic by her GP. She complains of a 2-month history of pain and numbness in the left hand, which is worse at night, together with difficulty gripping. On exam- ination, there is reduced sensation over the thumb, index and middle fin- ger together with some wasting of the thenar eminence and weakness of thumb abduction. Her symptoms can be recreated by forcibly bending the wrist.
Compression of which of the following structures is resulting in her symptoms?
A. Median nerve
B. Radial artery
C. Radial nerve
D. Ulnar artery
E. Ulnar nerve

A

A

651
Q

A 45-year-old woman has been diagnosed with gallstones. She opts to have an open cholecystectomy.
Which of the following surgical incisions would be most appropriate?
A. Gridiron
B. Lanz
C. Right Kocher’s
D. Right paramedian
E. Rooftop

A

C

652
Q

A 56-year-old woman is brought into the resuscitation room. Three days ago she suffered an insect bite to the abdomen which has now spread, causing redness across the whole of her abdomen. On examination, she is confused, with a temperature of 38.3°C, heart rate 106/min and blood pressure 100/56 mmHg.
Which of the following is specific to the treatment of this type of shock?
A. Antibiotics
B. Antihistamines
C. Atropine
D. Fluids
E. Inotropes

A

A

653
Q

A 21-year-old girl has been out drinking with her friends. She is escorted into hospital by the paramedics after vomiting up a large amount of fresh blood. She is normally fit and well.
Which of the following is the most likely cause?
A. Boerhaave syndrome
B. Epistaxis
C. Haemoptysis
D. Mallory–Weiss tear
E. Oesophageal varices

A

D

654
Q
A 15-year-old boy attends the emergency department with a 2-week history of pain in the left lower leg and fever. On examination, there is a tender, irregular swelling of the tibia. X-ray of the leg shows a lytic lesion with a laminated periosteal reaction.
What is the most likely diagnosis?
A. Chondrosarcoma B. Enchondroma
C. Ewing sarcoma 
D. Osteoid osteoma
E. Osteosarcoma
A

C

655
Q
A 72-year-old woman presents to the emergency department with a 5-hour history of abdominal pain, bloating and vomiting. On examination, the abdomen is distended and auscultation reveals intermittent high-pitched sounds. An abdominal X-ray shows air in the biliary tree.
What is the most likely diagnosis?
A. Emphysematous cholecystitis 
B. Gallstone ileus
C. Paralytic ileus
D. Perforated gallbladder
E. Pyogenic cholecystitis
A

B

656
Q
A 52-year-old man presents to the GP complaining of an enlarging lump on his face. On examination, he has a 1 cm pigmented, raised lesion on his left cheek that has a shiny rolled edge.
What is the most likely diagnosis?
A. Basal cell carcinoma 
B. Bowen disease
C. Keratoacanthoma
D. Seborrhoeic keratosis
E. Squamous cell carcinoma
A

A

657
Q

A 43-year-old woman presents with bright red fresh rectal bleeding and intermittent rectal discomfort. She has recently noticed a mass in her rec- tum. On examination, a non-tender lump is seen emerging from the rec- tum. It cannot be reduced.
Which of the following would be the most appropriate treatment?
A. Anal dilatation
B. Banding
C.Haemorrhoidectomy D. Injection sclerotherapy
E. No intervention required

A

C

658
Q

A 42-year-old woman presents to the emergency department with dif- ficulty breathing and facial flushing which began soon after having a few drinks at the pub. She tells you that she has had a few of these epi- sodes recently, always after having an alcoholic drink. Examination is unremarkable. Her observations include: heart rate 82/min and blood pressure 122/86 mmHg.
Which investigation will most likely confirm the underlying diagnosis?
A. 17-hydroxyprogesterone levels
B. 24-hour urinary vanillylmandelic acid
C. 24-hour urinary 5-hydroxyindole acetic acid
D. Serum calcitonin
E. Short synacthen test

A

C

659
Q

A 62-year-old man presents to the GP with a lump in the left groin which has been present for over 2 months. On examination, the lump is above the inguinal ligament. It is reducible and has a cough impulse, but does not extend into the scrotum.
Which of the following is the most likely diagnosis?
A. Direct inguinal hernia
B. Femoral hernia
C. Gluteal hernia
D. Indirect inguinal hernia
E. Obturator hernia

A

A

660
Q
A 44-year-old man presents to the GP having noticed a painless swelling in the right side of the scrotum. On examination, the scrotum is swollen and non-tender. The swelling transilluminates and the testis itself is not palpable.
What is the most likely diagnosis?
A. Epididymo-orchitis 
B. Hydrocele
C. Inguinal hernia 
D. Testicular torsion
 E. Varicocele
A

B

661
Q
A 27-year-old woman presents to the GP with a single lump in the left breast that has been present for 6 months. On examination, there is a 3 cm smooth lump in the upper outer quadrant of the breast that is mobile and not attached to the overlying skin.
What is the most likely diagnosis?
A. Breast abscess B. Breast cyst
C. Fibroadenoma 
D. Gynaecomastia
E. Phyllodes tumour
A

C

662
Q

A 6-month-old boy is brought into the emergency department by his par- ents. He appears to have been having paroxysms of intense pain since that morning, described by his parents as episodes of crying and drawing of his legs up to the abdomen. This evening he has passed some mucus-like blood from the back passage. His temperature is 36.8°C.
Which of the following would be the best treatment option in the first instance?
A. Barium enema
B. Continued observation
C. Intravenous antibiotics
D. Laparotomy
E. Scrotal exploration

A

A

663
Q

A 67-year-old woman presents to the emergency department with a 1-month history of malaise, weight loss, worsening cough and haemoptysis. You notice her saturations are 92% on air and perform a blood gas test with the follow- ing results: pH 7.37, pO2 6.7, pCO2 7.4, bicarbonate 35, saturations 91.7%.
Reference ranges: pO2 >11.0 kPa, pCO2 4.6–6.0 kPa, bicarbonate 22–28 mmol/L, pH 7.35–7.45.
What blood gas picture does this represent?
A. Fully compensated metabolic acidosis B. Fully compensated metabolic alkalosis C. Fully compensated respiratory acidosis
D. Type I respiratory failure E. Venous sample

A

C

664
Q

A 40-year-old woman presents to the emergency department with a 4-hour history of right upper abdominal pain and fever. She tells you that she has
previously been investigated for abdominal pain and was found to have gall- stones. She says that she has never felt this unwell with the pain before. On examination, she is markedly jaundiced with a temperature of 39°C and pulse rate of 130/min. There is localized peritonism in the right upper abdomen.
Which of the following complications of gallstones has this patient presented with?
A. Acute cholecystitis
B. Ascending cholangitis
C. Biliary colic
D. Carcinoma of the gallbladder
E. Gallstone ileus

A

B

665
Q

A 56-year-old man presents to the pre-assessment clinic prior to an elec- tive knee operation. He tells you he smokes 20 cigarettes a day and suffers from occasional angina.
Which of the following best describes his preoperative morbidity?
A. ASA grade I
B. ASA grade II
C. ASA grade III
D. ASA grade IV
E. ASA grade V

A

B

666
Q
A 41-year-old woman presents to the GP with a 1-week history of feeling tired, muscle aches, neck pain and restlessness. On examination, she has a smooth enlarged thyroid. Her heart rate is 120/min and blood pressure 128/88 mmHg, with a temperature of 37.3°C.
What is the most likely diagnosis?
A. De Quervain thyroiditis 
B. Hashimoto thyroiditis
C. Pharyngitis
D. Riedel thyroiditis
E. Thyroglossal cyst
A

A

667
Q

A 33-year-old man presents with a 2-hour history of upper abdominal pain that radiates to his back and shoulders. On examination, he is sweaty and his abdomen is rigid. His observations include heart rate 106/min and blood pressure 98/58 mmHg.
Which of the following is the most likely cause of his symptoms?
A. Epigastric hernia incarceration
B. Gastric carcinoma
C. Gastritis
D. Hiatus hernia
E. Perforated peptic ulcer

A

E

668
Q

A 43-year-old man is brought into the resuscitation room having been involved in a house fire. On arrival you estimate him to have 20% burns. The man is around 80 kg in weight.
How much fluid should the patient have over the next 12 hours?
A. 300 mL
B. 600 mL
C. 1200 mL
D. 2400 mL
E. 3000 mL

A

D

669
Q

A 17-year-old boy is referred to the endocrinology clinic with a 1-month history of paroxysmal sweating, anxiety and palpitations. He has had a thyroid tumour in the past which was successfully resected. On examina- tion, you notice that he is tall and thin with long fingers.
What is the most likely diagnosis?
A. Carcinoid syndrome
B. Multiple endocrine neoplasia type I
C. Multiple endocrine neoplasia type IIa D. Multiple endocrine neoplasia type IIb
E. Nelson syndrome

A

D

670
Q

A 54-year-old woman has been having episodes of epigastric pain asso- ciated with food. She is referred for an endoscopy which shows gastric ulceration. A rapid urease test performed on a biopsy sample is positive.
Which of the following would you use to treat this patient?
A. Augmentin and metronidazole
B. Omeprazole, clarithromycin and metronidazole
C. Omeprazole, prednisolone and augmentin
D. Omeprazole and ibuprofen
E. Ranitidine

A

B

671
Q
A 7-month-old boy is taken by his parents to the GP with a red lesion on his scalp which has been enlarging over the past 2 months. On examina- tion, the lesion is 1 cm in size, raised, bright red and well defined. The boy is otherwise well.
What is the most likely diagnosis?
A. Cavernous haemangioma
B. Deep capillary naevus
C. Dercum disease
D. Pyogenic granuloma
E. Superficial capillary naevus
A

A

672
Q
A 12-month-old girl is brought to the GP by her mother with a large mass in the right side of her neck which she has noticed for a few months. On examination, the mass is fleshy and compressible, and lies posterior to the right sternocleidomastoid.
What is the most likely diagnosis?
A. Branchial cyst 
B. Cystic hygroma
C. Pleomorphic adenoma
D. Sternocleidomastoid tumour
E. Thyroglossal cyst
A

B

673
Q

A 3-year-old boy is brought into the GP practice by his father. The child has had a yellowish discharge from his right nostril for the last 3 days. Examination of the nostril is difficult as the child is distressed, but you see what appears to be a small piece of plastic.
Which of the following would you do?
A. Continue to try to remove the foreign body yourself
B. Give oral antibiotics and see the child in a week
C. Perform cauterization
D. Perform nasal packing
E. Refer to ENT on call to remove foreign body

A

E

674
Q
A 24-year-old woman presents to the emergency department complaining of a 2-day history of lower abdominal pain with associated foul-smelling vaginal discharge. On examination, she is tender and guarding in both the suprapubic region and the right iliac fossa. A urine dipstick shows leukocytes and protein.
What is the most likely diagnosis?
A. Appendicitis
B. Ectopic pregnancy
C. Mittelschmerz
D. Pelvic inflammatory disease
 E. Urinary tract infection
A

D

675
Q

A 64-year-old woman who was conservatively treated in plaster for a Colles fracture re-presents to the fracture clinic 2 weeks after plaster removal complaining of a burning pain in the whole hand. On examina- tion, the hand is slightly swollen, red and shiny. There is a reduction in all hand and finger movements.
Which of the following is most likely to be the cause of her symptoms?
A. Carpal tunnel syndrome
B. Compartment syndrome
C. Malunion of the fracture
D. Rupture of the extensor pollicis longus
E. Sudeck atrophy

A

E

676
Q

A 26-year-old man has been referred to the fracture clinic with instabil- ity of the left knee. His symptoms followed a football injury in which he heard a ‘pop’ in his knee while being tackled. He tells you that he attended the emergency department immediately and a large amount of blood was drained from the knee.
On examination which of the following would you expect to find?
A. Positive Lachman test
B. Positive McMurray test
C. Positive Simmonds test
D. Positive patellar apprehension test E. Valgus instability of the knee joint

A

A

677
Q

A 32-year-old man is involved in a high speed car accident. On arrival at the resuscitation room he complains of chest pain and is short of breath and tachycardic. He has an area of paradoxical movement of the chest wall.
Which of the following would be the most appropriate management measure?
A. Analgesia and respiratory support B. Chest drain insertion
C. Needle thoracocentesis
D. Pericardiocentesis
E. Thoracotomy

A

A

678
Q

A 47-year-old man presents to the emergency department with a 2-day history of a painful, watery, red left eye. On examination, the cornea is
injected and examination under cobalt-blue light after the instillation of fluorescein, reveals a branch-like corneal lesion.
Which of the following is the most likely diagnosis?
A. Anterior uveitis
B. Bacterial conjunctivitis
C. Corneal abrasion
D. Episcleritis
E. Herpes simplex keratitis

A

E

679
Q

A 62-year-old smoker presents with a 2-week history of worsening short- ness of breath, haemoptysis and weight loss. On examination, he appears cachexic and his liver is enlarged with an irregular border.
Which of the following is the most likely cause of his hepatomegaly?
A. Budd-Chiari syndrome
B. Hepatocellular carcinoma
C. Liver cirrhosis
D. Metastatic liver disease
E. Riedel lobe

A

D

680
Q
A 34-year-old woman presents to the GP with a 3-week history of left nipple discharge. She describes the discharge as thick and greenish. On examination, no mass is palpable within the breast. A beta-hCG urine test is negative.
What is the most likely diagnosis?
A. Breast cancer 
B. Duct ectasia
C. Intraductal papilloma 
D. Lactating breast
E. Prolactinoma
A

B

681
Q
A 1-day-old baby girl is having repeated bile-stained vomiting. Her mother was noted to have polyhydramnios during the pregnancy. An abdominal X-ray reveals gas in the stomach and first part of the duode- num, but nowhere else in the abdomen.
What is the most likely diagnosis?
A. Choledochal cyst B. Duodenal atresia
C. Oesophageal atresia 
D. Pyloric stenosis
 E. Renal agenesis
A

B

682
Q

A 73-year-old man presents to the emergency department with sudden- onset right-sided back pain radiating to the abdomen. He has a past medi- cal history of hypertension and angina and is known to have gallstones. On examination, he is afebrile, haemodynamically stable and tender in the right side of the abdomen. Blood tests show normal inflammatory markers. Urine dipstick shows a trace of blood.
What investigation would you perform next?
A. CT abdomen
B. Intravenous urogram
C. Laparotomy
D. MRI spine
E. Ultrasound abdomen

A

A

683
Q
A 21-year-old man presents to the emergency department with a collapse that was preceded by dizziness and right arm pain. He was sanding a table at home when the event happened. On examination, he has a bony lump palpable in the right side of the neck. Neurological and vascular examina- tions are otherwise normal.
What is the most likely diagnosis?
A. Cerebrovascular attack 
B. Deep venous thrombosis
C. Embolus
D. Subclavian steal syndrome
E. Transient ischaemic attack
A

D

684
Q

A 42-year-old woman is taken to theatre after fracturing her left hip in a road traffic collision. Postoperatively, it is noticed that she is unable to flex or extend her left foot. There is no sensation present below the left knee except over the medial aspect of the leg.
Which nerve has most likely been affected?
A. Common peroneal nerve
B. Femoral nerve
C. Sciatic nerve
D. Sural nerve
E. Saphenous nerve

A

C

685
Q

A 33-year-old man comes to see you at the GP surgery complaining of increasing difficulty passing urine with poor flow. He has previously been treated for gonorrhoea on three separate occasions.
Which of the following investigations would be most useful in estab- lishing the cause of his symptoms?
A. Intravenous urogram
B. Ultrasound of the renal tract
C. Urethral swabs D. Urethrography
E. Urinary flow rates

A

D

686
Q
A 76-year-old man presents to the emergency department after a fall. He now feels weak in his arms and legs. On examination, he has mild weak- ness in both arms, and a moderate weakness in the legs. He is unable to feel pain in his legs.
What is the most likely diagnosis?
A. Anterior cord syndrome 
B. Brown-Séquard syndrome
C. Central cord syndrome 
D. Posterior cord syndrome
E. Syringomyelia
A

A

687
Q

A 54-year-old man presents with a recent change in bowel habit associ- ated with a constant desire to defaecate. He has lost two stone in weight over the last month and feels lethargic.
Which of the following tumour markers would be associated with his condition?
A. CA 125
B. CA 15-3
C. CA 19-9
D. CEA
E. PSA

A

D

688
Q

An 8-year-old boy is brought into the emergency department with a 3-day his- tory of right-sided testicular pain. His mother tells you he was off school a week ago with complaints of feeling unwell and jaw pain on eating. On examina- tion, there is unilateral generalized swelling and tenderness of the right testis.
Which of the following treatment options is most appropriate?
A. Analgesia and bed rest
B. Antibiotics
C. Hydrocelectomy D. Orchidectomy
E. Reduction and fixation of testis

A

A

689
Q
A 31-year-old man presents with episodic perianal pain for more than a year. He has occasionally noticed some discharge. On examination, a pit is seen in the lower natal cleft. It is filled with hair and a mini- mal amount of discharge is produced when pressure is applied to the region.
What is the most likely diagnosis?
A. Perianal abscess
B. Perianal haematoma
C. Pilonidal sinus D. Proctalgia fugax
E. Rectal prolapse
A

C

690
Q

A 68-year-old man attends the emergency department following two epi- sodes of weakness in the left side of his face, each lasting 2 hours. These resolve spontaneously and completely. Examination is unremarkable except for a left carotid bruit.
Which of the following steps would you take next?
A. Start aspirin, advise smoking cessation and refer for outpatient CT head
B. Start aspirin, advise smoking cessation and refer for outpatient carotid
Doppler
C. Start aspirin and refer for urgent inpatient assessment
D. Start aspirin and refer to ophthalmology
E. Start steroids, advise smoking cessation and refer for CT head

A

C

691
Q

A 65-year-old woman attends the GP practice. Yesterday, while garden- ing, she experienced sudden-onset pain in her right shoulder. On exami- nation, there is some bruising of the upper arm, and on flexion of the elbow a bulge is seen in the upper arm.
Which of the following is the most likely diagnosis?
A. Distal rupture of biceps tendon
B. Proximal rupture of biceps tendon
C. Rotator cuff tear D. Rupture of triceps
E. Supraspinatus tendonitis

A

B

692
Q
A 27-year-old man presents to the emergency department with a 2-hour history of severe left-sided loin pain which woke him from sleep. The pain is constant but occasionally dulls. He has been feeling sick with the pain and vomited twice. On examination, the patient is writhing in agony and
has mild tenderness in the left loin only. His temperature is 36.9°C. Urine dipstick shows 2+ of blood.
What is the most likely diagnosis?
A. Biliary colic
B. Dissecting aortic aneurysm
C. Pyelonephritis
D. Renal cell carcinoma
E. Renal colic
A

E

693
Q

A 64-year-old woman presents to the GP complaining of pain and dif- ficulty in swallowing. It is becoming progressively worse and now she complains it is even hard to drink water. On examination, she appears cachexic.
Which of the following is the most likely diagnosis?
A. Oesophageal candidiasis
B. Oesophageal carcinoma
C. Oesophageal spasm
D. Oesophageal stricture
E. Oesophageal web

A

B

694
Q
An 8-year-old boy is brought in to the GP practice by his mother. He has been complaining of pain in his right hip for a few weeks and has started walking with a limp. Examination reveals limited range of movement at the hip. An X-ray of the pelvis shows a collapsed femoral head on the right.
What is the most likely diagnosis?
A. Femoral osteomyelitis 
B. Perthes disease
C. Proximal femoral fracture
D. Slipped upper femoral epiphysis
E. Transient synovitis
A

B

695
Q
A 32-year-old man presents to the GP with pain in the right foot which is exacerbated by weight bearing. He tells you he jogs on a daily basis. On examination, he has a tender swelling over the second metatarsal. An X-ray of his foot shows a periosteal reaction over the neck of the second metatarsal.
What is the most likely diagnosis?
A. Gout
B. Hallux rigidus
C. Jones fracture
D. Lisfranc fracture E. March fracture
A

E

696
Q
A 62-year-old woman presents with a large painless, ulcerated lesion over her lower leg. It has been developing gradually over an area of scarring where she sustained a burn several years ago.
What is the most likely diagnosis?
A. Hypertrophic scars 
B. Infected ulcer
C. Keloid scar
D. Marjolin ulcer
E. Perforated ulcer
A

D

697
Q
A 27-year-old woman presents to the GP with a 2-month history of wors- ening swelling in both legs. On examination, her lower legs and thighs are very oedematous and firm. The skin is thick, hard and grey in colour.
What is the most likely diagnosis?
A. Filariasis
B. Hereditary angioedema
C. Lymphoedema praecox 
D. Lymphoedema tarda
E. Milroy disease
A

A

698
Q

A 56-year-old woman presents to the GP complaining of episodes of severe retrosternal chest pain after eating, associated with a bitter taste in her mouth. She has no difficulty or pain when swallowing. Examination is unremarkable.
Which of the following investigations would be most useful in estab- lishing the cause of her symptoms?
A. 24-hour lower oesophageal pH
B. Barium swallow
C. Erect chest X-ray D. Manometry
E. Upper gastrointestinal endoscopy

A

A

699
Q

The orthopaedic surgeon on call has been called down to the emergency department to see a patient who fractured his elbow 3 weeks ago and is suspected to have an ulnar nerve injury.
What would she expect to find on examination?
A. Complete paralysis of the forearm
B. Wasting of the hand, except the thenar eminence, and clawing of the
ring and little fingers
C. Wasting of the thenar eminence and clawing of the index and middle
fingers
D. Wasting of the thenar eminence with inability to abduct the thumb
E. Wrist drop and loss of sensation over the anatomical snuffbox

A

B

700
Q

A 32-year-old woman who has been undergoing fertility treatment pres- ents with a 2-hour history of right-sided lower abdominal pain, associ- ated with nausea. On direct questioning she tells you she has had several similar, but less severe, episodes in the past. She has had no other genito- urinary symptoms. Her pregnancy test is negative.
Which of the following is the most likely diagnosis?
A. Appendicitis
B. Ectopic pregnancy
C. Pelvic inflammatory disease
D. Torsion of the ovary
E. Urinary tract infection

A

D

701
Q

A 36-year-old man presents to the emergency department following a road traffic collision. He has fractured his right tibia and also complains of pain in his neck. On examination, he is unable to lift his right arm, which is medially rotated and extended at the elbow. There is a loss of sensation on the lateral side of the right arm and forearm.
Which nerve has most likely been affected?
A. Lower brachial plexus
B. Median nerve
C. Radial nerve
D. Ulnar nerve
E. Upper brachial plexus

A

E

702
Q

A 19-year-old man is brought into the emergency department following a gunshot wound to the abdomen. Despite resuscitation he remains hae- modynamically unstable. The decision is made to take him to theatre for exploration.
Which of the following surgical incisions would be most appropriate?
A. Battle incision
B. Median thoracotomy
C. Midline laparotomy
D. Pfannenstiel incision
E. Thoraco-abdominal incision

A

C

703
Q

A 34-year-old man has presented to the emergency department following a large haematemesis. On examination, the patient appears agitated. His pulse is 120/min, blood pressure 122/84 mmHg and respiratory rate 22/ min.
How would you classify the patient’s current condition?
A. Class I haemorrhagic shock
B. Class II haemorrhagic shock
C. Class III haemorrhagic shock
D. Class IV haemorrhagic shock
E. Class V haemorrhagic shock

A

B

704
Q
A 52-year-old woman attends the GP practice having noticed a crater-like lesion on the sole of her right foot. She has a history of type 2 diabetes mellitus and hypertension. On examination, the lesion is painless.
What is the most likely diagnosis?
A. Diabetic dermopathy 
B. Necrobiosis lipoidica
C. Neuropathic ulcer
D. Pyoderma gangrenosum
E. Vitiligo
A

C

705
Q
A 54-year-old man presents with a 2-day history of upper abdominal and shoulder tip pain on the right side. He also complains of difficulty breath- ing and episodes of fever and sweats. He underwent a laparotomy 2 weeks ago following a perforated appendix.
What is the most likely diagnosis?
A. Appendix abscess 
B. Cholecystitis
C. Diverticular abscess 
D. Polycystic liver disease
E. Subphrenic abscess
A

E

706
Q
An 18-year-old has been physically assaulted outside a bar. On arrival at the emergency department she appears drowsy and there is gross swell- ing to the right side of her face. On examination, she is making groaning sounds but is not verbally interactive. She is unable to obey commands but withdraws to pain and opens her eyes only to painful stimulus.
What is her Glasgow coma score?
A. 3 
B. 6 
C. 8
D. 10 
E. 13
A

C

707
Q
A 24-year-old man, who has been suffering from intermittent fresh bleed- ing per rectum, presents to the emergency department with a 6-hour history
of right-sided abdominal pain, fevers and nausea. On examination, he has tenderness and guarding in the right iliac fossa. His temperature is 38.2°C.
What is the most likely diagnosis?
A. Appendicitis 
B. Haemorrhoids
C. Meckel diverticulitis 
D. Renal colic
E. Shigella infection
A

C

708
Q

A 72-year-old man presents with difficulty swallowing and regurgitation of food. He has also had several chest infections over the last few months. On examination, you see a soft swelling in the anterior triangle of the neck. On palpation, the swelling makes a gurgling sound.
What is the most likely cause of his dysphagia?
A. Carotid artery aneurysm
B. Enlarged lymph node
C. Laryngocoele
D. Pharyngeal pouch
E. Thyroglossal cyst

A

D

709
Q

A 22-year-old man is brought in following a road traffic collision. He is suspected to have intra-abdominal injuries and there is bruising over his trunk. On arrival, he has a pulse rate of 140/min and his systolic blood pressure is 80 mmHg.
Which of the following fluids would raise the patient’s blood pressure most rapidly?
A. 5% dextrose
B. Fresh frozen plasma
C. Packed red cells
D. Mannitol
E. Normal saline

A

C

710
Q

36-year-old man presents to the surgical outpatient clinic with a 3-week history of soiled underwear. He is known to have Crohn disease. On examination, an opening is seen at the posterior margin of the anus which is discharging faeculent material. Subsequent imaging demonstrates an anal fistula that tracks through the puborectalis muscle.
What would be the most appropriate management of this condition?
A. Diltiazem ointment
B. Excision of the fistula
C. Laying open of the fistula tract
D. Lord procedure
E. Seton insertion

A

E

711
Q

Which term best fits the description of the hernia given below?
This hernia arises from a triangle bounded by the external oblique
muscle, the latissimus dorsi and the iliac crest below.
A. Amyand
B. Littre
C. Lumbar
D. Obturator
E. Sciatic

A

C

712
Q
A 27-year-old man presents to the GP practice having noticed a painless swelling of his right testicle. He is otherwise well. On examination, the testis is enlarged, firm and has a nodular texture.
What is the most likely diagnosis?
A. Epididymal cyst B. Gumma
C. Haematocele 
D. Orchitis
E. Testicular cancer
A

E

713
Q
A 58-year-old woman presents with itchy, dry skin around the left areola. On examination, the skin is dry and cracked, and there appears to be a 1 cm nodule underlying the affected area.
What is the most likely diagnosis?
A. Atopic dermatitis 
B. Fibrocystic disease
C. Mondor disease D. Paget disease
E. Peau d’orange
A

D

714
Q

A 7-year-old boy is brought to the emergency department by his father, having told him that he had swallowed a five pence coin earlier that day. The child has not experienced any symptoms and on examination is well.
Which of the following investigations would you perform in the first instance?
Practice Papers 3: Questions
A. Barium swallow B. Laryngoscopy
C. Neck, chest and abdominal X-rays
D. No investigation required
E. Upper gastrointestinal endoscopy

A

C

715
Q
A 37-year-old man presents to the GP with a 2-month history of thirst and frequency of urination. He has no significant past medical history and examination is unremarkable. Routine bloods demonstrate the fol- lowing: random glucose 5.6 mmol/L, sodium 142 mmol/L and potassium 2.9 mmol/L.
What is the most likely diagnosis?
A. Addison disease
B. Congenital adrenal hyperplasia
C. Conn syndrome
D. Multiple endocrine neoplasia
E. Phaeochromocytoma
A

C

716
Q

A 74-year-old woman presents to the emergency department complain- ing of dizziness and tiredness. On direct questioning she tells you she has recently been having black stools. She is currently awaiting a total hip replacement for arthritis.
Which of the following would be most useful in establishing the pri- mary cause of her symptoms?
A. Barium meal
B. Coagulation screen
C. Faecal occult blood
D. Full blood count
E. Oesophagogastroduodenoscopy

A

E

717
Q
A 42-year-old woman presents to her GP with a new rash on her left hand. She has a history of diabetes mellitus. On examination, the lesion is on the dorsal hand and is made up of reddish bumps arranged in a ring. She is otherwise well.
What is the most likely diagnosis?
A. Dercum disease
B. Granuloma annulare
C. Lipoma
D. Pyogenic granuloma
 E. Seborrhoeic keratosis
A

B

718
Q

A 6-year-old girl is referred by her GP to the paediatric clinic with preco- cious puberty. On examination, she is found to have clitoromegaly and some pubic hair. She is on the 98th centile for height and weight.
Which of the following will be most helpful in determining the under- lying diagnosis?
A. 17-hydroxyprogesterone levels
B. 24-hour urinary vanillylmandelic acid
C. 24-hour urinary 5-hydroxyindole acetic acid
D. Serum calcitonin
E. Short synacthen test

A

A

719
Q

A 6-year-old girl is brought to the emergency department with multiple crusted lesions on her face which have spread over the last two days. They are very itchy and often bleed. She is otherwise well.
Which of the following is the most appropriate course of management?
A. Analgesia
B. Intravenous antibiotics
C. Oral antibiotics D. Surgical excision
E. Reassurance

A

C

720
Q
A 42-year-old man presents to the GP with a slowly enlarging mass in the left side of his neck which has been present for 3 months. On examina- tion, the mass is 2 cm in diameter, non-tender, pulsatile and can be moved from side to side, but not up or down.
What is the most likely diagnosis?
A. Branchial cyst 
B. Cervical rib
C. Chemodectoma
D. Pleomorphic adenoma
E. Virchow node
A

C

721
Q

A 23-year-old man presents to the emergency department with a nose bleed. He tells you this has been a recent recurrent problem and that his father was the same. On examination, you note that he has multiple dilated blood vessels around the nose and mouth.
Which of the following is the most likely cause of his epistaxis?
A. Haemophilia B B. Hypertension
C. Osler-Weber-Rendu syndrome D. Septal polyps
E. Sturge-Weber syndrome

A

C

722
Q
A 33-year-old man presents to the emergency department with an acutely painful swollen right knee. On examination, he has a swollen, red hot knee with minimal range of movement. Aspiration of the joint shows tur- bulent fluid with a raised neutrophil count and no crystals.
What is the most likely diagnosis?
A. Gout
B. Pseudogout
C. Reiter’s syndrome
D. Rheumatoid arthritis
E. Septic arthritis
A

E

723
Q

A 22-year-old man is brought into the resuscitation room with multi- ple stab wounds to the chest. On arrival he is tachycardic, hypotensive and has engorged jugular veins. His heart sounds are barely audible on auscultation.
Which of the following is the most likely cause of his symptoms?
A. Cardiac tamponade
B. Cardiogenic shock
C. Haemothorax
D. Haemorrhagic shock
E. Tension pneumothorax

A

A

724
Q

A 42-year-old woman presents with an upper midline mass that has been present for over a year. She has a history of a partial gastrectomy for a per- forated ulcer. On examination, the mass is 5 cm in size, soft, non-tender and reducible.
Which of the following is the most likely diagnosis?
A. Direct inguinal hernia
B. Epigastric hernia
C. Hiatus hernia
D. Incisional hernia
E. Paraumbilical hernia

A

D

725
Q

An 18-year-old boy is brought into the emergency department hav- ing fallen down a flight of stairs outside his flat. His cervical spine is
immobilized by the attending paramedics. The patient is saying he does not have neck pain and wants the collar removed.
In which of the following circumstances is it appropriate to remove the collar?
A. He has been drinking alcohol
B. He has lost consciousness for only 5 minutes
C. He has no cervical spine tenderness
D. He is in shock with a splenic injury which requires a laparotomy
E. He has no peripheral neurological signs

A

C

726
Q

An 18-month-old girl has been referred to the orthopaedic outpatient clinic. Her mother has noticed that she is walking with an increasing limp on the left side. The girl denies any pain. A pelvic X-ray shows a vertically orientated acetabular roof and poorly developed femoral head.
What is the most likely diagnosis?
A. Developmental dysplasia of the hip B. Genu valgum
C. Genu varum
D. Perthes disease
E. Slipped upper femoral epiphysis

A

A

727
Q
A 27-year-old man presents to the GP with a sore throat and fevers. On examination, you notice that he is jaundiced and the tonsils are inflamed. On direct questioning the patient tells you that he has noticed this discol- ouration of his skin before, particularly when run down or after drinking alcohol. His blood tests are all normal except for a mildly raised bilirubin.
What is the most likely diagnosis?
A. Alcoholic hepatitis 
B. Gallstones
C. Gilbert syndrome
D. Type 1 Crigler-Najjar syndrome
E. Viral hepatitis
A

C

728
Q

You are called to see a 62-year-old man on the ward who was admitted earlier in the day with frank haematuria with the passage of clots. He is now complaining of lower abdominal pain and an inability to pass urine. On examination the bladder is distended.
Which of the following would you perform in the first instance?
A. Catheterization using a Foley catheter
B. Catheterization using a three-way catheter
C. Cystoscopy
D. Suprapubic catheterization
E. Ultrasound of the bladder

A

B

729
Q

A 40-year-old man is being investigated for an enlarging painless lump in the right testicle. Blood results show a significantly raised alpha fetopro- tein and a normal beta-hCG.
Which of the following testicular tumours is most likely to be the cause of his symptoms?
A. Choriocarcinoma B. Leydig cell tumour
C. Seminoma
D. Testicular lymphoma
E. Yolk sac carcinoma

A

E

730
Q

A 38-year-old woman who has a history of intermittent right upper quadrant pain associated with eating, attends the emergency depart- ment with a 3-day history of severe, unremitting abdominal pain and jaundice. A bedside ultrasound scan shows a 1 cm stone in her common bile duct.
Which of the following would be most suitable management?
A. Cholecystectomy B. ERCP
C. Fragmentation of the stone D. Laparotomy
E. MRCP

A

B

731
Q

A 65-year-old woman with a past medical history of hypertension and sta- ble angina presents to the GP practice complaining of episodes of severe central abdominal pain. They occur around half an hour after meals and can last up to an hour. She is becoming reluctant to eat because of the pain and as a result she has lost nearly a stone in weight over the past 2 months. Examination is unremarkable.
Which of these investigations would be the most informative?
A. Barium follow through
B. CT scan
C. Colonoscopy
D. Exercise tolerance test
E. Mesenteric angiography

A

E

732
Q

A 27-year-old man who has just returned from a holiday in Asia presents to the GP surgery having noticed a painless hard ulcer over the penile glans. He is otherwise well. On examination, the patient has generalized painless lymphadenopathy.
Which of the following is the most likely cause of his symptoms?
A. Escherichia coli
B. Haemophilus ducreyi
C. Herpes simplex virus
D. Human papilloma virus
E. Treponema pallidum

A

E

733
Q

A 69-year-old man presents with a 6-week history of lower back and right hip pain. An X-ray of the hip shows an osteosclerotic lesion in the proxi- mal femur. On direct questioning he tells you he has been having prob- lems passing urine over the past few months.
Which of the following is the most likely cause of his symptoms?
A. Benign prostatic hypertrophy
B. Paget disease
C. Prostatic carcinoma
D. Prostatitis
E. Testicular carcinoma

A

C

734
Q

A 65-year-old man presents to the vascular outpatient clinic with a history of severe cramping pains in the right calf on walking. The pain has been so severe that he has had to stop his activities. Over the last few months he has also had pain at rest, particularly at night, which is relieved on hang- ing his leg off the bed. Examination of the right leg shows it is cool and there is difficulty palpating the pulses. There are no gangrenous changes or ulcers.
Which of the following ankle brachial pressure index results would you expect?
A. 0.1
B. 0.3
C. 0.7
D. 1.0
E. 1.4

A

B

735
Q

A 34-year-old man presents to the emergency department with a 6-hour history of abdominal pain and vomiting. On examination, the abdomen
is distended and tympanic. He tells you that for the last 6 months he has been having episodes of cramping right-sided abdominal pain with inter- mittent episodes of blood in his stools.
Which of the following complications has resulted in his presentation today?
A. Abscess formation
B. Fistula formation
C. Primary sclerosing cholangitis
D. Stricture formation
E. Toxic megacolon

A

D

736
Q

A 27-year-old man presents with fevers and right lower abdominal pain following a 1-week history of bloody diarrhoea. He has recently returned from holiday in the Maldives, but has previously been fit and well. On examination, he has a tender palpable liver.
Which of the following is the most likely cause of his symptoms?
A. Acute cholecystitis
B. Amoebic liver abscess
C. Fitz-Hugh–Curtis syndrome
D. Polycystic liver
E. Pyogenic liver abscess

A

B

737
Q

A 72-year-old man presents to the emergency department with a 3-hour history of a painful, cold right leg. He has a past medical history of atrial fibrillation. On examination, the right leg is cold and tender. The pedal pulses on the right are not palpable.
What would be the most appropriate initial management?
A. Amputation
B. Aorto-bifemoral bypass graft
C. Conservative management
D. Embolectomy
E. Fasciotomy

A

D

738
Q

A 33-year-old man presents to the emergency department having hit the end of his finger with a hammer while at work. He is now unable to lift the end of his finger. On examination, the index finger is flexed at the distal interphalangeal joint.
Which of the following is the most likely diagnosis?
A. Boutonniere deformity
B. Duck bill deformity
C. Mallet finger
D. Swan neck deformity
E. Trigger finger

A

C

739
Q

A 27-year-old man with a long history of constipation presents with a history of severe pain on defaecation and bright red blood on the tissue paper. On examination, you see a skin tag at the anal verge. Per rectum examination is not possible due to severe pain.
Which of the following treatments would you suggest?
A. Analgesic suppositories
B. Excision of skin tag
C. Haemorrhoidectomy D. Incision and drainage
E. Topical GTN ointment

A

E

740
Q

A 34-year-old builder comes to see you in the GP practice complaining of a 3-week history of lower back ache. The pain is worst at the end of the day and partially relieved by lying down. He has no other associated symp- toms of note. Examination is unremarkable.
What would be the most suitable management option in the first instance?
A. Blood tests for ESR and bone profile
B. Lumbar X-ray
C. Refer for MRI of the spine
D. Simple analgesia and bed rest
E. Simple analgesia and gentle mobilization

A

E

741
Q

A 25-year-old woman has been investigated with lower gastrointestinal endoscopy and biopsy following complaints of frequent bloody diarrhoea which is still continuing. The results show diffuse superficial inflamma- tion with ulceration in the rectum only.
Which of the following treatment options would be most suitable in the first instance?
A. Azathioprine
B. Colectomy
C. Mesalazine
D. Prednisolone
E. Surveillance colonoscopy

A

C

742
Q
A 67-year-old woman was gardening at home when she developed sudden- onset lower back pain. She found it difficult to get back into the house before she called an ambulance. She now has pain in both her legs and is unable to pass urine. On examination, there is reduced sensation around the perineum.
What is the most likely diagnosis?
A. Anterior cord syndrome 
B. Brown-Séquard syndrome
C. Cauda equina syndrome 
D. Posterior cord syndrome
E. Syringomyelia
A

C

743
Q

A 46-year-old pre-menopausal woman has a 5 cm lump over her right breast which has been confirmed to be an invasive tumour. She is other- wise fit and well.
Which is the next most appropriate course of action?
A. Mastectomy
B. Radiotherapy
C. Tamoxifen
D. Trastuzumab
E. Wide local excision

A

A

744
Q

A 38-year-old man presents to the emergency department. He says that while playing football he felt like he had been kicked in the back of the ankle and fell to the floor. He now has severe pain in his ankle on weight bearing.
Which of the following is the most likely cause of his problem?
A. Achilles tendonitis
B. Achilles tendon rupture
C. Calcaneal fracture
D. Calcaneal spur
E. Posterior cruciate ligament rupture

A

B

745
Q
A 36-year-old woman presents to the GP with a 2-month history of tremors. This is interfering with her work as a typist and the worry has caused her to lose over a stone in weight over the last few weeks. On examination, the thyroid appears normal although a bruit can be heard overlying it.
What is the most likely diagnosis?
A. De Quervain thyroiditis
B. Graves disease
C. Thyroid adenoma
D. Thyroid storm
E. Toxic multinodular goitre
A

B

746
Q

A 6-year-old boy has a fall onto his outstretched hand. On examination, he is tender over the distal radius. An X-ray confirms a fracture through the distal radius growth plate which includes a metaphyseal and epiphy- seal fragment.
What classification is most suitable for this fracture?
A. Salter–Harris I B. Salter–Harris II
C. Salter–Harris III D. Salter–Harris IV E. Salter–Harris V

A

D

747
Q

A 5-year-old boy is brought into the resuscitation room with acute dif- ficulty breathing. His mother tells you that he has been complaining of a sore throat for the past 2 days. On arrival he is febrile, tachypnoeic, sitting upright and drooling at the mouth. There is no ENT on call in the hospital.
Which of the following would you do as a priority?
A. Assess the back of the throat using a tongue depressor
B. Attempt to perform a fibre-optic laryngoscopy
C. Contact the anaesthetist on call and ask them to attend urgently
D. Give oral antibiotics
E. Request an urgent lateral radiograph of the neck

A

C

748
Q

A 60-year-old woman was involved in a house fire and is brought to the emergency department. She has partial thickness burns over the whole of her left arm and superficial burns covering her back.
Estimate the percentage burn she has sustained.
A. 9%
B. 18%
C. 20%
D. 27%
E. 36%

A

D

749
Q

A 46-year-old woman has been involved in a road traffic collision. Her right knee hit the dashboard of the car. On arrival at the emergency department she is unable to flex her toes on the right side. Examination reveals an absence of the ankle jerk and loss of sensation over the sole of the right foot.
Which of the following nerves is most likely to have been affected?
A. Femoral nerve B. Obturator nerve
C. Sciatic nerve
D. Sural nerve
E. Tibial nerve

A

E

750
Q
An 8-year-old boy is brought into the emergency department by his father with a 1-day history of fever, right-sided abdominal pain and diarrhoea. He has no past medical history of note, but has recently recovered from a cold. On examination, he has a temperature of 39°C and is tender, but not guarding, in the right iliac fossa.
What is the most likely diagnosis?
A. Appendicitis 
B. Crohn disease
C. Coeliac disease D. Gastroenteritis
E. Mesenteric adenitis
A

E

751
Q

A 65-year-old man presents to the GP surgery complaining of increas- ing pain in his back and lower left leg. His mobility is now very limited. On examination, he has a marked kyphosis of the spine and bowing of the left lower limb. Locally the leg is markedly warm. Blood tests are taken.
Which of the following biochemical abnormalities would one expect to find?
A. Hypercalcaemia and hyperphosphataemia B. Hypocalcaemia and hypophosphataemia
C. Hypocalcaemia and raised alkaline phosphatase
D. Raised alkaline phosphatase and normal calcium
E. Reduced parathyroid hormone levels and normal calcium

A

D

752
Q

A 33-year-old woman presents to the surgical outpatient clinic with increasing dysphagia for solids and liquids, retrosternal chest pain on eating, regurgitation of undigested food and weight loss. Oesophageal and gastric biopsy at endoscopy was normal. Oesophageal manometry showed impaired relaxation of the lower oesophageal sphincter and barium swallow revealed a dilated proximal oesophagus with distal narrowing.
Which of the following surgical procedures would be indicated?
A. Billroth I
B. Billroth II
C. Heller myotomy
D. Ramstedt procedure
E. Whipple procedure

A

C

753
Q

A 24-year-old man was involved in a house fire and is brought to the emergency department. He has partial thickness burns over the whole of his front torso and right arm, and superficial burns on his left hand.
Estimate the percentage burn he has sustained.
A. 18%
B. 25%
C. 28%
D. 36%
E. 37%

A

C

754
Q

A 45-year-old man is brought to the emergency department having fallen off a ladder. He complains of pain in the right hip. He is normally fit and well. Hip X-rays show a displaced intracapsular fracture.
Which of the following is the best treatment option?
A. Conservative management
B. Dynamic hip screw
C. Hemiarthroplasty
D. Reduction and internal fixation
E. Total hip replacement

A

D

755
Q

An 18-year-old boy is attacked in a bar fight, during which he is stabbed in the right groin. He attends the emergency department as he is unable to extend his knee afterwards. On examination, the right knee jerk is absent and there is loss of sensation over the front of the thigh and medial aspect of the leg.
Which nerve has most likely been affected?
A. Common peroneal nerve
B. Femoral nerve
C. Obturator nerve D. Sciatic nerve
E. Tibial nerve

A

B

756
Q

A 23-year-old man attends the emergency department after smashing his hand through a window. He has sustained a small laceration to his distal left index finger which you decide to suture.
Which would be the most appropriate anaesthetic agent to use?
A. Bupivacaine
B. Cocaine
C. Lidocaine alone
D. Lidocaine/prilocaine mixture
E. Lidocaine with adrenaline

A

C

757
Q
A 59-year-old woman with a long history of constipation presents with a 4-month history of episodic left-sided abdominal pain relieved by defae- cation. She has also had two episodes of fresh bleeding per rectum. There is no history of weight loss and she is otherwise well. Examination is unremarkable. She is referred for a barium enema.
What features would you expect to find?
A. Apple-core lesion
B. Birds beak appearance
C. Diverticular outpouchings 
D. Non-filling of the appendix
E. Thumb printing of the colon
A

C

758
Q

A 69-year-old man presents to the GP with a 2-month history of epigas- tric pain. He also feels full after eating small amounts of food and has lost a stone in weight over the last 2 months. Examination is unremarkable.
Which of the following is the most likely diagnosis?
A. Duodenal ulcer B. Gastric cancer
C. Gastric ulcer
D. Menetrier disease
E. Pernicious anaemia

A

B

759
Q

A 56-year-old woman presents with progressive difficulty swallowing. She has also noticed increased difficulty in articulating her speech. On examination, you notice that her tongue is small and stiff. She has an exaggerated gag reflex.
Which of the following is the most likely cause of her symptoms?
A. Bulbar palsy
B. Myasthenia gravis
C. Paralysis of the vocal cords
D. Pseudobulbar palsy
E. Scleroderma

A

D

760
Q

A 58-year-old woman is brought to the emergency department hav- ing been found unresponsive by her husband at home. On examina- tion, you note that she is overweight with coarse features. Observations include heart rate 42/min, blood pressure 100/68 mmHg and tempera- ture 34.6°C.
Which of the following would be the most appropriate next step in your management?
A. Intravenous thyroxine
B. Intravenous propranolol
C. Oral propylthiouracil
D. Oral thyroxine
E. Thyroidectomy

A

A

761
Q

A 52-year-old man is brought into hospital by his wife with a 2-day his- tory of confusion. On examination, he is jaundiced and has a distended abdomen with dilated veins around his umbilicus.
Which of the following is the most likely cause of his jaundice?
A. Alcoholic liver disease
B. Autoimmune hepatitis
C. Hepatitis A
D. Hepatitis B
E. Reye syndrome

A

A

762
Q

A 42-year-old woman presents with episodes of severe hunger despite regular meals. She states that if she does not eat she becomes sweaty and agitated. Her symptoms are exacerbated by exercise. As a result of need- ing to eat so regularly she has gained a lot of weight.
Which of the following blood results would correlate with her symptoms?
A. Decreased insulin levels
B. Increased C-peptide levels
C. Increased glucagon levels
D. Increased somatostatin levels
E. Raised fasting blood glucose

A

B

763
Q

A 4-year-old boy is brought to the urology clinic by his mother. She is con- cerned as he is having difficulty urinating properly and is causing a mess. He is otherwise growing and developing normally. On examination, you see the urethral meatus is present on the shaft of the underside of the penis.
Which of the following treatment options would you recommend?
A. Antibiotics
B. Circumcision
C. Intracavernosal injection of phenylephrine
D. Referral to a specialist for corrective surgery
E. Steroid cream

A

D

764
Q

A 3-day-old boy is having increasing abdominal distension and vomiting. He is also noted not to have passed meconium since birth.
Which of the following investigations would be most helpful in con- firming the cause of obstruction?
A. Abdominal X-ray B. Barium enema
C. Rectal biopsy
D. Rectal examination
E. Ultrasound scan

A

C

765
Q

A 13-year-old girl presents to the emergency department following a fall on the outstretched hand. An X-ray of the affected upper limb shows a fracture of the upper ulna with dislocation of the radial head.
Which of the following would be the most appropriate term to describe this fracture?
A. Barton fracture B. Colles fracture
C. Galeazzi fracture
D. Monteggia fracture
E. Smith fracture

A

D

766
Q

A 35-year-old woman presents to the emergency department with a 1-day history of colicky central abdominal pain and vomiting. She last opened her bowels 2 days ago. She has no past medical history of note but tells you she had an operation on her abdomen as a child although she is not sure what this was for. On examination, she looks unwell and moderately dehydrated. The abdomen is distended and tender with hyperactive bowel sounds.
Which of the following is most likely to have caused her symptoms?
A. Adhesions
B. Colorectal tumour
C. Constipation
D. Gallstone ileus
E. Paralytic ileus

A

A

767
Q

A 36-year-old man complains of a 6-month history of intermittent pain in the rectum which is unrelated to stools. The pain tends to occur at night. He denies passing any mucus or blood per rectum and examina- tion is unremarkable.
Which of the following is the most likely diagnosis?
A. Anal fissure
B. Anal fistula
C. Perianal abscess
D. Perianal haematoma
E. Proctalgia fugax

A

E

768
Q
Which term best fits the description of the hernia given below?
A hernia that has two parts, each lying either side of the inferior epi-
gastric artery.
A. Gluteal 
B. Lumbar
C. Maydl
D. Obturator
E. Pantaloon
A

E

769
Q

A 50-year-old woman has been hit by a car and thrown 5 metres. There was probable loss of consciousness for a few minutes. She is brought in by ambulance, immobilized. On arrival she is alert, ori- entated and talking happily. Within a few minutes however she starts to deteriorate rapidly and becomes unconscious with a Glasgow coma score of 5.
Which of the following measures is needed to maintain her airway?
A. Endotracheal intubation
B. Hyperextend the neck
C. Nasopharyngeal airway
D. Oropharyngeal airway
E. Supplemental oxygen

A

A

770
Q
A 48-year-old-woman presents with a long-standing, painless swelling on her right thigh which disappears on lying flat. She is otherwise well. On examination, the swelling is bluish and non-pulsatile. It lies below and lateral to the pubic tubercle.
What is the most likely diagnosis?
A. Femoral hernia B. Inguinal hernia
C. Psoas abscess D. Saphena varix
 E. Sebaceous cyst
A

D

771
Q

A 29-year-old woman presents to the emergency department with a 3-hour history of lower abdominal pain. On examination, there is marked rebound tenderness in the left iliac fossa. She has a heart rate of 92/min and a blood pressure of 134/92 mmHg.
Which of the following investigations would you perform in the first instance?
A. Abdominal ultrasound
B. Abdominal X-ray
C. Cervical swabs D. Urine dipstick
E. Urine pregnancy test

A

E

772
Q
A 27-year-old man presents to the emergency department with sudden- onset pain and swelling in his right knee which occurred while walking. He denies any trauma to the knee. The man describes his knee ‘locking’ at the time, although this sensation is not present now.
What is the most likely diagnosis?
A. Anterior cruciate ligament injury 
B. Chondromalacia patellae
C. Osgood–Schlatter disease 
D. Osteochondritis dissecans
E. Pre-patellar bursitis
A

D

773
Q

A 28-year-old woman comes to see you at the GP practice having noticed a lump in the right breast. She is concerned that it may be cancer. On examination, you find a smooth, mobile, non-tender mass around 2 cm in diameter in the upper outer quadrant of the breast. There are no overlying skin changes.
Which would be the most appropriate action to take?
A. Advise the patient to return if the lump gets bigger or starts becoming painful
B. Prescribe antibiotics
C. Reassure the patient that there is no evidence the lump is cancerous and
discharge her
D. Refer the patient for urgent mammography
E. Refer the patient for urgent ultrasound scan

A

E

774
Q
A 46-year-old woman attends a follow-up appointment at the endocrine surgery outpatient clinic. She had an operation for Cushing disease in the
past. She complains that her skin is becoming darker and denies sun bed use or excess sunlight exposure.
What is the most likely diagnosis?
A. Acromegaly
B. Carcinoid syndrome
C. Conn syndrome
D. Hypoparathyroidism
E. Nelson syndrome
A

E

775
Q

A 73-year-old woman presents to the surgical outpatient clinic with a 3-week history of rectal bleeding, a sensation of incomplete emptying despite defaecation and rectal discomfort. She is found to have a tumour 2 cm from the anal margin.
Which of the following would be the most appropriate operative intervention?
A. Abdominoperineal resection
B. Anterior resection
C. Hartmann procedure
D. Left hemicolectomy
E. Right hemicolectomy

A

A

776
Q
A 43-year-old woman presents to the GP with a lump in the right groin, which she says has been there on and off for a few months, but is now persistent. She has no other symptoms of note. On examination, there is a grape-sized lump below and lateral to the pubic tubercle. It is not tender and there is no cough impulse felt.
What is the most likely diagnosis?
A. Incarcerated femoral hernia 
B. Incarcerated inguinal hernia
C. Obstructed femoral hernia 
D. Strangulated femoral hernia
E. Strangulated inguinal hernia
A

A

777
Q

A 32-year-old man presents to the GP with multiple lesions on his trunk which have appeared over the last few months. Each lesion is raised, firm and rubbery, and causes a tingling sensation when touched.
Which of the following is the most likely diagnosis?
A. Ganglion
B. Granuloma annulare
C. Kaposi sarcoma
D. Neurofibroma
E. Sebaceous cyst

A

D

778
Q

An 82-year-old woman attends the emergency department following a fall. On examination, there is bruising over the left upper arm and the patient is unable to extend the metacarpals on the same side. There is loss of sensation over the lateral dorsal aspect of the hand.
Which nerve has most likely been affected?
A. Anterior interosseous nerve B. Median nerve
C. Posterior interosseous nerve
D. Radial nerve
E. Upper brachial plexus

A

D

779
Q
A 56-year-old woman presents with a lump on the left side of her neck which has been slowly enlarging over the last few months. On examina- tion, there is a 2 cm firm, painless, mobile lump near the angle of the left jaw. She is still able to move her facial muscles.
What is the most likely diagnosis?
A. Chemodectoma
B. Pleomorphic adenoma
C. Salivary duct carcinoma 
D. Salivary duct stone
E. Sternocleidomastoid tumour
A

B

780
Q

A 46-year-old woman presents to the GP complaining of gradually progressive hearing loss, imbalance and tingling on the left side of her face. On examination, the patient admits that a vibrating tuning fork is louder when placed by the ear on the left side, rather than on the mastoid process.
Which of the following is the most likely cause of her symptoms?
A. Acoustic neuroma
B. Ménière disease
C. Otosclerosis
D. Perforated eardrum
E. Presbycusis

A

A

781
Q

A 34-year-old man with a history of renal stones presents with a 3-hour history of right-sided loin pain. On examination, he looks unwell and his
right kidney is palpable. The presence of a large stone is confirmed on X-ray. Blood results show a urea of 25 mmol/L and a creatinine of 400 μmol/L. On his previous admission last month, his urea and electrolytes were normal.
Which of the following would be indicated in the first instance?
A. Extracorporeal shock wave lithotripsy B. Nephrectomy
C. Percutaneous nephrolithotomy
D. Percutaneous nephrostomy
E. Ureteroscopy

A

D

782
Q

A 27-year-old man is brought into the emergency department following an assault. On arrival he has multiple bruises over the left side of his chest and upper abdomen. On examination, he has abdominal tenderness with guarding throughout. His observations include a heart rate of 132/min and blood pressure 86/42 mmHg. A chest X-ray shows a lower rib fracture and elevated diaphragm on the left.
Which of the following is the most likely cause of his symptoms?
A. Hepatic injury
B. Left-sided haemothorax
C. Left-sided pneumothorax
D. Ruptured diaphragm
E. Splenic injury

A

E

783
Q

A 58-year-old man is admitted to hospital with a strangulated paraumbil- ical hernia that requires urgent operative intervention. He has a history of severe depression and is currently under Section for treatment of his mental condition. The patient does not consent to treatment even though he understands his condition, the benefits of having an operation and the potential outcomes of not having it.
Which is the most appropriate course of action?
A. Doctor can consent for the patient
B. Doctor can treat the patient under duty of care
C. Doctor can treat the patient under the Mental Health Act
D. Patient’s refusal to treatment is valid and cannot be overridden
E. Patient’s refusal to treatment is invalid and can be overridden

A

D

784
Q

A 71-year-old man presents to the GP with a painful, enlarging lump just in front of his right ear, which has been present for 2 months. He woke this morning to find that the right side of his face was drooping.
What is the most likely cause of his symptoms?
A. Acute bacterial sialothiasis
B. Parotitis
C. Pleomorphic adenoma
D. Salivary gland calculi
E. Salivary gland carcinoma

A

E

785
Q

A 73-year-old woman presents to the emergency department with abdom- inal pain, vomiting and an inability to open her bowels. On examination, the abdomen is distended and the rectum is empty. The patient’s husband tells you that she has lost a significant amount of weight over the last month and has been having blood in her stool.
Which of the following is the most likely cause of the presentation?
A. Colorectal malignancy
B. Constipation
C. Diverticulitis
D. Inguinal hernia
E. Ulcerative colitis

A

A

786
Q

A 40-year-old woman presents with intermittent episodes of severe right upper quadrant pain associated with meals. She is otherwise well and haemodynamically stable. Examination is unremarkable.
Which of the following investigations would you perform in the first instance?
A. Abdominal ultrasound
B. Abdominal X-ray
C. ERCP
D. MRCP
E. Upper GI endoscopy

A

A

787
Q

A 12-year-old boy presents to the emergency department with left-sided hip pain radiating to the knee. It started after he was running for a bus that morning. On examination, the boy is afebrile, overweight and the left leg is shortened and externally rotated.
Which of the following is the most likely cause of his symptoms?
A. Acute fracture of the hip
B. Perthes disease
C. Septic arthritis
D. Slipped upper femoral epiphysis
E. Spontaneous haemarthrosis

A

D

788
Q

A 53-year-old man presents to the emergency department in the middle of the night complaining of sudden-onset numbness and weakness below the waist. He has also had some urinary incontinence. On examination, his ankle reflexes are absent and he has marked weakness on both dorsal and plantar flexion of the feet. There is reduced anal tone on per rectum examination.
Which of the following is the most appropriate step?
A. Admit for urgent CT spine in the morning
B. Admit for urgent MRI spine in the morning
C. Obtain pelvic X-rays
D. Request urgent urology input
E. Transfer patient to a specialist spinal centre

A

E

789
Q
A 69-year-old man with poorly controlled insulin-dependent diabetes mellitus comes to see you in the urology outpatient department com- plaining of troublesome incontinence. He tells you he does not feel the urge to pass urine.
What is the most likely cause?
A. Atonic bladder
B. Benign prostatic hypertrophy
C. Bladder calculi
D. Detrusor instability
E. Urinary tract infection
A

A

790
Q
A 45-year-old man is brought into the emergency department following a fall from the balcony of his fourth floor flat. On arrival he makes no sound, does not open his eyes to pain and makes no motor response to any stimulus.
What is his Glasgow coma score?
A. 0 
B. 1 
C. 3 
D. 6 
E. 9
A

C

791
Q

An 87-year-old woman with a history of chronic obstructive pulmonary disease with recurrent episodes of bronchitis comes to see you at the GP practice. She has noticed a lump in the lower abdomen when she stands. It does not cause her any concern but she just wanted to get it checked out.
On examination, she is obese and there is a soft, non-tender reducible mass in the lower abdomen. You note lower midline scarring from previ- ous caesarean sections overlying the lump.
What would be the best course of management?
A. Conservative treatment unless symptoms change B. Elective laparoscopic repair
C. Elective open repair
D. Emergency open repair
E. Prompt laparoscopic repair

A

A

792
Q

A 55-year-old man who has been a smoker for over 30 years has been referred to the surgical outpatient clinic with a 6-month history of severe calf pain on walking which is eased by rest. Blood tests requested by the GP are unremarkable.
Which of the following imaging investigations would you send him for?
A. CT scan
B. Digital subtraction angiography
C. Labelled white cell scan
D. MR angiogram
E. Ultrasound Doppler

A

E

793
Q

A 42-year-old woman presents to the emergency department with swell- ing and redness in her left cheek and malaise which has been present for 1 day. On examination, the area of erythema is well demarcated and is hot to touch. Her observations include: heart rate 106/min, blood pressure 108/68 mmHg and temperature 38.6°C.
Which of the following is the best course of management?
A. 5-Fluorouracil
B. Analgesia
C. Intravenous antibiotics
D. Oral antibiotics
E. Surgical debridement

A

C

794
Q

A 68-year-old man is brought into the resuscitation room having been found collapsed outside a shop. On examination, he is pale, drowsy, and has a pulsatile mass in the abdomen. His observations include a heart rate of 126/min and a blood pressure of 86/38 mmHg.
Which of the following would be most appropriate?
A. Abdominal ultrasound
B. Abdominal X-ray
C. Aggressive fluid resuscitation
D. CT abdomen
E. Laparotomy

A

E

795
Q

A 64-year-old woman presents to the emergency department with an acutely painful left leg. She is complaining that it feels ‘cold and numb’. There is no history of trauma and she has never had similar symptoms in the past.
Which of the following would you not expect to be found on examination?
A. A tender, pulsatile, expansile mass in the abdomen
B. Absent left leg pulses
C. ECG showing an irregular rhythm
D. Pan-systolic murmur on auscultation
E. Wet gangrene

A

E

796
Q

A 57-year-old man presents with a progressive deformity of his right hand which has been present for over a year. On examination, he has thicken- ing over the palm with fixed contracture of the ring and little finger. He tells you that the deformity is limiting his day-to-day work as a computer programmer.
Which of the following management options would be recommended?
A. Conservative management
B. Physiotherapy
C. Splinting
D. Steroid injections
E. Surgery

A

E

797
Q
A 46-year-old woman attends her general practice complaining of low mood and tiredness. She has had this for a few months and has had to take time off work. On further questioning she admits to being consti- pated and to having gained 2 stone in weight over the previous month. On examination, no abnormality is apparent in the neck and no lymphade- nopathy is palpable.
What is the most likely diagnosis?
A. Haemorrhage into a cyst 
B. Hashimoto thyroiditis
C. Medullary carcinoma 
D. Myxoedema coma
 E. Primary myxoedema
A

E

798
Q
A 3-year-old boy is brought to the GP practice by his mother. He has been complaining intermittently about abdominal pain. His mother feels that his abdomen is distended. On examination, there is a large mass on the left side of the abdomen which does not cross the midline.
What is the most likely diagnosis?
A. Hepatoblastoma B. Lipoma
C. Nephroblastoma D. Neuroblastoma
E. Polycystic kidney
A

C

799
Q

A 53-year-old woman has fallen onto her outstretched right hand, result- ing in an undisplaced fracture of the distal radius and ulna. She was previously fit and well. She wants to know how long it will take for the fracture to heal.
How long would you expect this fracture to take to heal?
A. 1 week
B. 3 weeks
C. 6 weeks
D. 12 weeks
E. 16 weeks

A

C

800
Q

Tented T waves on electrocardiograms are thought to reflect hyperkalae- mia. A new proposed definition of tented T waves is ‘a T wave that is taller than 2 large squares’. A study looking at the feasibility of this definition found 1000 ECGs: 200 with tented T waves (according to the proposed definitions) and 800 without. The study found that of the 200 people with ECGs with tented T waves, 100 had hyperkalaemia. Of the 800 people without tented T waves, a further 100 had hyperkalaemia.
What is the specificity of this definition of tented T waves with relation to hyperkalaemia?
A. 12.5%
B. 25.0%
C. 50.0%
D. 87.5%
E. 95.0%

A

D

801
Q

A 55-year-old woman presented to the emergency department with a frac- ture of the distal radius following a low-velocity injury. She was referred for a DEXA scan, which showed her bone density to be three standard deviations below the mean.
Which of the following has resulted in this patient’s fracture?
A. Osteogenesis imperfecta
B. Osteomalacia
C. Osteopenia
D. Osteopetrosis
E. Osteoporosis

A

E

802
Q

A 74-year-old man presents to the emergency department with a 6-hour history of colicky lower abdominal pain accompanied by gross abdominal distension. He has had similar episodes in the past that have been relieved by opening his bowels. Today he is not even able to pass flatus. Abdominal X-ray shows a grossly dilated loop of large bowel.
Which of the following is the most likely diagnosis?
A. Adhesions
B. Faecal impaction
C. Intussusception D. Sigmoid volvulus
E. Strangulated inguinal hernia

A

D

803
Q

You are called to see a 72-year-old man who has fallen on the ward two days post-laparotomy. On arrival, the patient appears to be unconscious and his breathing is noisy and laboured. His oxygen saturations are 76% on air.
Which of the following would be the next most appropriate step in your management?
A. Endotracheal intubation
B. Head-tilt chin-lift
C. Needle cricothyroidotomy
D. Supplemental oxygen
E. Surgical tracheostomy

A

B

804
Q

A 56-year-old woman presents to the emergency department with an acutely painful lump in the right groin. She has vomited six times in the last hour and is also complaining of worsening abdominal pain. On examination, there is a grape-sized lump below and lateral to the pubic tubercle. It is warm, red and exquisitely tender. No cough impulse is felt.
What would be the best course of management?
A. Elective repair
B. Emergency repair
C. Observation alone
D. Prompt repair
E. Urgent repair

A

B

805
Q

A 56-year-old man is having a spinal anaesthetic for a lower limb opera- tion as he has been told he is not fit enough for a general anaesthetic.
Which would be the most appropriate local anaesthetic agent?
A. Bupivacaine alone B. Lidocaine
C. Lidocaine/prilocine mixture
D. Lidocaine with adrenaline
E. Prilocaine

A

A

806
Q

A 65-year-old man is on the surgical ward being treated for pancreatitis. Initially he appeared to be improving, however on day 4 he develops fever and increasing epigastric pain. His serum C-reactive protein is noted to be rising.
Which of the following is the most likely cause?
A. Ascites
B. Gastrointestinal haemorrhage
C. Ileus
D. Infective pancreatic necrosis
E. Pancreatic pseudocyst

A

D

807
Q

A 33-year-old man is brought into the emergency department following an assault with a baseball bat. He has bruising over the abdomen and right loin and complains of abdominal pain. On examination, his chest is clear with good air entry but his abdomen is rigid. On arrival, his heart rate is 140/min and blood pressure 90/60 mmHg. He is given O-negative blood in the resuscitation room but his observations remain unchanged.
What would be the next step in his management?
A. Diagnostic peritoneal lavage B. Laparotomy
C. Urgent CT scan
D. Urgent intravenous urogram
E. Urgent ultrasound scan

A

B

808
Q

A 68-year-old man presents with acute lower chest and upper abdominal pain following an episode of vomiting. On examination, he has marked tenderness in the epigastrium and you note the presence of subcutaneous emphysema. ECG shows a sinus tachycardia with no ST segment changes, and a chest X-ray demonstrates mediastinal air.
Which of the following is the most likely diagnosis?
A. Boerhaave syndrome
B. Chest infection
C. Mallory-Weiss tear
D. Myocardial infarction
E. Pulmonary embolus

A

A

809
Q

A 34-year-old woman is being investigated for jaundice. Investigations show elevations of bilirubin, aspartate transaminase and alkaline phos- phatase in the blood. Gamma GT levels are normal. There is no urobilino- gen in the urine.
Which of the following conditions would be compatible with the above results?
A. Gallstones
B. Gilbert syndrome
C. Hereditary spherocytosis
D. Liver metastasis
E. Viral hepatitis

A

A

810
Q
A 37-year-old woman presents to the GP with a 3-week history of fre- quent, loose motions. She is very tired and complains of muscle weakness.
A subsequent colonoscopy demonstrates a 1.5 cm sessile growth that has multiple projections. The remainder of the colon was unremarkable.
What is the most likely diagnosis?
A. Adenomatous polyp 
B. Colorectal carcinoma
C. Infective colitis
D. Pseudomembranous colitis
E. Villous adenoma
A

E

811
Q

A 45-year-old man presents with episodic upper abdominal pain, severe heartburn, episodes of dark vomitus and diarrhoea. He has been tak- ing over-the-counter indigestion medication with no relief. On endos- copy, he has multiple ulcers throughout the duodenum, some of which are large.
Which of the following is most likely to have caused this?
A. Alcohol
B. Aspirin use
C. Gastrinoma
D. Helicobacter pylori infection
E. Smoking

A

C

812
Q

Which term best fits the description of hernia given below?
A hernia that contains a ‘W’ loop of intestine within its sac.
A. Gluteal
B. Inguinal
C. Maydl
D. Obturator
E. Sciatic

A

C

813
Q

A 65-year-old man presents to the emergency department with a 3-hour history of left loin pain radiating to the left groin. He is known to have benign prostatic hypertrophy and recurrent urinary tract infections. The intravenous urogram (IVU) shows a standing column on the left.
Which of the following types of stone is most likely to have caused his symptoms?
A. Calcium
B. Cholesterol
C. Cysteine
D. Magnesium ammonium phosphate
E. Uric acid

A

D

814
Q

A 34-year-old man is receiving a blood transfusion after a large upper gastrointestinal bleed. Soon after his transfusion starts he develops a high fever and chest pains. He is found to have a heart rate of 120/min and a blood pressure of 102/68 mmHg.
Which type of hypersensitivity reaction is this?
A. Type I hypersensitivity
B. Type II hypersensitivity
C. Type III hypersensitivity
D. Type IV hypersensitivity
E. Type V hypersensitivity

A

B

815
Q

A 39-year-old man presents to the emergency department with a painful lump in the groin. He tells you that this lump has been present on and off for a few months and has previously been asymptomatic. Today, the lump has become painful. On examination, there is an erythematous, tender, irreducible lump above and medial to the pubic tubercle. The patient denies abdominal pain, vomiting or constipation.
Which is the best descriptive term for the hernia above?
A. Incarcerated hernia
B. Perforated hernia
C. Richter hernia
D. Sliding hernia
E. Strangulated hernia

A

C

816
Q

A 65-year-old woman presents to the emergency department with right wrist pain following a fall onto an outstretched hand. An X-ray shows a fracture of the distal radius with volar displacement of the distal frag- ment. There is no intra-articular involvement.
Which of the following would be the most appropriate term to describe this fracture?
A. Barton fracture B. Colles fracture
C. Galeazzi fracture D. Monteggia fracture
E. Smith fracture

A

E

817
Q

A 45-year-old woman attends the orthopaedic follow-up clinic to receive the results of a knee X-ray. She has been getting increasing pain in the left knee, aggravated by walking.
Which of the following changes on X-ray would most suggest a diagno- sis of osteoarthritis?
A. Joint subluxation
B. Juxta-articular erosions
C. Narrowed joint space
D. Soft tissue swelling
E. Subchondral sclerosis

A

E

818
Q
A 4-year-old boy is brought into hospital by his mother. He has been hav- ing episodes of intermittent high fever associated with a generalized non- itchy, pink rash over the body and swelling of the knees on and off for the last 2 weeks.
What is the most likely diagnosis?
A. Juvenile pauciarticular arthritis 
B. Juvenile polyarticular arthritis
C. Septic arthritis D. Still’s disease
E. Transient synovitis
A

D

819
Q

A 22-year old woman presents to the GP complaining of a 2-week his- tory of milky discharge from both her breasts. She is otherwise well. Clinical examination of the breasts is unremarkable and a pregnancy test is negative.
Which of the following investigations would be indicated in the first instance?
A. Dopamine levels
B. Fine needle aspiration
C. Mammogram
D. MRI of the breast
E. Prolactin levels

A

E

820
Q

You are called to see a 52-year-old woman who has just undergone a thy- roidectomy. She complains of a 20-minute history of neck pain and short- ness of breath. A rapidly expanding mass is noted around the operation site.
Which of the following would you do next?
A. Anaesthetic review
B. Bedside opening of incision
C. Book next available theatre slot
D. Intravenous antibiotics
E. Laryngoscopy

A

B

821
Q

A 49-year-old woman presents with a 4-month history of perianal itching with occasional mucous discharge. She is otherwise well. On examina- tion, large fleshy pink lesions are seen around the anus.
What is the most likely diagnosis?
A. Condylomata lata
B. Condylomata acuminata
C. Anal carcinoma D. Rectal prolapse E. Haemorrhoids

A

B

822
Q
A 56-year-old man presents to the GP with a lesion on the tip of his right middle finger which has developed over the past week since he pricked his finger on a thorn in the garden. On examination, the lesion is 1 cm in size, dark red in colour and bleeds easily.
What is the most likely diagnosis?
A. Cavernous haemangioma 
B. Ganglion
C. Granuloma annulare 
D. Kaposi sarcoma
E. Pyogenic granuloma
A

E

823
Q
A 14-year-old girl presents to the GP surgery complaining of right knee pain which occurs after gymnastics training. She is otherwise well. On examination, there is a full range of movement at the right knee joint and a small tender lump over the upper tibia.
What is the most likely diagnosis?
A. Baker cyst
B. Bipartite patella
C. Chondromalacia patellae 
D. Dislocation of the patella
E. Osgood–Schlatter disease
A

E

824
Q
A 20-year-old man presents to the GP with a 2-day history of severe pain and swelling in his right testicle. On examination, there is a purulent discharge present at the urethral meatus and the right testicle is tender to palpation.
What is the most likely diagnosis?
A. Epididymo-orchitis 
B. Mumps orchitis
C. Testicular cancer 
D. Testicular torsion
E. Varicocele
A

A

825
Q
A 65-year-old man has been brought into hospital by his wife. She is concerned as he has become increasingly yellow in colour and has unin- tentional weight loss. The patient denies any pain. On examination he is jaundiced and has a palpable mass in the right upper quadrant.
What is the most likely diagnosis?
A. Cholangiocarcinoma B. Gallstones
C. Gastric carcinoma
D. Hepatocellular carcinoma
E. Pancreatic carcinoma
A

E

826
Q

A 21-year-old man is brought into the emergency department with penile pain and swelling following a fall onto the crossbar of a bicycle. On exam- ination, the penis is oedematous and bruised and there is blood at the urethral meatus. He is otherwise stable.
Which of the following investigations is indicated?
A. Bladder scan and urethral catheterization
B. CT abdomen and pelvis
C. Cystoscopy
D. Intravenous urogram
E. Retrograde urethrogram

A

E

827
Q
A 6-year-old girl is brought to the GP by her father with a smooth lump in the midline of her neck. She is otherwise well. On examination, there is a 1 cm painless, fluctuant swelling which moves upwards on swallowing and on tongue protrusion.
What is the most likely diagnosis?
A. Branchial cyst 
B. Chemodectoma C. Cystic hygroma
D. Thyroid adenoma E. Thyroglossal cyst
A

E

828
Q

A 44-year-old man presents to the GP practice complaining of a 2-month history of painless, purulent, foul-smelling discharge from his right ear.
His hearing is gradually getting worse. On examination, there is no obvi- ous abnormality externally.
Which of the following is the most likely cause of his symptoms?
A. Acute otitis media
B. Cerumen
C. Cholesteatoma D. Glue ear
E. Swimmer’s ear

A

C

829
Q

A 33-year-old man who was travelling on his motorbike has been hit by a car at 50 miles per hour. He is brought into the emergency department by his friend and it is noted that his left leg appears deformed. His pulse is 120/min and his blood pressure 100/65 mmHg.
What is the next step in your management?
A. Assess the patient’s breathing B. Give blood
C. Give fluids
D. Immobilize the patient
E. Splint the left leg

A

D

830
Q

A 27-year-old man presents to the emergency department with severe epi- gastric pain which started soon after eating his lunch. On examination, the upper abdomen is rigid. The patient says he has recently been suffer- ing from indigestion which started after medical treatment for a meniscal injury to his knee.
Which of the following medications could have contributed to his condition?
A. Aspirin
B. Codeine
C. Gaviscon
D. Omeprazole
E. Paracetamol

A

A

831
Q
A 62-year-old man presents to the GP with a lesion on his left forehead that has been present for 6 months. It has recently begun to ulcerate. On examination, there is a 1 cm raised, hyperkeratotic, crusty lesion with raised edges.
What is the most likely diagnosis?
A. Actinic keratosis
B. Basal cell carcinoma
C. Bowen disease
D. Keratoacanthoma
E. Squamous cell carcinoma
A

E

832
Q

A 37-year-old woman presents to the emergency department following a faint after standing up from a sitting position. She did not lose conscious- ness during this episode but reported feeling ‘dizzy and lightheaded’. She has no significant past medical history. On examination you notice mul- tiple areas of skin depigmentation but increased pigmentation in the pal- mar creases and on the elbows.
Which of the following is the best management for her condition?
A. Desmopressin
B. Hydrocortisone and fludrocortisone
C. Octreotide
D. Spironolactone
E. Thyroidectomy

A

B

833
Q
A 7-day-old preterm boy develops vomiting and bloody diarrhoea. On examination, his abdomen is grossly distended and discoloured. His tem- perature is 39.5°C.
What is the most likely diagnosis?
A. Exomphalos
B. Gastroenteritis
C. Gastroschisis
D. Meconium ileus
E. Necrotizing enterocolitis
A

E

834
Q

A 4-year-old girl is referred to the urology clinic by the GP as she has been having recurrent urinary tract infections. Examination of the child is unremarkable.
Which of the following is most likely to be causing the problem?
A. Hydronephrosis B. Pelvic kidney
C. Posterior urethral valve
D. Renal agenesis
E. Vesica-ureteric reflux

A

E

835
Q

An 81-year-old woman from a nursing home is brought into the emer- gency department following a fall. She is complaining of pain in her left
hip. On examination, her left leg is shortened and externally rotated. The home tells you this woman has dementia but is able to mobilize indepen- dently with a frame. An X-ray shows a displaced intracapsular femoral neck fracture.
Which of the following would be the most suitable management option?
A. Allow the patient to try to mobilize
B. Dynamic hip screw
C. Hemiarthroplasty
D. Open reduction and internal fixation
E. Total hip replacement

A

C

836
Q

A 45-year-old woman presents to the emergency department with a 12-hour history of worsening right upper abdominal pain and vomiting. On examination, she is locally peritonitic in the right upper quadrant. Her temperature is 38.1°C.
Which of the following treatment options would be indicated in the first instance?
A. Cholecystectomy
B. Intravenous antibiotics
C. Intravenous steroids
D. Intravenous proton pump inhibitor
E. Percutaneous drainage of the gallbladder

A

B

837
Q

A 59-year-old man presents with an 8-month history of urinary hesitancy, poor stream and the sensation of incomplete bladder emptying. He is oth- erwise well. On examination, his prostate is homogeneously enlarged and smooth.
Which of the following interventions would be indicated?
A. Bladder neck incision
B. Brachytherapy
C. External beam radiotherapy
D. Radical prostatectomy
E. Transurethral resection of prostate

A

E

838
Q

A new blood test has been produced to help diagnose suspected tumours of the membranous urethra. Of a study sample of 100 patients, 20 are known to have cancer of the membranous urethra. The blood test sug- gested that 30 people had the tumour, but of these only 15 were true positives.
What is the positive predictive value of this test?
A. 25%
B. 50%
C. 75%
D. 90%
E. 100%

A

B

839
Q

A 45-year-old male coach driver presents to the emergency department with a 3-day history of a swollen left leg and some shortness of breath. On examination, the left calf is swollen and tender. The pedal pulses are palpable and the foot is warm.
What would be the most appropriate initial management?
A. Elevation, rest and NSAIDs
B. Emergency surgery
C. Intravenous heparin
D. Subcutaneous low molecular weight heparin
E. Warfarin

A

D

840
Q
A 72-year-old man who has no past medical history of note has been referred to you by his GP. His GP found an incidental non-tender expans- ile mass in his abdomen. An ultrasound scan demonstrates an abdominal aortic aneurysm that is 5 cm in diameter.
What is the best course of management?
A. Admit for emergency repair
B. Commence on antihypertensives and advise to see GP if he develops
abdominal pain
C. Organize an elective repair
D. Repeat an ultrasound scan in 3 months
E. Repeat an ultrasound scan in 1 year
A

D

841
Q

A 34-year-old man is brought to the emergency department. Three days ago he suffered an insect bite to the upper leg; he is now complaining of severe pain in the thigh, fever and vomiting. On examination of the thigh, there is marked, spreading erythema with areas of blistering and skin necrosis.
Which of the following is the best course of management?
A. Intravenous antibiotics alone
B. Intravenous antibiotics and surgical debridement
C. Intravenous steroids
D. Oral antibiotics
E. Skin biopsy

A

B

842
Q
A 56-year-old man presents to the emergency department with a 3-hour history of severe abdominal pain and vomiting. On examination, the abdomen is rigid. A CT scan confirms free air in the peritoneum and a perforation of a sigmoid diverticulum.
What is the most appropriate operation?
A. Abdominoperineal resection 
B. Extended right hemicolectomy
C. Hartmann procedure 
D. Left hemicolectomy
E. Right hemicolectomy
A

C

843
Q

A 59-year-old woman has recently had a resection of a sigmoid adenocarcinoma. The pathology report said that the tumour invaded through the bowel wall but there was no evidence of lymph node involvement.
Which of the following best describes this stage of tumour?
A. Dukes A
B. Dukes B
C. Dukes C1
D. Dukes C2
E. Dukes D

A

B

844
Q

A 37-year-old woman presents with a 2-day history of increasing pain and swelling of the right middle finger. She tells you she sustained a minor cut to the inner surface of the finger 4 days ago. On examination, the finger is grossly swollen, red and held in the flexed position.
Which of the following is the most likely diagnosis?
A. Fascial space infection
B. Felon
C. Infective tenosynovitis
D. Paronychia
E. Whitlow

A

C

845
Q

A 27-year-old man presents to the emergency department following an episode of vomiting blood. On examination he is haemodynamically stable and is noted to have multiple freckles around his lips and mouth. A subsequent endoscopy demonstrates multiple polyps throughout the duodenum which are confirmed as benign.
What is the most likely morphology of the polyps?
A. Adenomatous polyp
B. Hamartomatous polyp
C. Juvenile polyp
D. Metaplastic polyp
E. Pseudo-polyp

A

B

846
Q

A 64-year-old man presents to the GP practice with a 2-month history of intermittent episodes of lower back pain radiating to both legs on walk- ing. His symptoms are relieved by sitting or leaning forward. He has no back pain at rest. Examination is unremarkable.
Which of the following is the most likely cause of his symptoms?
A. Intermittent claudication
B. Lumbar spondylosis
C. Lumbar spondylolisthesis
D. Spinal stenosis
E. Spinal tumour

A

D

847
Q

A 24-year-old woman with no past medical history of note comes to see you at the GP practice complaining of a 2-day history of urinary fre- quency together with a stinging sensation on passing urine. She is sys- temically well.
Which of the following organisms is most likely to have caused her symptoms?
A. Candida albicans B. Escherichia coli C. Herpes simplex
D. Proteus mirabilis
E. Pseudomonas aeruginosa

A

B

848
Q

A 27-year-old man was admitted to the ward after sustaining a right tibia and fibula fracture in a road traffic accident. He is currently in an above- knee backslab with the leg resting on a pillow, awaiting consultant review in the morning. You are called to the ward by the nurse saying the patient is in severe pain despite having morphine 2 hours ago. When you arrive at the ward the patient is crying in pain and complaining of an inability to move his toes.
Which of the following is the most appropriate measure to take?
A. Administer further analgesia and call senior orthopaedic doctor on call B. Complete the cast to give more stability to the fracture
C. Elevate the limb further
D. Prescribe more regular analgesia and review in the morning
E. Remove backslab as a temporary measure and review in the morning

A

A

849
Q

A 17-year-old boy presents with severe left shoulder pain following a tackle during a rugby match. On examination, the contour of the left shoulder is flattened and the humeral head is palpable just under the clavicle. There is also sensory loss at the upper lateral aspect of the arm.
Which nerve has most likely been affected?
A. Accessory nerve B. Axillary nerve
C. Long thoracic nerve
D. Median nerve
E. Radial nerve

A

B

850
Q

A 27-year-old man is admitted to the high dependency ward having sus- tained a significant head injury. Two days later he starts vomiting blood.
Which of the following is the most likely cause?
A. Curling ulcer
B. Cushing ulcer
C. Decubitus ulcer D. Dendritic ulcer
E. Martorell’s ulcer

A

B